Skip to main content

Full text of "Notebook on practical solid or descriptive geometry, containing problems with help for solutions"

See other formats


NOTE-BOOK 


ON 


PRACTICAL    SOLID 


OR 


DESCRIPTIVE    GEOMETRY 


NOTE-BOOK 

ON 

PRACTICAL  SOLID 

OR 

DESCRIPTIVE  GEOMETRY 

CONTAINING 

PROBLEMS   WITH  HELP  FOR  SOLUTIONS. 

BY 

J.   H.   EDGAR,   M.A, 

LECTURER  ON  MECHANICAL  DRAWING  AT  THE  ROYAL  SCHOOL  OF  MINES,  LONDON  ; 
/  ND   FORMERLY  LECTURER  ON  GEOMETRICAL  DRAWING,  KING's  COLLEGE,  LONDON  ; 

AND 

G.   S.   PRITCHARD, 

1  ATE  PROFESSOR  OF  DESCRIPTIVE  GEOMETRY,  ROYAL  MILITARY  ACADEMY,  WOOLWICH. 

FOURTH  EDITION, 
BY 

ARTHUR   G.   MEEZE, 

."  3SISTANT  LECTURER  ON  MECHANICAL  DRAWING,  ROYAL  SCHOOL  OF  MINES,  LONDON. 

Hontron : 
MACMILLAN  AND   CO. 

1880. 

\_The  Right  of  Translation  is  reserved,'] 


Camftrftge : 


PRINTED      BY     C.      J.     CLAY,     M.A. 
AT    THE    UNIVERSITY     PRESS. 


t 


0^ 


PREFACE. 

In  teaching  a  large  class,  if  the  method  of  lecturing  and 
demonstrating  from  the  black-board  only  is  pursued,  the  I 
more  intelligent  students  have  generally  to  be  kept  back, 
from  the  necessity  of  frequent  repetition,  for  the  sake  of  1/ 
the  less  promising ;  if  the  plan  of  setting  problems  to  each 
pupil  is  adopted,  the  teacher  finds  a  difficulty  in  giving 
to  each  sufficient  attention.  A  judicious  combination  of 
both  methods  is  doubtless  the  best,  though  this  is  not 
always  easy  of  attainment  in  working  with  numbers — the 
use  of  this  book  may  help  in  accomplishing  it. 

It  is  suggested  that  at  the  beginning  of  a  chapter,  and  in 
some  cases  with  each  problem,  the  teacher  should  give  a 
black-board  explanation,  carefully  pointing  out  any  fresh 
steps,  before  sending  his  pupils  to  their  work.  The  number  ''" 
of  examples  in  each  chapter  to  be  worked  out  by  the 
student  is,  of  course,  left  to  the  teacher's  judgment  of  the 
progress  and  requirements  of  his  pupil. 

The  student  will  here  be  assisted,  it  is  beheved,  because 
he  will  find  help  givenVhen  necessary,  and  thus  will  often 
be  able  to  go  on  with  his  work  by  himself,  with  the  satisfac- 
tion of  also  feeling  that  he  is  progressing  systematically. 
Moreover  he  will  become  practised  in  dealing  with  written 
questions — a  point  not  to  be  lost  sight  of  by  those  who  are 
preparing  for  Examinations, 

The  Second  Edition  has  been  enlarged  by  the  addition 
of  chapters  on  the  straight  line  and  plane  with  explanatory 
diagrams  and  exercises,  on  tangent-planes,  and  on  the  cases 
of  the  spherical  triangle. 

E.  G.  b 


vi  PREFACE. 

It  is  hoped  that  the  work,  thus  rendered  more  complete, 
i^may  prove  still  more  useful  as  a  class-book  and  means  of 
V  self -instruction  to  the  various  and  constantly  increasing 
classes  of  students  for  whom  it  is  designed.  It  was  originally 
intended  as  an  aid  in  teaching  the  Mechanical  Drawing 
Class  at  the  Royal  School  of  Mines  from  Professor  Bradley's 
Elements  of  Practical  Geometry.  The  authors  of  this  work 
were  associated  with  him  in  his  duties  at  King's  College, 
London,  and  the  Royal  Military  Academy,  and  learnt 
practically  the  value  of  his  treatise;  but  the  cost  of  that 
work  has  rendered  it  inaccessible  to  many  for  whom  the 
present  book  may  be  available. 

A  greater  number  of  diagrams  have  not  been  added,  in 
order  that  students  may  be  thrown  upon  their  own  resources,/ 
and  encouraged  to  consider  the  principles  upon  which  their 
work  proceeds,  more  than  they  would  probably  do  if  there 
were  figures  always  at  hand  for  reference. 

October,   1871. 


ADVERTISEMENT  TO  THE  FOURTH  EDmON. 

A  STEADY  demand  for  the  "Note-Book"  having  shown 
that  the  principles  on  which  it  was  designed  met  with  the 
recognition  of  teachers  and  students,  the  successive  editions 
have  been  enlarged  to  satisfy  the  requirements  of  those 
competing  in  the  various  public  examinations,  as  well  as  to 
adapt  it  to  the  gradual  development  of  the  class  at  the 
Royal  School  of  Mines. 

In  preparing  the  Fourth  Edition  the  original  scope  of  the 
book  has  been  considerably  extended.  The  aim  being  to 
make  it,  without  prejudice  to  the  elementary  character  of 
its  earlier  portions,  an  exhaustive  vade-mecum  for  the  work-  ^ 
ing  student.  Attention  may,  therefore,  be  called  to  the 
following  points  in  the  treatment  of  the  present  edition 
which  have  consequently  become  more  prominent : — 

1.  While  retaining  its  original  form  of  a  classified 
collection  of  "Problems  with  Help  for  Solutions,"  the  book 
has  been  placed  upon  a  more  independent  and   scientific 

'•  footing ;  by  adding  the  necessary  definitions  and  theorems, 
and  introducing  a  large  amount  of  matter  of  a  general 
character,  not  elsewhere  accessible,  in  a  cheap  and  compendi- 
ous form,  to  the  majority  of  English  students. 

2.  A  large  number  of  carefully  selected  problems  have 
been  added  to  illustrate  the  leading  principles  and  afford 

^  specific  exercises  for  self-examination,  while  in  writing  the 
solutions  the  constructions  have  been  studiously  varied  so 
as  to  bring  together  as  many  as  possible  of  the  methods 
likely  to  be  of  practical  utility  to  the  scientific  draughts-  V 
man. 


viii      AD  VER  TI SEME  NT  TO  FO  UR  TH  EDITION. 

3.  Diagrams  have  been  purposely  eschewed  and  a 
systematic  attempt  made  to  reduce  the  whole  subject,  as 
far  as  possible,  to  a  purely  verbal  form — sufficient  description 
of  the  various  constructions  being  given  to  enable  a  student 
who  works  steadily  through  the  earlier  chapters  to  build  up, 

h  under  his  own  hand  and  eye,  illustrative  drawings  for 
himself.  The  verbal  formulae,  besides  being  much  less 
fatiguing  to  peruse  than  the  plates  usually  accompan3dng 
books  on  this  subject,  soon  become  immeasurably  more 
powerful  and  suggestive  helps,  and  leave  a  wider  field  for 
the  exercise  of  the  learner's  ingenuity.     Moreover,  the  style 

\J  of  treatment  adopted'  has  rendered  it  possible  to  compress 
into  a  very  small  compass  a  quantity  of  matter  which  would 
otherwise  have  augmented  the  bulk  and  cost  of  the  book  so 
as  to  have  placed  it  out  of  the  reach  of  many  whose  wants 
it  is  intended  to  supply. 

4.  Some  of  the  more  advanced  problems  have  been 
dealt  with   in  general   terms   and  very  briefly,  experience 

1/  having  shown  that  when  a  student's  own  thought  or  theoreti- 
cal studies  have  prepared  him  for  the  necessary  geometrical 
conceptions,  these  higher  developments  are  really  the  most 
easy,  and  a  hint  is  almost  all  that  is  required  as  a  starting- 
point  for  fresh  knowledge — which  is  not  so  much  learned  as 

V  self-evolved  when  once  the  mind  is  set  upon  the  proper 
track. 


June,  1880. 


CONTENTS. 

PAGE 

Introductory  Explanations         .        .        .        .        •        •  ^ 

Definitions  and  Theorems 3 

Definitions  Illustrated  [Notation) 12 

Orthographic  Illustrations  [Points,  Lines,  and  Planes)      .  16 

Memoranda  for  Working  the  Problems  ....  20 

CHAPTER  I. 
Solids  in  Simple  Positions 2t 

CHAPTER  II. 
Combinations  and  Groups  of  Solids 38 

CHAPTER  III. 

Problems  on  the  Straight  Line  and  Plane     ...      42 
Exercises     ......         ...         .         .86 

CHAPTER   IV. 

Solids   with  the  Inclinations  of  the  Plane  of  One 

Face  and  of  One  Edge  or  Line  in  that  Face  given    .      92 

CHAPTER  V. 

Solids  with  the  Inclinations  of  Two  Adjacent  Edges 

or  Lines  given 97 

CHAPTER  VI. 

Solids  with  the  Inclinations  of  Two  Adjacent  Faces 

GIVEN  .  . 99 


X  CONTENTS. 

CHAPTER  VII. 

PAGE 

Properties  of  Curved  Surfaces  and  Problems  Based 

THEREON 103 

Problems  on  the  Projection  of  Curved  Surfaces: —  105 

The  Sphere     .       .         .         .         .         .         .         .         .  •     •  05 

The  Cylinder         .         .         .         .         .         .         .         .  .112 

The  Cone      .         .         .         .         .         .         .         .         .  .119 

Tangent-Planes  and  Normals  to  Curved  Surfaces  : —       125 

General  Remarks  .         . 125 

Tangent- Planes  to  Cones       .  .         .         .         .         .         -127 

Tangent- Planes  to  Cylinders  .         .         .         .         .         •     '3^ 

Tangent- Planes  to  Spheres     .  .         .         .         .         .         •     '  37 

Problems  on  Surfaces  of  Revolution       ....     144 

Tangent-Planes  and  Normals  to  Surfaces  of  Revolu- 
tion    .        . 149 

The  Hyperboloid  of  Revolution  of  One  Sheet      .        .150 

Undevelopable  Ruled  Surfaces  generally     .        .        -154 

CHAPTER  VIII. 

Sections  by  Oblique  Planes 158 

Preliminary  . .158 

General  Pemarhs 160 

Problems 161 


CONTENTS. 


CHAPTER   IX. 

PAGE 

Intersections 173 

General  Remarks ,         .         .         .         .         .         .         .         •     ^73 

Forms  Bounded  by  Plane  Surfaces  .         .         .         .         -174 

Forms  Bounded  by  Curved  Surfaces 182 

Combination   of   Curved  Forms   zvith    Forms  Bounded    by 

Plane  Surfaces 198 

Shadows        ..........     201 

CHAPTER   X. 
Figured  Projections  and  Scales  of  Slope       .       .        .    208 

CHAPTER  XI. 

Trimetric  Methods  of  Projection 215 

Trimelric  Projection     .         .         ■         .         .         .         .         .216 

Isometric  Projection .220 

Axial  Projection  .        .        .        .        .        .         .         .        .221 

Trihedral  Projection     .         .         .         .         .         .         .         .224 

CHAPTER  XII. 
Miscellaneous  Problems      . 228 

CHAPTER  XIII. 

Solutions  of  the  Trihedral  Angle  {^'Spherical  Triangles''^)    250 


ELEMENTARY 
SOLID  OR  DESCRIPTIVE  GEOMETRY. 


INTRODUCTORY    EXPLANATIONS. 


"The  object  of  Descriptive  Geometry  is  the  invention  of  methods 
by  which  w^e  may  represent  upon  a  plane  having  only  two  dimensions, 
namely  length  and  breadth,  the  form  and  position  of  a  body  which  pos- 
sesses three  dimensions,  namely  length,  breadth,  and  height." 

Hall's  Elements  of  Descriptive  Geometry. 


For  this  purpose  two  planes,  called  the  co-ordinate  planes,'/ 
are  conceived  at  right  angles  to  one  another,  intersecting  in 
a  line  called  the  ground  line,  xy,  and  named  from  their  usual 
positions  the  horizontal  zxid.  vertical  planes  of  projection.    See 
Fig.  I. 

Drawings   or  projections  on   the   horizontal  plane    are 
called  plans  ;  on  the  vertical  plane  elevations. 

The  plan  or  horizontal  projection  of  any  point  A  in  space, 
is  the  foot  of  the  perpendicular  let  fall  from  point  A  to  the 
E.  G.         -|  t  1 


2  SOLID   OR  DESCRIPTIVE   GEOMETRY. 

horizontal  plane,  and  is  marked  a;  the  elevation  of  A  is 
marked  a\  and  is  the  point  of  intersection  of  the  perpendi- 
cular from  A  to  the  vertical  plane.     See  Fig.  i. 

Fig.  I. 


The  projection  of  a  line  may  be  defined  to  be  the  sum 
of  the  projections  of  its  points. 

Forms  possessing  three  dimensions,  length,  breadth,  and 
height,  require  to  be  represented  or  projected  upon  both 
planes  of  projection,  so  that  ^'height,"  as  well  as  "length" 
and  "breadth,"  may  be  exhibited  or  determined  by  the 
drawings. 

But  as  we  can  have  for  drawing  only  one  plane,  that  is, 
one  flat  sheet  of  paper,  not  two  at  right  angles  to  each  other, 
the  vertical  plane  is  supposed  to  rotate  backwards  upon  xy, 


DEFINITIONS  AND   THEOREMS.  3 

its  intersection  with  the  horizontal  plane,  as  upon  a  hinge, 
•  until  it  coincides  and  forms  one  plane  with  the  horizontal. 
The  plan  a  and  elevation  c^  of  any  point  A  will  then  lie  in 
the  same  perpendiadar  to  the  ground  line. 

This  perpendicular,  called  the  "  projecting  line  "  of  the 
point  A,  will  thus  form  a  locus  of  points  a  and  a',  that  is  to 
say,  must  contain  points  a  and  a'. 

The  distance  of  a  from  xy  shows  the  distance  of  the 
point  A  from  the  vertical  plane. 

The  distance  of  a'  from  xy  shows  the  distance  of  the 
point  A  from  the  horizontal  plane,  i.e.  its  height. 


The  object  of  all  science  is  exact  knowledge,  and  as 
this  is  impossible  of  attainment  unless  language  be  made  to 
keep  pace  in  precision  with  the  advance  of  thought,  it  has 
been  considered  desirable  to  rigidly  define  at  the  outset  all 
the  important  terms  that  have  a  meaning  peculiar  to  the 
subject.  It  is  not  intended  however  that  the  student  at  the 
very  commencement  of  his  labours  shall  learn  these  by  rote, 
but  he  is  earnestly  requested  to  turn  back  from  time  to 
time,  and  assure  himself  that  he  has  at  least  mastered  the 
significance  of  such  technical  terms  as  form  part  of  the 
description  of  the  problem  he  is  working.  Many  a  difficulty 
will  be  got  over  by  a  timely  reference  to  the  definitions, 
-while  full  mastery  of  the  language  of  the  subject  is  the  only 
way  to  a  thorough  grasp  of  its  principles. 

DEFINITIONS  AND  THEOREMS. 

The  object  of  Solid  Geometry  is  systematised  knowledge 
of  the  geometrical  properties  of  forms  having  three  dimen- 
sions. 


4  SOLID   OR  DESCRIPTIVE   GEOMETRY. 

Descriptive  Geometry  deals  directly,  with  the  relations 
of  these  forms  to  two  co-ordinate  planes  at  right  angles,  and 
indirectly,  with  their  relations  to  one  another.  The  object 
of  the  science  being  to  systematise  our  knowledge  of  the 
methods  by  which,  and  the  principles  upon  which,  these  forms 
of  three  dimensions  can  be  represented  upon  a  plane  or 
surface  of  two  dimensions,  so  that,  from  the  representation 
alone,  the  real  shape,  size,  and  relationship  of  the  forms 
may  be  unerringly  inferred. 

The  method  of  Descriptive  Geometry  is  almost  wholly 
a  method  of  projections,  as  laid  down  in  the  definitions  that 
follow. 

The  principles  are  involved  in  the  application  of  the 
method  to  the  solution  of  problems,  many  of  the  leading 
results  of  which  find  expression  in  the  subjoined  theorems. 

PRELIMINARY  DEFINITIONS. 

1.  The  projection  of  a  point  on  a  plane  is  the  foot  of 
the  perpendicular  let  fall  from  the  point  to  the  plane. 

2.  The  projection  of  a  line  (straight,  curved,  or  broken), 
figure,  or  form  is  the  sum  of  the  projections  of  its  points. 

3.  The  plan  of  a  point,  line,  figure,  or  form  is  its 
projection  on  the  horizontal  plane;  its  elevation  is  its  projec- 
tion on  the  vertical  plane. 

4.  The  perpendicular  which  gives  the  projection  of  a 
point  on  a  plane,  is  called  the  projector  of  the  point,  and  the 
plane  on  which  the  point  is  projected,  is  called  the  plafie  of 
projection  of  the  point.  The  vertical  projector  is  the  one 
drawn  to  the  vertical  plane ;  the  horizontal  projector  the  one 
drawn  to  the  horizontal  plane  of  projection. 


PRELIMINARY  DEFINITIONS.  5 

5.  The  surface  containing  all  the  projectors  of  a 
straight  or  curved  line,  is  called  the  projecting  surface  of  the 
line.  The  intersection  of  this  surface  with  the  plane  of  pro- 
jection coincides  with  the  projection  of  the  line  upon  that 
plane. 

6.  A  trace  is  the  intersection  of  a  straight  line  or  of  a 
surface  (plane  or  curved)  with,  unless  otherwise  stated,  one 
of  the  planes  of  projection.  The  horizontal  trace  (H.T.)  is 
the  intersection  with  the  horizontal  plane  (H.P.);  the 
vertical  trace  (V.T.)  is  the  intersection  with  the  vertical 
plane  (V.P.). 

Straight  lines  are  represented  either  by  their  traces  or 
by  the  method  of  projections.  By  the  latter  when y?;zzV<?,  and 
by  the  former  when  the  length  is  indefinite  or  disregarded 
for  the  purposes  of  the  problem.  Similarly,  figures  are 
represented  by  their  projections,  and  planes,  indefinitely 
extended,  by  their  traces.  Curved  surfaces  are  frequently 
represented  by  a  combination  of  both.  The  traces  of  an 
indefinitely  extended  straight  line,  or  of  a  plane,  are  always 
sufficient  to  determine  it.  The  traces  of  a  curved  surface 
are  not  alone  sufficient  to  determine  it. 

JVote.  The  projections  made  in  Descriptive  Geometry, 
are  called  ^^orthographic"  projections,  to  distinguish  them 
from  radial  ox  perspective  projections. 

7.  A  section  plane  is  a  plane  cutting  through  a  form. 
The  ^^ section"  is  the  trace  of  the  form  on  this  plane. 

Planes  may  be  vertical,  horizontal,  inclined,  or  oblique. 

8.  A  vertical  plane  is  one  at  right  angles  to  the  horizon- 
tal plane.  It  may  make  with  the  vertical  plane  of  projec- 
tion any  angle  between  0°  and  90°.     In  the  former  position 


6  SOLTD   OR  DESCRIPTIVE   GEOMETRY. 

it  will  be  called  a  parallel  vertical  plane,  and  in  the  latter,  a 
right  vertical  plane.  See  Theorems  II.  and  III.,  and  Fig.  5, 
planes  n^o'  and  hZv. 

9.  A  horizontal  plane  is  one  at  right  angles  to  the 
vertical  plane  and  parallel  to  the  horizontal  plane  of  projec- 
tion. 

10.  An  inclined  plane  is  one  at  right  angles  to  the 
vertical  plane  and  not  parallel  to  the  horizontal  plane.  Ex. 
qlm'.     Fig.  5. 

11.  An  oblique  plane  is  one  not  at  right  angles  to  either 
plane  of  projection,  i.e.  is  a  plane  inclined  to  both.  See 
qlm,,,  Fig.  2,  and  hav',  Fig.  5.  A  plane  incUned  to  both  co- 
ordinate planes  and  parallel  to  xy,  is  called  a  parallel  oblique 
plane.  See  plane  rs,  t'u,  Fig.  5,  An  oblique  plane  equally- 
inclined  to  both  planes  of  projection  is  called  a  right 
oblique. 

12.  A  plane  is  said  to  be  ^'constructed''  or  ^' rabafted,'' 
when  it  is  rotated,  with  all  its  contained  points,  lines,  and 
figures  upon  it,  about  one  of  its  traces  into  the  correspond- 
ing plane  of' projection. 

13.  A  development  is  the  figure  produced  by  unrolling 
or  laying  out  in  one  plane  the  surface  or  surfaces  of  a  form. 
All  surfaces  are  not  developable,  i.e.  cannot  be  laid  flat  in  one 
plane  without  tearing  or  doubling.  All  forms  bounded  by 
plane  surfaces,  and  some  bounded  by  curved  surfaces,  are 
developable.     See  Chap.  VII.,  Curved  Surfaces. 

THEOREMS. 

Theorem  I.  The  traces  of  a  straight  line  ^&  points.  A 
line  parallel  to  either  plane  of  projection  has  no  trace  in  the 
plane  to  which  it  is  parallel,  and  therefore,  a  line  parallel  to 


THEOREMS.  7 

xy  (the  line  in  which  the  co-ordinate  planes  of  projection 
intersect),  being  parallel  to  both  planes  of  projection,  will 
have  no  trace  at  all 

Theorem  II.  The  traces  of  a  plane  are  straight  lines, 
and,  unless  the  plane  is  parallel  to  xy,  the  traces  will  meet  at 
some  point  in  that  line.  A  plane  parallel  to  xy,  and  not 
perpendicular  to  either  plane  of  projection,  will  have  for  its 
traces  two  straight  lines  parallel  to  xy.  A  plane  parallel  to 
xy,  and  at  right  angles  to  one  of  the  planes  of  projection, 
will  be  consequently  parallel  to  the  other  plane  of  projec- 
tion and  will  have  no  trace  in  that  plane.     Figs.  2  and  5. 

Theorem  III.  If  the  horizontal  trace  of  a  plane  is  at 
right  angles  to  xy,  the  plane  is  perpendicular  to  the  vertical 
plane  of  projection.  If  the  vertical  trace  of  the  plane  is  at 
right  angles  to  xy,  the  plane  is  perpendicular  to  the  hori- 
zontal plane  of  projection.     Fig.  5  planes  qhn'  and  n^o. 

Theorem  IV.  The  two  projections  of  a  point  are  in  the 
same  projecting  line  or  perpendicular  to  xy,  after  the  vertical 
plane  has  been  turned  into  the  horizontal.     Fig.  3. 

Theorem  V.  The  perpendicular  from  the  plan  of  a 
point  to  xy  is  parallel  to  the  vertical  projector  of  the  point, 
and  is  equal  to  the  distance  of  the  point  from  the  vertical 
plane.  Similarly,  the  perpendicular  from  the  elevation  of  a 
point  to  xy  is  parallel  to  the  horizontal  projector  of  the 
point,  and  is  equal  to  the  distance  of  the  point  from  the 
horizontal  plane. 

Theorem  VI.  When  two  planes  are  at  right  angles, 
every  line  drawn  from  a  point  in  one  of  them  perpendicular 
to  the  other,  lies  wholly  in  the  former,  and  meets  the  latter 


8  SOLID   OR  DESCRIPTIVE   GEOMETRY. 

in,  and  is  perpendicular  to,  the  line  of  intersection  of  the 
planes.     Whence  it  follows  :— 

(i)  A  point  in  one  plane  of  projection  has  its  pro- 
jection on  the  other  in  xy.  That  is,  if  the  point  is  in  the 
vertical  plane  its  plan  is  in  xy.  If  in  the  horizontal  plane 
its  elevation  is  in  that  line. 

(2)  If  a  given  plane  be  perpendicular  to  one  plane  of 
projection,  it  contains  all  the  projectors  drawn  from  any 
points,  hnes,  or  figures,  in  that  plane,  to  the  plane  of  pro- 
jection to  which  the  given  plane  is  perpendicular.  That  is 
to  say,  the  given  plane  is  the  projecting  plane  of  all  figures 
lying  in  it.  The  trace  of  this  plane  on  the  plane  of  pro- 
jection to  which  it  is  at  right  angles,  contains  the  projections 
on  this  plane  of  all  points,  lines,  or  figures  lying  in  the  given 
plane. 

(a)  Thus,  if  an  inclined  plane  be  perpendicular  to  the 
vertical  plane  of  projection,  the  elevations  of  all  points  in 
the  inclined  plane  will  be  in  its  vertical  trace.  E.g.  A 
circle  of  3  inches  diameter  lying  in  a  plane  inclined  at  30" 
to  the  horizontal  plane  and  perpendicular  to  the  vertical 
plane,  has  a  segment  of  the  vertical  trace,  3  inches  long,  for 
its  elevation. 

(/3)  A  plane  inclined  to  the  vertical  plane  of  pro- 
jection and  perpendicular  to  the  horizontal  plane,  has  the 
plans  of  all  points,  lines,  and  figures  lying  in  it  projected  in 
its  horizontal  trace.  Thus,  the  plans  of  the  vertical  faces  of 
a  right  prism  resting  on  its  base  coincide  with  the  lines 
forming  the  sides  of  that  base. 

Theorem  VII.  The  projection  of  a  straight  line  on  a 
plane  is  a  straight  line.  The  projections  upon  the  same 
plane  of  parallel  straight  lines  are  parallels. 


THEOREMS.  9 

The  intersection  of  a  plane  with  a  series  of  parallel 
planes  will  be  a  series  of  parallel  lines,  and  the  projections 
of  these  intersections  will,  according  to  this  theorem;  be 
parallels. 

Theorem  VIII.  The  orthographic  projection  of  a  finite 
straight  line  is  equal  to  the  real  line  if  the  latter  is  parallel 
to  the  plane  of  projection.  The  magnitude  of  the  pro- 
jection diminishes  as  the  angle  of  inclination  of  the  line 
increases,  and  becomes  a  minimum,  i.e.  a  point,  when  the 
angle  of  inclination  reaches  90",  or  when  the  line  is  perpen- 
dicular to  the  plane. 

The  projection  of  a  rectilineal  angle  on  a  plane  parallel 
to  that  of  the  angle,  is  an  angle  equal  to  the  given  one. 

The  projection,  on  the  same  plane,  of  the  circular  arc 
subtending  that  angle  will  also  be  equal  to  that  of  the  arc. 
See  Figs.  21  and  22. 

Generally,  the  projection  of  any  plane  figure  on  a  plane 
parallel  to  it  is  a  figure  equal  in  all  respects  to  the  figure  of 
which  it  is  the  projection.  And  the  area  of  the  projection 
of  a  given  figure  decreases  as  the  angle  between  the  plane 
of  projection  and  the  plane  of  the  figure  increases, — 
becoming  a  minimum,  i.e.  a  straight  line,  when  this  angle  is 
a  right  angle. 

Note.  This  is  a  principle  often  taken  advantage  of  in 
finding  the  true  shape  of  sections.     See  Prob.  i,  Chap.  I. 

Theorem  IX.  The  projections  of  a  perpendicular  to  a 
plane  are  perpendiculars  to  the  traces  of  the  plane.  The 
plan  is  at  right  angles  to  the  horizo?ital  trace ;  the  elevation 
is  at  right  angles  to  the  vertical  trace. 


lO  SOLID  OR  DESCRIPTIVE   GEOMETRY. 

Theorem  X.  If  a  straight  line  lies  in  a  plane  and  has 
traces  (Theorem  L),  they  will  be  in  those  of  the  plane. 
Thus  a  plane  can  be  drawn  to  contain  a  given  line  by- 
taking  the  traces  of  the  plane  through  those  of  the  line, 
when  the  latter  are  determinable. 

Theorem  XL  The  angle  which  the  tangent  plane  to  a 
right  cone  makes  with  the  plane  of  the  base  of  the  cone,  is 
measured  by  the  plane  angle  which  a  generatrix  of  the  cone 
makes  with  the  base,  or  is  the  complement  of  the  angle 
between  a  genera.trix  and  the  axis. 

Theorem  XII.  If  a  plane  touches  a  conic  surface  and 
a  second  plane  intersects  them,  the  trace  of  the  tangent 
plane  on  the  intersecting  one  is  a  tangent  to  the  trace  of  the 
conic  surface  on  the  same  plane.  Generally,  any  plane 
cutting  a  curved  surface  and  its  tangent  plane  through  a 
point  of  contact  of  the  tangent  plane  with  the  surface,  will 
intersect  the  tangent  plane  in  a  Hne  which  is  a  tangent  to 
the  curve  in  which  the  cutting  plane  intersects  the  surface. 

Theorem  XIII.  The  tangent  plane  at  any  point  of  a 
curved  surface  contains  the  tangent  lines  drawn  at  the 
same  point  to  all  the  lines  traced  on  the  surface  at  that 
point. 

Theorem  XIV.  The  tangent  plane  to  a  cone  or  a 
cylinder  at  a  given  point  is  the  tangent  plane  to  the  surface 
at  every  other  point  in  the  generatrix  passing  through  the 
given  point,  and  all  the  normals  to  the  surface  along  the 
generatrix  are  in  a  plane  containing  the  generatrix  and  per- 
pendicular to  the  tangent  plane. 

Theorem  XV.  Parallel  sections  of  a  conic  surface  are 
similar  but  unequal  curves. 


THEOREMS.  II 

Theorem  XVI.  Points  in  the  generatrix  of  a  surface  of 
revolution  describe  circles  in  planes  perpendicular  to  the 
axis  of  revolution  whose  centres  are  the  points  of  inter- 
section of  the  axis  with  their  respective  planes. 

Theorem  XVII.  All  sections  of  a  surface  of  revolution 
made  by  planes  containing  the  axis  are  similar  and  equal 
curves. 

Theorem  XVIII.  A  plane  which  bisects  a  chord  of  a 
sphere  at  right  angles  passes  through  the  centre  of  the 
sphere. 

Theorem  XIX.  The  section  of  a  sphere  by  a  plane 
passing  through  its  centre,  is  a  circle  whose  radius  is  equal 
to  the  radius  of  the  sphere — hence  called  a  great  circle. 
Any  other  plane  section  cuts  a  small  circle. 

Theorem  XX.  When  a  plane  is  '■^constructed"  or 
^'  rabatted"  (Def.  12)  about  one  of  its  traces  every  point  in 
it  will  have  for  its  locus  a  straight  line  drawn  at  right  angles 
to  the  trace  used  through  the  projection  of  that  point  on 
the  plane  of  projection  containing  the  trace. 

Thus  if  the  horizontal  trace  be  used  as  the  axis  of 
"rabatment"  or  "construction,"  the  loci  of  all  points  in  the 
plane  will  be  the  perpendiculars  to  the  trace  through  their 
respective  plans. 

The  reason  for  this  relation  or  "locus"  will  be  obvious  by 
considering  that  every  point  in  a  "constructed"  plane  must 
describe  a  circle  about  the  trace  forming  the  axis,  and  that 
the  projecting  plane  of  the  path  of  the  point  is  a  plane 
perpendicular  to  that  trace. 

These  loci  are  made  use  of,  for  example,  in  determining 
the  true  shape  of  a  section,  and  the  projection  of  a  frustum 


12  SOLID   OR  DESCRIPTIVE   GEOMETRY. 

of  a  solid  on  a  plane  which  passes  through  it.  The  solution 
of  many  problems  whose  data  lie  in  one  plane  is  often 
greatly  facilitated  by  this  method  of  rabatments. 


DEFINITIONS   ILLUSTRATED. 

The  co-ordinate  planes,  upon  which  the  projections  are 
made,  intersect  at  right  angles  in  a  line  called  the  ground 
line  {xy),  and  are  named  from  their  usual  positions  the 
horizontal  and  vertical  planes  of  projection.     Fig.  i. 

The  four  dihedral  angles  formed  by  the  intersecting 
planes  of  projection  are  known  as  the  ist,  2nd,  3rd, 
and  4th. 

The  ist  dihedral  angle  is  that  contained  between  the 
upper  face  of  the  H.P.  and  the  front  face  of  V.P. 

The  2nd  is  that  between  the  upper  face  of  the  H.P.  and 
the  back  face  of  V.P. 

The  3rd  is  between  the  lower  face  of  H.P.  and  the  back 
.face  of  V.P. 

The  4th  between  the  lower  face  of  H.P.  and  the  front 
face  of  V.P. 

Notation.       .  -  -  . 

A  point  in  space  is  indicated  by  a  capital  letter  A,  its 
projections  by  itaHcs  a  and  a':  the  accented  letter  denoting 
the  vertical  projection  or  elevation  of  the  point,  and  the 
unaccented  italic  the  plan.     Figs,  i  and  3. 

Similarly,  a  line  in  space  is  denoted  by  capital  letters,  as  ; 
AB:  and  its  projections  by  italics,  as;  ab,  a'b'.   Figs,  i  and  4. 


DEFINITIONS  ILLUSTRATED. 


13 


A  plane  is  denoted  by  three  letters,  one  on  each  trace 
and  the  third  on  the  point  of  intersection  in  xy.  Figs.  2 
and  5. 

Fig.  I. 


Note.  Figs.  I  and  2  are  merely  pseudoperspective 
sketches  illustrative  of  the  elementary  principles  and  defini- 
tions, and  must  on  no  account  be  mistaken  for  the  kind 
of  drawing  to  be  made  in  the  solution  of  the  problems. 
Figs.  3,  4,  and  5  are  orthographic  illustrations  of  the  mode 
of  determining  points,  lines,  and  planes,  and  may  be  taken 
as  elementary  types  of  the  kind  of  drawing  to  be  made. 


14  SOLID   OR  DESCRIPTIVE    GEOMETRY. 

Fig.  I.  The  points  aa'  are  the  projections  of  the  point 
A.     Def.  X. 

The  lines  ah,  a'b'  are  the  projections  of  the  line  AB. 
Def.  2.  The  line  ^^  is  the  plan,  and  «y  the  elevation. 
Def.  3. 

The  perpendiculars  Aa,  Aa',  and  j5/^,  Bb',  are  the 
projectors  of  the  points  ^  and  ^;  Aa,  Bb,  and  ^«',  ^i!'', 
being  the  horizotital  and  vertical  projectors  of  the  points  ^ 
and  B  respectively,  and  the  planes  H.  and  V.  the  horizontal 
and  vertical  pla?ies  of  projection.     Def.  4. 

The  planes  AB,  ab,  AB,  a'b',  containing  the  given 
straight  line  AB  and  the  projectors  Aa,  Aa',.  and  Bb,  Bb', 
are  the  projecting  planes  of  the  line  AB,  and  the  intersection 
of  these  planes  with  the  planes  of  projection  give  the  lines 
ab,  ab'  which  are  the  projections  of  the  lines  AB.     Def.  5. 

As  our  drawings  can  only  be  made  conveniently  on  one 
plane,  we  assume  that  the  vertical  plane,  with  the  vertical 
projection  on  it,  is  rotated  backwards  in  the  direction  shown 
,  by  the  arrows,  about  xy  as  an  axis,  until  it  coincides  with  the 
horizontal  plane.  In  this  position  the  vertical  is  oftentimes 
more  or  less  superposed  upon  the  horizontal  projection,  and 
as  in  all  our- reasonings  upon  this  subject  it  is  necessary  to 
be  clear  as  to  what  points,  lines,  &c.  are  in  the  vertical,  and 
what  in  the  horizontal  planes,  some  conventional  system  of 
notation,  such  as  that  described  above,  has  to  be  adopted. 
When  the  planes  are  thus  rotated  the  plan  a  and  the  elevation 
(i!  of  any  point  A  will  lie  in  the  same  perpendicular  to  the 
ground  line,  xy.     Theorem  IV. 

This  perpendicular  is  therefore  a  locus  of  points  a  and  a', 
that  is,  will  contain  a  and  a'. 


DEFINITIONS  ILLUSTRATED. 


15 


The  distance  of  a  from  xy  shows  the  distance  of  the 
point  A  from  the  vertical  plane;  and  the  distance  of  a'  from 
xy  shows  the  distance  of  the  point  A  from  the  horizontal 
plane,  i.e.  its  height.  Thus  aa  is  parallel  and  equal  to  Aa\ 
and  a'o.  is  parallel  and  equal  to  Aa.    Theorem  V. 

Fig.  2. 


hor.  trace 


Fig.  2  shows  the  traces  (Def.  6)  of  an  "oblique"  plane 
(Def  1 1)  meeting  in  xy  (Theorem  II.).  The  traces  of  every 
plane  which  is  not  a  parallel  vertical,  a  horizontal  or  a 
parallel  oblique  (Defs.  8,  9,  and  11),  will  if  produced  meet 
in  xy. 


i6 


SOLID   OR  DESCRIPTIVE   GEOMETRY. 


ORTHOGRAPHIC  ILLUSTRATIONS. 

Fig-  3- 
Points. 


*u 


/ 


/ 


V 


Examples  of  points  determined  in  various  positions  by 
their  projections. 

Point  A  or  aa!  is  in  the  ist  dihedral  angle  of  the  planes  of 

projection. 
„     B  or  bb'         ,,         2nd         „         „         „         „         „ 
,,     C  or «  ,,         3^^         11         11         11         "         " 

„     D  or  dd!        ,,         4th         „         „         „         „         „ 
„     E  or  e^  is  in  the  horizontal  and  behind  the  vertical 

plane. 
„     Foxff  is  in  both  planes. 
,,     G  ox  gg  is  in  the  vertical  and  below  the  horizontal 

plane. 
„     H  or  hh'  is  equidistant  from  the  horizontal  and  ver- 
tical planes. 
„    K  or  kk'  is  in  the  vertical  and  above  the  horizontal 
plane. 


ORTHOGRAPHIC  ILLUSTRATIONS. 


17 


Observe,  That  to  find  the  projections  of  points  satis- 
fying given  conditions  of  position  with  regard  to  the  planes 
of  projection  the  relations  indicated  in  Theorems  IV.  and  V. 
will  suffice. 


Examples  of  lines  in  various  positions  determined  by 
;heir  projections, 


E.  G. 


1 8  SOLID   OR  DESCRIPTIVE   GEOMETRY. 

The  segment  AB  or  ab,  a'b  is  in  the   ist  dihedral  angle  of 

planes. 
„  CD  or  cd,  c'd!        „         and         ,,         „ 

„  EFoxef,e'f'  „         3rd         „ 

„  GHoxgh.^H       „         4th        ,, 

„  AfJV  or  Pin,  nin'   „  ist         „         „ 

and  the  line  is  perpendicular  to  the  vertical  plane. 
„  OP  or  op,  dp'  is  in  the  ist  dihedral  angle. 


Fig.  5- 


£*la. 


ORTHOGRAPHIC  ILLUSTRATIONS.  19 

Examples  of  planes  defined  or  expressed  by  their  traces. 

The  portion  included  between  the  infinite  branches  ah 
and  av'  of  the  given  plane  is  in  the  first  dihedral  angle  of 
the  planes  of  projection. 

The  portion  between  the  branches  a?/  and  af  is  in  the 
second  dihedral  angle. 

The  portion  between  af  and  ag'  is  in  the  third  angle, 
and  the  portion  between  branches  a^  and  ah  is  in  the 
fourth  angle. 

The  plane  glm'  is  perpendicular  to  the  vertical  plane 
md  makes  an  angle  of  6"  with  the  horizontal. 

The  plane  ;z/?^'  is  at  right  angles  to  the  horizontal  and 
nakes  an  angle  of  i^"  with  the  vertical  plane. 

The  plane  rs,  t'u'  is  parallel  to  xy  and  meets  both 
)lanes. 

The  plane  hlv'  is  at  right  angles  to  xy  and  therefore  to 
both  planes  of  projection. 


MEMORANDA 

FOR   WORKING   THE   PROBLEMS   IN    ENSUING   CHAPTERS. 

All  the  solids  given  are  assumed  to  be  ^ right,'  unless 
Otherwise  expressed. 

All  dimensions  and  arrangements,  not  expressly  men- 
tioned and  limited,  may  be  assumed  at  pleasure. 

The  inclinations  of  all  lines  and  planes  must  be  under- 
stood, unless  otherwise  mentioned,  to  be  to  the  horizontal 
plane. 

Invisible  edges  are  those  which  are  hidden  by  the  solid 
from  the  eye  when  looking  in  the  direction  in  which  the 
projectors  are  drawn. 

Sections.  The  teacher  should  give  directions  for  sections 
and  developments,  as  far  as  possible,  with  each  problem. 

Also  several  plans  and  elevations  may  advantageously  be 
completed  of  the  solids  given  in  many  of  the  problems. 


I. 


SOLIDS  IN   SIMPLE  POSITIONS. 

I.  Draw  plan  and  elevation  of  a  pyratnid,  3-5  inches 
highy  with  square  base  0/2'^  inches  side,  when  resting  ivith 
its  base  on  the  horizontal  plane,  and  with  one  side  of  the  base 
?naki?ig  an  a?tgle  of  30"  with  the  vertical  plane. 

Commence  with  the  plan.  This  will  be  the  square 
ABCD  of  the  base  with  the  opposite  corners  joined  for  the 
plans  of  the  slant  edges  of  the  solid.  The  point  in  which 
the  diagonals  of  the  square  cross  will  be  the  plan  v  of  the 
vertex  V. 

For  the  elevation  take  the  ground  line  xy  inclined  30" 
with  one  side  of  the  square  and  draw  perpendiculars  to  xy 
(Theorem  IV.)  from  the  four  corners  abed  and  the  centre  v  of 
the  square.  The  elevations  cib'c'd'  of  the  four  corners  of 
the  square  will  be  in  the  ground  line,  because  the  base  of 
the  solid  rests  on  the  horizontal  plane.  The  height  of  the 
pyramid,  3*5  inches,  must  be  set  up  for  its  vertex  v'  from 
the  point  where  the  perpendicular  from  the  centre  of  the 
square  meets  the  ground  line,  and  at  right  angles  to  that 
line  (Theorem  v.).  The  whole  elevation  will  be  completed 
by  joining  the  elevation  v'  of  the  vertex  to  the  four  points 
determined  cib'c'd'  on  the  ground  line  for  the  elevations  of 
the  four  corners  of  the  base. 

Invisible  edges.     In  this  plan  all  the  edges  of  the  solid 


22  SOLID   OR  DESCRIPTIVE   GEOMETRY. 

are  visible.  In  this  elevatmi  the  slant  edge  of  the  pyramid 
nearest  xy  is  invisible.  The  two  sides  of  the  base  nearest 
xy  are  also  invisible,  but  as  they  coincide  with  the  two  other 
sides  in  the  xy,  they  cannot  be  shown  in  dotted  lines. 

Note.  It  must  be  understood  that  from  any  one  plan 
and  elevation  any  number  of  elevations  can  be  drawn  by 
assuming  vertical  planes,  that  is,  by  taking  new  ground  lines, 
in  different  positions  round  the  solid,  and  working  from  the 
plan  to  them.  For  example,  if  a  solid  were  placed  on  the 
floor  of  a  room,  four  elevations  of  it  might  be  shown  on  the 
four  walls  of  the  room,  which  are  so  many  vertical  planes  of 
elevation.  And  so  from  any  one  elevation  aftd  plan  any 
nnmber  of  plans  may  be  determined  by  assuming  horizontal 
planes  in  the  desired  position  about  the  elevation,  and 
working  from  the  elevation  to  them. 

E.g.  For  a  new  elevation.  The  new  ground  line,  xy, 
being  taken,  draw  perpendiculars  to  it  from  the  plans  of  the 
various  points  of  the  solid,  and  on  these  perpendiculars  from 
xy  mark  the  various  heights  of  the  points,  to  be  taken  from 
the  elevation  already  drawn,  and  complete  by  joining  the 
points  thus  found  as  already  joined  in  plan. 

For  a  new  plan.  The  new  xy  being  taken,  draw  perpen- 
diculars to  it  from  the  elevations  of  the  various  points  of  the 
solid,  and  on  these  perpendiculars  mark  from  xy  the  various 
distances  which  the  points  are  from  the  first  plane  of  eleva- 
tion to  the  pla7i  first  drawn,  and  complete  by  joining  the 
points  as  already  joined  in  elevation. 

To  work  a  section  of  the  solid.  Draw  a  line  through 
the  middle  point  of  the  elevation  of  the  axis  making  an 
angle  of  40"  with  xy.     Assume  this  to  be  the  vertical  trace 


SOLIDS  IN  SIMPLE  POSITIONS.  23 

(Def.  6  and  Theorem  II.)  of  an  inclined  section  plane  (Defs. 
7  and  10).  Produce  the  vertical  trace  to  meet  xy  and  from 
the  point  of  intersection  of  these  hnes  draw  the  horizojital 
trace  perpendicular  to  xy.     (Theorem  III.). 

The  points  i',  2',  3',  4',  in  which  the  given  line  or 
vertical  trace  cuts  respectively  the  lines  a'v' ,  b'v',  c'v',  and 
d'v'  are  the  elevations  of  the  points  I.,  II.,  III.,  IV.  in  which 
the  section  plane  cuts  the  edges  AV,  BV,  CV  and  DF  of 
the  pyramid.  (Theorem  VI.  (2)  and  (a)).  The  plans  of 
these  points  will  be  found  by  drawing  lines  from  their 
elevations  at  right  angles  to  xy  to  cut  the  corresponding 
lines  in  plan.  (Theorem  IV.).  £.g.  A  line  at  right  angles  to 
xy,  drawn  from  the  point  i'  in  a'v'  to  cut  the  plan  av,  will 
give  the  point  i  which  is  the  plan  of  the  real  point  I.,  in  the 
section,  of  which  i'  is  the  elevation.  Similarly,  the  points 
2,  3,  4  in  the  plan  may  be  found.  The  figure  got  by 
joining  i  to  2,  2  to  3,  3  to  4,  4  to  i  is  the  plan  of  the 
section.  This  for  clearness  should  be  conventionally 
crossed  with  a  series  of  parallel  lines.  The  elevation  of  the 
section  is  shown  by  the  points  i',  2',  3',  4'  in  the  vertical 
trace.     (Theorem  VI.  (2)  and  (a)). 

To  find  the  true  shape  of  the  section. 

This  is  done  by  the  inethod  of  rabatments  or  '■^con- 
struction'^ (Def  12  and  Theorem  XX.).  That  is,  the 
plane  of  section  is  rotated  about  one  of  its  traces  as  an  axis 
into  the  plane  of  projection  containing  the  trace  which  is 
chosen  for  the  operation.     See  Theorem  XX. 

a.  To  find  the  true  shape  of  the  section  by  rabatting  the 
plane  about  its  vertical  trace. 

The  vertical  projectors  (Def.  4)  of  the  points  I.,  II.,  III., 


24  SOLID   OR  DESCRIPTIVE   GEOMETRY. 

IV.  of  the  section,  i.e.  the  perpendiculars  from  these  points 
to  their  elevations  i',  2',  3',  4',  lie  in  the  plane  of  section 
(Theorem  'VI.  (2)),  are  at  right  angles  to  the  vertical 
trace  (Theorem  VI.),  and  are  equal  in  length  to  the  per- 
pendiculars drawn  from  the  plans  of  the  points  i,  2,  3,  4 
to  xy,  (Theorem  V.) — i.e.  \he  plane  of  section  is  \h&  projecting 
plane  of  the  section.  If,  therefore,  lines  be  drawn  from  the 
points  i',  2',  3',  4',  of  the  elevation,  at  right  angles  to  the 
vertical  trace  of  the  section  plane,  equal  in  length  to  the 
respective  distances  of  the  points  i,  2,  3,  4  of  the  plan  from 
xy,  and  the  points  I.,  II.,  III.,  IV.,  so  found,  at  the  extremi- 
ties of  the  perpendiculars,  be  joined  in  order,  the  figure 
produced  will  be  that  resulting  from  the  rabatment  of  the 
plane  of  section  about  its  vertical  trace,  i.e.  will  be  the  true 
shape  of  the  section. 

p.  To  find  the  true  shape  by  rabatfing  the  plane  about 
its  horizontal  trace.  If  the  plane  be  rotated  about  the  H.T. 
as  an  axis  into  the  horizontal  plane,  the  points  I.,  II.,  III., 
IV.  of  the  section  will  describe  segments  of  circles  about 
the  H.T.  in  planes  perpendicular  to  it,  and  the  loci  of  these 
points  in  plan  will  be  straight  lines  drawn  from  the  plans  of 
the  points  i,  2,  3,  4  of  the  section  at  right  angles  to  the 
H.T.  of  the  section  plane.  If  on  these  lines  are  measured 
off  from  H.T.  the  distances  of  the  corresponding  points 
i',  2',  3',  4'  up  the  vertical  trace  from  xy  (i.e.  the  radii  of  the 
segments  of  the  circles  described  by  the  points  of  the 
section  when  the  plane  is  being  rabatted),  the  figure  pro- 
duced by  joining  these  points  will  be  the  true  shape  of  the 
section. 

Note.  To  avoid  the  confusion  that  sometimes  results 
from  rabatting  the  section  plane  directly,  the  section  may 


SOLIDS  IN  SIMPLE  POSITIONS.  25 

be  projected  on  a  plane  taken  parallel  to  the  plane  of 
section,  and  will  then  be  shown  in  true  shape.  (Theorem 
VIIL).  This,  which  is  merely  the  method  of  rabatments 
under  a  disguise,  is  of  little  use  unless  the  section  plane  be 
perpendicular  to  one  of  the  planes  of  projection.  The  xy 
for  the  parallel  plane  must  obviously  be  taken  parallel  to 
the  trace  of  the  section  plane  on  that  plane  of  projection 
to  which  the  section  plane  is  perpendicular.  Finish  by 
drawing  the  projection  of  the  frustum  of  the  pyramid  on 
this  plane. 

To  find  the  true  length  and  inclination  of  a  slant  edge  of 
the  pyrainid. 

a.  Make  a  projection  on  a  plane  parallel  to  the  edge 
whose  length  is  required,  i.e.  take  xy  parallel  to  the  plan  of 
the  slant  edge.  The  elevation  will  show  the  inclination 
and  the  edge  in  true  length.    Theorem  VIIL  or. 

By  a  special  construction, 

13.     Make  a  right-angled  triangle  having  its  two  sides 

respectively  equal  to  the  plan  of  the  edge  and  the  length  of 

the  axis — the  hypotenuse  will  give  the  true  length  of  the 

edge;  and  the  angle  subtended  by   the   axis  will  be  the 

angle  which  the  edge  makes  with  the  horizontal  plane. 

• 
To  work  the  development  of  the  frusitcm.     See  Def.  13. 

Describe  a  circle  with  radius  equal  to  the  slant  edge  of 
the  pyramid  and  cut  off  four  consecutive  chords  of  this 
circle  equal  to  AB,  BC,  CD,  DA,  the  sides  of  the  base  of 
the  pyramid.  Draw  AB,  BC,  CD,  DA,  and  join  the  points 
A,  B,  C,  D,  with  the  centre  V  of  the  circle.  Describe  a 
square  on  one  of  the  bases  of  the  four  triangles  thus 
:)roduced.     The    four    triangles   and   the   square   together 


26  SOLID   OR  DESCRIPTIVE   GEOMETRY. 

forhi  the  development  of  the  pyramid.  Find  now  the  points 
I.,  II.,  III.,  IV.  in  the  edges  VA,  VB,  VQ  VZ>,  by  dividing 
each  of  these  edges  in  the  same  proportion  as  the  corre- 
sponding elevations  are  divided  by  the  vertical  trace  of  the 
section  plane.  Join  the  points  in  order  (noting  that  in  the 
development  one  edge  and  its  point  of  division  necessarily 
occurs  twice)  and  cut  the  part  next  the  vertex  away.  This 
is  the  development  of  the  frustum.  Build  up  into  a  model 
and  fit  on  the  true  shape  of  the  section. 

2.  P/an  and  elevation  of  a  pyramid  with  base  a  regular 
hexagon  0/1')$  in.  side,  when  resting  on  its  base,  and  with 
one  side  of  base  inclined  40°  to  the  vertical  plane.  Axis  3*5 
inches. 

Work  also  a  section  by  a  vertical  plane  [Fig.  5  n^o'^^,  the 
angle  <^"  being  35",  and  the  horizontal  trace  '25  in.  from  the 
plan  of  the  axis.  Show  new  plan  and  new  elevation  of  the 
frustum. 

Find  true  shape  of  section  by  rabatting  or  "constructing" 
the  plane  of  section  about  one  of  its  traces,  and  develop  the 
frustum. 

3.  Plan  and  elevation  of  a  square  prism  3*5  in.  long, 
bases  2*5  in.  side. 

(a)  Wheti  standing  on  one  of  its  bases  and  a  rectangular 
face  making  an  angle  of  ^0°  with  the  vertical  plane  of 
projection. 

()8)  When  resting  on  a  rectangular  face  and  its  bases 
making  a?igles  of  ^o^  with  the  vertical  plane. 

To  work  the  prism  in  (a)  with  section,  true  shape,  pro- 
jections of  the  frustu?n,  aJid  development. 


SOLIDS  IN  SIMPLE  POSITIONS.  2/ 

The  plan  will  be  a  square  with  one  side  inclined  at  30" 
to  xy. 

Begin  by  drawing  xy.  Set  off  from  xy  a  line  at  30°  in 
the  H.P.  and  describe  on  any  segment  ab,  2-5  in.  long,  of 
this  line  a  square  abed.  This  will  be  the  plan  of  the  prism, 
i.e.  of  its  two  bases — one  on  H.P.  and  the  other  3*5  in. 
above — and  its  four  vertical  faces.  The  elevations  db'c'd' 
of  the  four  corners  A  BCD  of  the  lower  base  will  be  in  xy, 
and  the  elevations  a^^^^c^d^  of  the  four  corners  A^B^CJ)^ 
of  the  upper  base  will  be  in  a  line  parallel  to  xy  and  3*5  in. 
above  it.  The  segments  da^,  b'b^,  c'c^,  d'd^  of  the  lines 
drawn  from  the  four  corners  of  the  plan  abed,  at  right  angles 
to  xy  for  the  elevation,  will  be  the  elevations  of  the  edges 
AA^,  BB^,  CQ,  DD^,  respectively.  Make  the  edge  AA^, 
which  is  nearest  the  V.P.,  dotted  in  the  elevation;  and  the 
plan  and  elevation  will  be  complete. 

Section.  Take  a  line  cutting  the  elevations  of  the 
upper  base  a^b^e^d^,  and  three  of  the  edges  a'a^,  b'b^,  e'c^, 
in  the  points  i',  2',  3',  respectively,  for  the  vertical  trace 
of  the  section  plane.  The  elevation  of  the  section  will  be 
in  this  line.  For  the  plan,  draw  a  line  from  the  point  in 
,vhich  the  vertical  trace  cuts  the  elevation  of  the  upper  base, 
it  right  angles  to  xy,  and  produce  it  to  cut  the  sides  ad  and 
ie  of  the  plan  abed  in  the  points  j"  and  z.  The  figure  sabez 
vill  be  the  plan  of  the  section.  Find  the  true  shape  by 
"abattiftg  the  section  plane  about  one  of  its  traces  as  in 
?rob.  I. 

New  elevation  of  frustum.     Take   a   new   xy  at   right 

ingles  to  the  xy  first  drawn,  and  project  as  for  a  complete 

'  ilevation  of  the  prism.    Show  the  elevations  of  S  and  Zin  the 

lew  elevation  of  the  upper  base,  and  find  the  points  i',  2',  3', 


2S  SOLID   OR  DESCRIPTIVE   GEOMETRY. 

by  measuring  the  heights  of  these  points  from  xy  in  the 
elevation  first  drawn  and  setting  them  off  on  the  cor- 
responding edges  in  the  new  elevation.  Join  up  the  points 
of  the  section  for  the  complete  elevation  of  the  frustum. 

A  new  plan  of  the  frustum  can  be  made  in  a  similar 
manner  by  taking  a  new  xy  at,  say  30",  with  the  elevations  of 
the  long  edges  of  the  prism — using  either  of  the  above 
elevations  of  the  prism. 

Development  of  frustum.  Draw  two  parallel  lines  3*5 
in.  apart  and  measure  off  on  the  lower  one  the  segments 
AB,  BC,  CD,  DA,  equal  to  the  sides  of  the  square  base  of 
the  prism.  From  the  points  A,  B,  C,  D,  A,  draw  lines  at 
right  angles  to  cut  the  second  parallel  line  in  the  points 
A^.,  Bi,  Ci,  D.2,  A2.  On  DA  and  D-^A^  describe  the 
squares  DABC  and  D^A^B^Q.  This  completes  the  de- 
velopment of  the  prism. 

For  the  frustum  find  the  points  I.,  II.,  III.  in  the  edges 
AA2,  BB.^,  CC2,  and  the  points  S  and  Zin  AiD^  and  D^C^. 
The  lengths  AL,  BIl.,  CIIL,  can  be  got  from  the  eleva- 
tions and  the  lengths  D^S  and  D^Z  from  the  plan.  Fit  on 
the  true  shape  of  the  section  and  build  up  the  model. 

The  prism  in  (/8)  the  student  should  now  be  able  to 
work  for  himself. 

4.  Blan  and  elevation  of  a  right  cylinder;  axis  3*5  ///. 
horizofital  and  inclined  40°  to  the  vertical  plane.  Diameter  of 
base  2*5  in. 

Conceive  the  cylinder  to  be  inscribed  in  the  prism  (/S) 
Prob.  3.  The  plan  of  the  prism,  which  is  a  rectangle  3-5  in. 
long  and  2-5  wide,  is  also  the  plan  of  the  cylinder.  The 
elevation  of  the  cylinder  is  made  by  inscribing  ellipses  in  the 


SOLIDS  IN  SIMPLE  POSITIONS.  29 

rectangles  which  are  the  elevations  of  the  square  ends  of  the 
prism. 

To  work  a  section  of  the  cylinder  take  an  auxiliary  xy  at 
right  angles  to  the  plan  of  the  axis  and  draw  the  circle 
which  is,  in  this  position,  the  elevation  of  the  cylinder. 
Divide  this  circle  into,  say,  sixteen  equal  parts  and  draw,  from 
each  of  the  points,  lines  in  plan  parallel  to  the  axis.  These 
will  be  the  plans  of  lines  lying  on  the  surface  of  the  cylinder. 
Show  these  lines  on  the  first  elevation  of  the  cylinder  by 
measuring  their  heights  in  the  auxiliary  elevation  and 
transferring  them.  A  vertical  or  inclined  section  plane  may 
now  be  taken  and  the  section  determined  by  finding  the 
projections  of  the  points  in  which  the  given  plane  cuts  these 
lines.  The  points  found  must  be  joined  by  a  curved  line 
put  in  by  hand.  The  section  of  a  right  cylinder  by  a  plane 
which  is  neither  parallel  nor  at  right  angles  to  its  axis,  as  in 
this  case,  is  an  ellipse.  Find  the  true  shape  by  rabatting  the 
plane. 

To  work  the  development.  Measure  a  straight  line  equal 
to  the  circumference  (found  by  multiplying  the  diameter  of 
the  cyHnder  by  3-1416)  and  divide  this  line  into  sixteen 
equal  parts.  Draw  lines  i,  2,  3,  4,  &c.  from  these  points  and 
make  each  equal  to  the  length  from  the  base  to  the  section 
plane  of  the  corresponding  line  on  the  cylinder. 

Join  the  points  so  found  by  a  curve  for  the  development 
of  the  cylindrical  surface. 

5.  Right  cotie  with  axis  3*5  in.  and  diameter  of  base 
2*5  in. 

a.  Plan  and  elevation  when  standing  on  its  base,  and  a 
ucond  plan  when  resting  o?i  its  conical  surface. 


30  SOLID   OR  DESCRIPTIVE  GEOMETRY. 

p.  Plan  and  elevation  when  the  axis  is  horizontal,  2  in. 
above  the  H.P.,  and  itidined  35"  /^  the  V.P. 

a.  The  plan  of  the  cone  when  standing  on  its  base  will 
be  simply  a  circle  2*5  in.  diameter,  and  its  elevation  an 
isosceles  triangle,  base  2*5  in.,  and  vertex  3*5  in.  high.  For 
the  second  plan  xy  must  be  taken  parallel  to  one  of  the 
slant  sides  of  the  isosceles  triangle.  To  find  the  new  plan 
of  the  base,  circumscribe  the  base  first  drawn  by  a  square 
with  one  side  parallel  to  xy.  The  new  plan  of  this  square 
will  be  a  rectangle  in  which  an  ellipse  can  be  drawn  for 
the  new  plan  of  the  base  of  the  cone,  and  the  projection 
completed  by  drawing  tangents  from  the  new  plan  of  the 
vertex  to  this  ellipse. 

(3.  The  projection  of  the  cone  in  this  position  presents 
no  special  difficulty. 

Sections  of  the  cone  by  inclined  and  vertical  planes  may 
here  be  worked  as  in  the  case  of  the  cylinder  by  dividing 
the  base  of  the  cone  into  any  number  of  parts  and  drawing 
the  generatrices  of  the  cone  from  these  points  in  the  base 
to  the  vertex.  Treating  these  generatrices  as  the  edges  of  a 
polyhedral  pyramid,  the  projections  of  their  intersections 
with  the  section  plane  may  be  found,  as  in  Probs.  i  and  2, 
and  joined  by  a  curved  line. 

Note  on  the  true  shape  of  the  sections  of  a  right  conic 
surface.  When  the  section  plane  cuts  all  the  generatrices, 
the  section  is  a  closed  curve  and  is  generally  an  ellipse,  the 
exception  being  when  the  plane  is  at  right  angles  to  the 
axis,  in  which  case  the  section  is  a  circle.  A  plane  parallel 
to  two  generatrices  cuts  an  hyperbola,  the  limit  of  which  is 
the  pair  of  generatrices  to  which  the  section  plane  is  parallel. 


SOLIDS  IN  SIMPLE  POSITIONS.  3 1 

When  the  section  plane  is  parallel  to  only  one  generatrix 
the  section  is  2^  parabola^  and  its  limit  is  the  parallel  genera- 
trix, in  which  case  the  section  plane  becomes  a  tangent  plane 
to  the  cone. 

The  development  of  the  conic  surface  will  be  a  segment 

of  a  circle  with  radius  equal  to  a  generatrix  of  the  cone,  the 

vxdo 
circular  arc  of  which  segment  subtends  an  angle  of  ^^ — 

g 
degrees  at  the  centre;  where  r  =  radius  of  base  and  g=  length 
of  generatrix. 

6.  T'/an  and  elevation  of  a  square  prism  3 -5  in.  long, 
'.dge  of  base  2*5  i7i.,  when  its  axis  is  horizontal,  inclined 
it  45"  to  the  vertical  plane,  and  its  lowest  face  inclined  30" 
'0  H.P. 

Assume  xy.  Since  the  horizontal  axis  is  at  right  angles 
:  o  the  base  and  inclined  45",  the  base  will  be  inclined  also  at 
45"  to  the  vertical  plane,  i.e.  at  the  complement  of  the 
inclination  of  the  axis.  Take,  therefore,  a  line  in  the  H.P. 
;.t  45"  with  xy  and  assume  this  line  as  the  xy  of  an  auxiliary 
-^  ertical  plane  at  right  angles  to  the  axis  of  the  prism.  The 
]  projections  of  the  two  bases  of  the  prism  on  this  plane  will 
(  oincide  and  will  be  shown  in  true  shape.  (Theorem  VIIL). 
Draw  the  square  base  on  this  auxiliary  vertical  plane  with 
<  ne  side  at  30°  with  the  xy  and  deduce  the  plan.  This 
1  Ian  will  be  the  plan  of  the  square  prism  with  its  axis 
1  orizontal,  inclined  45"  to  the  vertical  plane  first  assumed, 
i  nd  its  lowest  face  inclined  at  30".  Complete  the  elevation 
c  n  first  vertical  plane  as  in  other  problems. 

For  the  section,  assume  three  points — one  in  each  of 
t  iree  edges  of  the  solid — and  apply  the  principle  enunciated 


32  SOLID    OR  DESCRIPTIVE    GEOMETRY. 

in  Theorem  VII.  to  the  completion  of  the  plan  and  eleva- 
tion of  the  section  made  by  a  plane  passing  through  these 
points. 

7.  Plan  and  elevation  of  a  prism,  3-5  in.  long,  with  bases 
regtdar  hexagons  of  i'^  in.  side,  when  its  lowest  face  is  inclined 
20**,  its  axis  horizontal,  and  inclined  40"  to  the  V.P.  Section 
as  i?i  the  last  problem. 

The  work  is  similar  to  that  of  the  last  problem,  the  aux- 
iliary xy  being  taken  at  50"  to  the  xy  first  drawn. 

8.  Plan  and  elevation  of  your  instrument  case  when  the 
lid  is  open  60" ;  the  long  edges  horizontal  and  inclined  30°  to 
the  V.P. 

This  is  worked  in  a  similar  manner  to  Probs.  6  and  7. 

9.  Plan  and  elevation  of  a  tetrahedron  with  one  face 
horizontal  and  i  inch  above  the  horizontal  plane.  Edge  3 
inches.  One  horizontal  edge  to  be  inclined  1$"  to  the  vertical 
plane. 

HYiQplan  will  be  an  equilateral  triangle  of  3  inches  side, 
with  the  corners  joined  to  the  centre  for  the  plans  of  the 
slant  edges  of  the  solid. 

The  base  being  horizontal,  its  elevation  will  be  a 
straight  line  and  at  the  required  height,  i  inch,  above  the 
ground  line. 

The  edge  of  the  tetrahedron  being  known,  the  height  of 
the  vertex  above  the  horizontal  base  can  be  determined  by 
making  a  vertical  section  containing  the  vertex  and  one  slant 
edge.  To  find  this,  make  a  right-angled  triangle  with  the 
length  from  a  comer  of  the  base  to  its  centre  for  the  base  of 
the  triangle,  and  the  true  length  of  the  edge  of  the  tetrahe- 


SOLIDS  IN  SIMPLE  POSITIONS.  33 

dron  for  the  hypotenuse.     The  perpendicular  will  be  the 
leight  required. 

10.  The  surface  of  a  sphere,  diameter  3  feet,  is  divided 
'nto  eqtcal  portions  by  4  great  circles,  one  horizontal  and  three 
\  vertical.     Draw  an  elevation  of  the  sphere  and  circles  on  a  plane 
parallel  to  one  of  them. 

Work  also  a  section  by  a  plane,  perpendicular  to  V.  P., 
'inclined  40",  and  '5  in.  from  centre  of  sphere.     Scale  -jV- 

11.  A  street-lamp  is  formed  of  a  right  pyramid,  and  a 
J  yustum  of  one  ;  the  slant  edges  of  former  inclined  40°.  The 
cnnmon  base  of  the  forms  is  a  square  of  20  inches  side,  and  the 
I'twer  base  or  end  of  frustmn  is  a  square  of  12  inches  side  and 
2  o  inches  below  the  other. 

(a)  Determine  an  elevation   on   a  plane  parallel  to  a 
diagonal  of  the  plan. 

(b)  Dcteri7iine  an  elevation  on  a  plane  which  makes  an 
angle  of  2,0'^  with  a  horizontal  edge. 

Scale  i. 

IhQplati  is  a  square  of  2*5  in.  with  the  opposite  corners 
j(  ined  for  the  slant  edges,  and  with  a  second  square  of  i  "5  in. 
w  ;thin  and  parallel  to  the  first. 

In  the  elevation  (a)  the  slant  edges  are  drawn  at  40"  with 
tl  e  elevation  of  the  base  of  pyramid. 

12.  A  ladder,  ^o  feet  long  and  of  tJu  uniform  width  of 
T)  feet,  rests  against  a  vertical  wall — 37^<?/'  thick,  $ofcet  high, 
2(  feet  wide — and  makes  an  angle  of  ']o'^  with  the  horizontal 
pi  ine  on  which  it  stands. 

(a)  Draw  apian,  with  an  elevation  on  a  plane  parallel 

to  that  of  the  wall. 
E.  G.  3 


34  SOLID  OR  DESCRIPTIVE  GEOMETRY. 

(b)  Aiso  an  elevation  on  a  plane  making  an  angle  of  50" 
with  that  of  the  wall. 

Scale  TO  feet  to  i  inch. 

N,B. — Rungs  and  sides  to  be  represented  each  by  a  single 
line.  Three  rungs  only  to  be  shown,  dividing  the  length  of  the 
ladder  equally. 

Commence  with  an  auxiliary  elevation  on  a  vertical 
plane  parallel  to  the  ladder. 

For  this,  draw  xy  and  lay  off  the  angle  70",  and  on  this 
line  measure  the  length  of  the  ladder :  thence  determine 
plan, 

13.  A  right  prism,  4  inches  lo?ig,  with  square  bases  of 
2*5  inches  side,  is  to  be  drawn 

(a)    with  one  diagonal  of  the  solid  vertical, 
(f)     with  the  same  diagonal  horizontal. 

Draw  a  plan  and  elevation  of  the  solid  resting  on  one 
of  the  square  bases,  with  one  diagonal  of  the  square  at  right 
angles  to  the  ground  line. 

If  two  opposite  corners  of  the  rectangle  which  is  the 
elevation  of  the  solid  be  joined,  this  Une  will  be  the  eleva- 
tion of  the  diagonal  of  the  solid,  shown  from  its  position 
in  its  true  length.  A  plan  may  then  be  determined  with 
this  diagonal  vertical  or  horizontal  by  taking  (for  a)  xy 
at  right  angles  to,  or  (for  b)  parallel  with,  the  elevation  of  this 
diagonal. 

14.  Flan  and  elevation  of  a  pyramid  with  hexagonal 
base. 

a.  With  one  face  ABV  horizontal. 

b.  When  one  face  is  vertical. 


SOLIDS  IN  SIMPLE  POSITIONS.  35 

€.      When  one  edge  is  horizontal. 
d.      When  one  edge  is  vertical. 
Determine  plan  of  the  pyramid  when  resting  on  its  base. 

For  a.  Determine  elevation  on  a  vertical  plane  taken 
£t  right  angles  with  the  plane  of  face  ABV.  Then  the 
raquired  plan  may  be  deduced  by  taking  xy  parallel  to 
t  le  line  which  represents  the  elevation  of  face  AB  V,  and 
V  orking  to  it. 

For  b.  Take  xy  perpendicular  to  the  line  representing 
the  elevation  of  face  ABV. 

For  c  and  d.  Determine  an  elevation  in  which  the  edge 
is  shown  in  its  true  length.  Then  xy  taken  parallel  for  c, 
p  irpendicular  for  d,  to  the  elevation  of  this  edge  will  place 
tl  e  p)n-amid  in  proper  position  for  the  plans  required. 

15.     Plan  and  elevation  of  an  odahedrofi,  edge  3  inches. 

(a)  When  one  axis  of  the  solid  is  vertical  and  another  is 
inclined  at  TS°  to  the  vertical  plane. 

{b)      When  resting  with  one  face  on  the  horizontal  plane. 

(a)  Describe  a  square  of  3  inches  side  and  join  the 
di  Lgonals,  which  will  be  the  plans  of  the  horizontal  axes. 
Tl.is  square  will  be  a  plan  of  the  solid  in  the  required 
pciition.  For  the  elevation  it  is  only  necessary  to  know 
th.  .t  the  length  of  the  vertical  axis  is  equal  to  a  diagonal  of 
th( :  square,  and   that  the  heights   of  the  horizontal  edges 

!ar(  half  that  diagonal.     Take  xy  at  an  angle  of  75°  with  the 
Ipk  n  of  the  diagonal. 

(b)  Work  this  first  in  the  same  way  as  Prob.  14,  (a),  and 
Ithe  n  by  the  following  special  construction  : — 

3—2 


36  SOLID   OR  DESCRIPTIVE   GEOMETRY. 

Describe  an  equilateral  triangle  of  3  inches  side, 
and  from  its  centre  with  radius  to  one  of  the  corners  de- 
scribe a  circle.  In  this  circle  complete  the  hexagon,  of 
which  the  three  corners  of  the  triangle  are  alternate  points. 
This  hexagon  with  the  alternate  points  duly  joined  is  the 
required  plan. 

For  the  elevation  it  is  necessary  to  find  the  distance 
between  the  parallel  faces.  This  is  done  by  a  vertical  sec- 
tion through  one  axis  of  the  solid.  This  section  is  a  rhom- 
bus, the  shorter  diagonal  of  which  is  an  edge  of  the  solid, 
and  the  side  of  which  is  the  altitude  of  the  equilateral  tri- 
angle forming  the  horizontal  or  any  other  face  of  the  solid. 
The  distance  between  the  opposite  sides  of  the  rhombus 
gives  the  distance  between  the  parallel  faces  of  the  solid. 

Note. — It  should  be  observed  that  this  distance  is  equal 
to  the  height  of  the  tetrahedron  whose  side  is  equal  to  that 
of  the  given  octahedron. 

{c)     Develop  the  solid. 

16.  A  solid  is  formed  from  a  cube  of  2  in.  edge  by  planes 
which  pass  through  the  middle  points  of  its  edges  and  cut  off 
the  eight  corners. 

(a)  Plan  when  resting  on  one  of  the  cubic  faces ;  and 
elevation  on  a  plane  taken  at  30°  to  one  of  the  vertical  faces  of 
the  primary  cube. 

{b)     Flan  when  a  diagonal  of  the  primary  cube  is  vertical. 

17.  A  solid  is  formed  from  a  cube  i"5  in.  edge  by 
producing  a  diagonal  of  the  cube  to  equal  distances  of  2' 5  in.  on 
opposite  sides,  and  then  taking  planes  through  the  extremities  of 
the  produced  diagonal  and  the  six  lateral  edges.  Show  plan  and 
elevation  when  resting  on  one  face. 


SOLIDS  IN  SIMPLE  POSITIONS.  3/ 

1 8.  A  square,  2  in.  side,  in  the  H.P.,  is  the  base  of  an 
o'^liqtie  prism  ;  the  edges,  4  in.  long,  are  inclined  at  55"  and  in 
f'an  parallel  to  a  diagonal  of  the  square.  Elevation  on  a 
vertical  plane  inclined  20°  with  the  plans  of  the  long  edges. 

Work  also  a  section  and  the  development  of  the  prism. 

1 9.  An  equilateral  triangle  abc,  of  3  in.  edge,  centre  v,  is 
ti  e  plan  of  a  solid,  bounded  by  plane  faces,  of  which  the  points 
A,  B,  C,  andN,  are  respectively  '25  in.,  '5  in.,  i  in.,  and  3 '5  in. 
h'gh.  Determine  the  elevatio?i  on  a  plane  taken  at  an  angle 
of  \^  with  the  line  ab  in  plan,  a?id  work  a  section  and 
development  of  the  solid. 


II. 


COMBINATIONS  AND   GROUPS   OF  SOLIDS. 

I.  A  prism,  with  bases  equilateral  triangles,  is  placed 
7vith  one  face  on  the  horizontal  plane.  On  the  centre  of  it 
rests  a  brick,  g  bj>  $  bj>  4'$  inches,  with  one  of  the  short 
edges  of  a  large  face  touching  the  ground,  with  t/ie  centre  of  that 
face  touching  the  highest  edge  of  the  prism  and  with  the  face 
itself  inclined  45".  Horizontal  edge  of  prism  9  inches.  The 
side  of  the  equilateral  base  to  be  determined  from  the  question. 
Scale  \.  Draw  plan  and  elevation,  and  a  second  elevation 
taking  the  vertical  plane  at  30°  with  any  horizotital  edge  of  the 
prism. 

Commence  with  an  elevation  so  placed  that  both  bases 
of  the  prism  coincide.  To  do  this,  draw  first  the  elevation 
of  the  brick,  which  will  be  simply  a  rectangular  parallelo- 
gram. Through  the  lowest  corner  take  xy  making  the  re- 
(juired  angle  with  the  line  that  represents  the  face  inclined 
45°,  and  make  the  centre  of  this  line  the  vertex  of  the  tri- 
angle, recollecting  that  the  elevation  of  the  base  of  that 
triangle  must  according  to  the  conditions  be  in  xy,  the 
ground  line.  The  work  for  \htplan  is  simple.  The  second 
elevation  will  be  determined  by  taking  the  new  ground  line 
30"  with  any  horizontal  edge  of  the  prism,  and  working  to  it. 


COMBINATIONS  AND   GROCTPS   OF  SOLIDS.         39 

2.  Instead  of  the  prism  take  a  cylinder  with  brick  touch- 
ing it  and  otherwise  disposed  as  above.  The  base  of  the  cylin- 
der to  be  determined  from  the  question;  i.  e.  so  t/iat  the  horizon- 
lal  bisecting  line  of  the  face  of  the  brick  may  be  in  contact  with 
I  he  cylinder. 

3.  A  block,  3  in.  square  a?td  i  in.  thick,  is  pierctd  by  a 
rexagonal  prism  i  in.  side  and  4  in.  long.  Axis  of  prism 
2' as  sing  through  centres  of  square  faces  of  block  and  projecting 
ipially  on  each  side.  Two  faces  of  the  prism  to  be  parallel  to 
two  narrow  faces  of  the  block. 

(a)  Plan  when  the  block  stands  on  one  of  its  narrow  faces 
7nth  two  faces  of  the  prism  vertical:  elevation  on  a  vertical  plane 
inclined  at  40"  to  the  short  edges  of  the  horizontal  faces  of  the 
I  lock. 

Section  and  tru£  shape  by  art  inclined  plane. 

[b)  Plan  when  two  of  the  faces  of  the  prism  not  parallel  to 
those  of  the  block  are  horizontal :  elevation  on  a  plane  at  30"  with 
the  axis  of  the  prism. 

Section  and  true  shape  by  a  vertical  plane. 

Commence  with  an  auxiliary  elevation  so  placed  that 
the  projections  of  the  bases  coincide.  In  this  position  the 
e  evation  of  the  block  will  be  a  square,  and  ■  of  the  prism  a 
haxagon,  with  its  .centre  in  the  centre  of  the  square  and  two 
c  f  its  sides  parallel  to  two  sides  of  the  square.  From  this 
aixiliary  elevation  both  the  plans  in  {a)  and  ip)  can  be 
c  2termined,  and  thence  the  required  elevations. 

4.  The  faces  of  a  tetrahedron  3"  edge  are  the  bases  of  four 
p  nsms  whose  axes  are  equal  in  length  to  half  the  height  of  the 
ti  trahedron.  Plan  and  elevation  when  an  axis  of  the  tetrahe- 
a  'on  is  vertical. 

5.  Draw  a  line  5  in.  long  in  the  H.P.  parallel  to  xy. 


40  SOLID   OR  DESCRIPTIVE   GEOMETRY, 

From  each  end  of  this  line  cut  off  segmcjits  of  i'^  in.,  and 
describe  on  them  two  regular  pentagons.  Consider  these  as  the 
bases  of  two  prisms  2*5  in.  long,  and  make  a  plan  and 
elevatio7i  when  spanned  by  a  pentagonal  Gothic  arch — the 
cetitres  of  the  circles  as  shown  in  elevation  being  the  middle 
points  of  the  elevation  of  the  upper  bases. 

6.  A  solid  is  forjned  of  a  slab  i  in.  thick,  having  regular 
hexagons  of  2  in.  side  for  its  faces,  and  aii  equilateral  triangular 
prism  4  i7i.  long  passing  through  the  slab  and  projecting 
equally  from  each  of  its  faces — the  long  edges  of  the  prism 
containing  the  alternate  short  edges  of  the  slab.  Plan  and 
elevation  when  resting  on  one  corner  of  the  slab  atid  one  edge  of 
the  base  of  the  prism. 

7.  Plan  and  elevation  of  a  cylinder,  4  inches  long, 
diameter  of  base  2  inches,  passing  through  the  centre  of  a 
circular  slab,  i  ifich  thick  and  3  inches  in  diameter;  the  axis 
of  the  cylifider  at  right  angles  to  and  extending  equally  from 
both  faces  of  the  slab.  The  whole  to  be  placed  resting  on  the 
rim  of  the  slab  and  the  rim  of  the  cylinder. 

8.  Describe  three  circles  with  radii  i'5  in.,  each  touching 
the  other  two,  and  join  their  centres.  Co?isider  the  circles  as  the 
plans  of  three  spheres  resting  on  the  horizontal  plane,  and  the 
triangle  as  the  plan  of  the  base  of  a  tetrahedron  resting  upon 
them.  Co?nplete  plan  and  elevation  of  the  whole,  and  draw 
a  second  plan  when  ojie  of  the  slant  edges  of  the  tetrahedron  is 
horizontal. 

For  the  second  plan  take  xy  parallel  to  the  elevation  of 
any  slant  edge  of  the  tetrahedron. 

9.  A  tetrahedron,  edge  2*5  inches,  stands  on  a  square 
block,  with  an  edge  parallel  to  a  side  of  the  sqjiare.     The  block 


COMBINATIONS  AND   GROUPS  OF  SOLIDS.         4 1 

is  I  inch  thick,  a?id  its  square  faces  have  sides  of  2,'S  i^<^hes. 
An  axis  of  the  tetrahedron,  if  produced,  passes  through  the 
centres  of  the  squares. 

Draw  plan  and  elevation — 
{a)      When  the  square  faces  of  block  are  horizontal  and 
the  plane  of  elevation  makes  30"  with  a  side  of  a 
square, 
{b)      When  only  one  slant  edge  of  tetrahedron  is  horizontal. 

10.  A  circular  slab,  i  inch  thick,  diameter  3  inches,  rests 
on  ofie  face.  On  it  is  placed  a  tetrahedron  of  2  inches  edge, 
axis  vertical  and  over  the  centre  of  the  slab.  Draw  plan  and 
elevation,  and  a  second  plan  when  the  rim  of  the  slab  and  the 
vertex  of  the  tetrahedron  tojich  the  horizontal  plane. 

For  the  second  plan  take  xy  touching  the  elevations  of 
the  vertex  and  the  rim  of  the  slab. 

11.  A  solid  is  formed  from  a  cube  of  7,  in.  edge  by  placing 
a  square  pyramid  on  each  of  its  faces;  axis  of  pyramid  equal 
to  half  the  diagonal  of  the  face.  Plan  and  elevation  when  one 
face  of  the  solid  is  horizontal. 

12.  A  regular  hexagonal  slab  75  in.  thick,  side  of 
hexagon  1*5  in.,  stands  on  one  of  its  rectangular  faces,  the  long 
edges  of  which  itiake  angles  of  40°  with  xy.  A  sphere  of 
2*25  in.  diameter  is  embedded  in  the  slab,  the  centre  of  the 
sphere  coinciding  with  the  centre  of  one  of  the  hexagonal  faces. 
Show  plan  and  elevation. 

13.  An  equilateral  triangle  of  ^  in.  side  is  the  outline  of 
the  plan  of  four  tetrahedrons,  2  in.  edge  each,  three  of  which 
stand  upon  the  H.  P. ;  the  fourth  rests  upon  the  others  with 
the  three  corners  of  its  horizontal  face  coinciding  with  their 
summits.     Plan  and  elevation. 


III. 


PROBLEMS   ON  THE  STRAIGHT  LINE 
AND   PLANE. 

Obs.  To  express  points,  lines  and  planes,  see  Problems 
I  to  7. 

Problem  I. 

Given  the  projectmis  ab,  a'b'  of  a  finite  straight  line  to 
determine: 

1.  The  length  of  the  segment  AB  of  the  given  line,  or 
the  distance  between  points  A  and  B. 

2.  The  angle  of  inclination  of  the  line  to  each  plane  of 
projection. 

3.  The  traces  of  the  given  line  and  the  distance  between 
them. 

I.  The  line  AB  lies  in  each  of  its  projecting  planes: 
'construct'  or  rabat  either  of  these  into  the  corresponding 
plane  of  projection  and  the  segment  AB  is  shown. 

Let  the  projecting  plane  for  the  plan  ab  revolve  about 
this  line  as  an  axis,  into  the  horizontal  plane. 


STRAIGHT  LINE  AND  PLANE. 
Fig.  6. 


43 


From  points  a  and  b  set  off  the  projectors  aA  and  bB, 
equal  to  aa'  and  )8^'  respectively,  and  at  right  angles  to  ab, 
on  the  same  side  of  it. 

Join  A  and  B ;  AB  is  the  required  distance  or  length 
of  segment. 

Observe.  The  length  AB  (being  the  hypotenuse  of  the 
right-angled  triangle  of  which  the  sides  are  ab  and  the 
difference  of  the  projectors  Bb  and  Aa)  may  be  found  by 
setting  this  difference  .from  b  along  bB  to  B'  and  joining 
B'  to  a. 

2.  The  angle  a  straight  line  makes  with  a  given 
plane  is  the  angle  between  the  line  and  its  orthographic 
projection  on  that  plane,  or  the  angle  which  AB  makes 
with  ab,  and  is  therefore  the  angle  6. 


44  SOLID    OR  _DESCRIPTIVE    GEOMETRY. 

For  the  same  reason  ^  is  the  angle  which  the  given  line 
makes  with  the  vertical  plane  of  projection. 

3.  The  'traces'  of  the  line  being  points  in  the 
planes  of  projection  have  each  of  them  a  projection  in  xy, 
viz.  h'  and  v  (Theorem  VI.) ;  but  the  elevation  db'  pro- 
duced contains  h' ,  therefore  the  intersection  of  these  lines, 
namely,  xy  and  the  elevation,  gives  point  //';  and  Theorem 
IV.  and  the  plan  of  the  line  give  point  h.  The  correspond- 
ing loci  of  V  and  v'  determine  those  points. 

The  distance  between  the  traces  H  and  V  of  the  given 
line  is  obviously  the  length  h  F,  as  shown  in  figure. 

Corollaries  given  below. 

As  an  exercise  on  this  problem  determine  the  inclina- 
tions of  the  given  lines,  AB,  CD,  EF,  &c.,  and  their  traces, 
and  the  lengths  of  the  segments  indicated.     See  Fig.  4. 

Converse  of  Problem  i  : 

(i)     Given  the  traces  of  a  straight  line;  or 

(2)  Its  inclination  and  plan ;  or 

(3)  Its  length  and  inclination ;  or 

(4)  Length  and  the  position  or  loci  of  the  extremities 
of  a  finite  portion ;  to  draw  or  determine  the  projections  of 
the  line. 

(i)     If  the  traces  hh'  and  vv'  be  given, — 
Join  h  and  v  and  h'  and  v'. 
Then  hv,  h'v'  are  the  projections  required. 

(2)  If  the  plan  hv  be  given  making  a  given  angle  hva. 
with  xy,  and  the  line  inclined  at  an  angle  of  0  degrees. 

At  any  point  h  in  the  given  plan  draw  h  V,  making  the 
angle  6"  with  hv. 


STRAIGHT  LINE  AND  PLANE.  45 

Assume  points  a  and  b  in  the  plan,  draw  the  perpen- 
diculars aA  and  bB,  and  find  points  a'  and  b'  by  Theorems 
IV.  and  V.  and  join  a ,  b'. 

Then  ab,  a'b'  are  the  projections. 

(3)  If  the  length  AB  be  given,  its  inclination  6,  and 
the  position  aa'  of  one  extremity  A. 

Through  a  draw  the  indefinite  plan  hv. 
Set  a  A  at  right  angles  to  ab  and  equal  to  ad. 
Through  A  draw  h  V,  making  the  angle  6  with  kv. 
From  A  set  off  AB  along  this  line  and  determine  b  and  b'. 
ab,  a'b'  are  the  required  projections. 

(4)  If  the  length  AB  be  given  and  the  positions  of 
its  extremities  A  and  B  with  regard  to  the  two  planes. 

Draw  ad  the  projections  of  one  extremity  A,  from  the 
conditions.     Theorems  IV.  and  V. 

Also  any  point  //'  from  the  conditions  for  the  other 
extremity  B  and  through  /  draw  a  parallel  to  xy.  this  is 
a  locus  of  b  the  other  extremity  of  the  plan  ab  required. 
Determine  the  length  ab"^  of  the  plan ;  measure  this  length 
from  a  to  a  point  b  in  the  parallel,  and  the  position  of  ab 
is  fixed ;  b'  may  be  found  of  the  height  'of  /',  and  ab,  a'b' 
are  determined. 

The  student  will  observe  that  the  problems  admit  of 
one,  two,  or  more,  or  an  infinite  number  of  solutions.  These 
cases  should  be  studied.     See  Problem  2. 

*  The  base  of  a  right-angled  triangle,  hypotenuse  AB  and  altitude 
equal  the  difference  of  the  heights  of  A  and  B% 


4^  SOLID  OR  DESCRIPTIVE  GEOMETRY. 

I.  (3),  Cor.  I.  To  determine  the  ^ traces^  of  a  given  plane 
when  the  data  are  /       ,  - 

(i)  Two  straight  lines  {parallel  or  meeting. 

(2)  A  straight  line  and  a  point, 

(3)  Three  points. 

(1)  The  horizontal  and  vertical  traces  of  the  given 
lines  must  be  found  by  Problem  i,  and  the  corresponding 
traces  of  the  plane  drawn  through  the  points  thus  found. 
Theorem  X. 

(2)  Find  the  traces  of  the  given  line. 

Draw  the  projections  of  a  line  parallel  to  it,  through 
those  of  the  given  point  (Theorem  VII.),  and  find  its  traces. 

Or,  by  Theorem  IV.,  determine  the  projections  of  any 
point  qq'  in  the  given  straight  line ;  join  the  projections  of 
this  point  with  pp'  those  of  the  given  one;  then  the  line 
FQ  lies  in  the  given  plane. 

The  traces  of  the  two  lines  give  those  of  the  plane. 

(3)  Join  the  corresponding  projections  of  the  given 
points;  and  determine  the  traces  of  the  three  lines  of 
which  those  joining  lines  are  the  projections. 

The  traces  of  the  three  lines  give  those  of  the  plane. 

Obs.  If  the  given  plane  meets  xy  only  three  traces  of 
tie  lines  lying  in  it  need  be  found  ;  e.g.  two  in  the  horizon^ 
tal  trace  of  the  plane  and  one  in  its  vertical  trace  :  the 
latter  joined  to  the  point  of  intersection  of  the  horizontal 
trace  of  the  plane  with  xy  gives  the  vertical  trace  of  the 
plane. 

I.  Cor.  2  (see  figure).  Given  the  traces  of  a  plane  and 
one  projection  zh  of  a  straight  line  which  lies  in  that  plane; 
to  determine  the  second  projection  a'b'  of  the  line.   ■ 


STRAIGHT  LINE  AND  PLANE. 


47 


Since  the  traces  M'  and  vv'  of  the  line  must  be  m  those 
of  the  given  plane  (Theorem  X.),  determine  these  points, 
then  the  line  drawn  through  points  /i',  z/  is  a  If'  the  required 
elevation. 

Fig.  7. 


Observe.  Point  H  lies  in  the  given  straight  line  AB, 
hence  the  projections  M'  of  the  point  are  in  those  of  the 
line.  Thus  ab  is  a  locus  of  h^  and  of  v;  the  horizontal  trace 
Df  the  plane  is  a  second  locus  of  h,  and  xy  of  v,  which 
ooints  are  thus  determined  :  perpendiculars  to  xy  through 
hem  are  loci  of  >^'  and  v\  Theorem  IV.,  which  points  are 
hus  also  determined. 


48 


SOLID   OR  DESCRIPTIVE  GEOMETRY. 


I.     Cor.  3.     To  determine  a  point  pp'  of  given  altitude 
a  inches  {or  a")  in  a  given  plane  hav'. 

Fig.  8. 


In  the  given  vertical  trace  arj ,  take  a  point/',  «"  high, 
Theorem  V. ;  find/  in  x}\  Theorem  IV. ;  then/?)'  is  the  re- 
quired point. 

I.  Cor.  4.  To  determine  a  horizontal  line  a"  high^  in  a 
given  plane  hav'.     Fig.  8. 

Determine/^"  as  before. 

Through/  draw/<^  parallel  to  ho,  Theorem  VII. 

Through/'  draw/'/  parallel  to  xy. 

Then/^,  /'/  is  the  required  line. 


STRAIGHT  LINE  AND  PLANE. 


49 


Problem  II.       ;   , 

Through  a  given  point  aa'  to  draw  a  straight  line  in- 
dined  6°  to  the  horizontal  plane. 

Fig.  9. 


B^ 


If  a  right  cone  be  determined  having  its   vertex  in 
tlie  given  point  A,  axis   equal  to   a' a  and   generatrix   in- 
<  h'ned  dy  every  generatrix   of  the  cone  fulfils  the  given 
E.  G.  4 


50  SOLID   OR  DESCRIPTIVE   GEOMETRY. 

conditions.     The  conic  surface  is  therefore  a  locus  of  the 
required  line. 

To  work  the  problem  :     " 

Through  point  d  draw  a! s'  making  the  angle  d s' a  equal 
to  b. 

Through  a  draw  as  parallel  to  xy,  then  as,  a's'  is  parallel 
to  the  vertical  plane  and  inclined  6". 

With  centre  a  and  radius  equal  to  as'  describe  the 
circular  base  klm  of  the  cone  which  gives  the  complete 
solution. 

Observe.  The  four  lines  AH,  AK,  AL,  AM  are  in- 
clined $°,  and  make  equal  angles  with  the  vertical  plane. 

Problem  III. 

To  determine  a  straight  line  which  shall  make  a  given 
angle  6  with  the  horizontal  plane  and  an  angle  <}>  with  the 
vertical  plane  of  projection. 

A  line  inclined  6°  will  lie  on  the  surface  of  a  right 
cone  with  axis  vertical  and  generatrix  inclined  B"  to  the 
base. 

Determine  such  a  cone  having  its  vertex  in  point  oo' 
in  the  plane  of  elevation :  Rps  will  be  its  circular  base. 
That  generatrix  of  the  cone  which  makes  an  angle  ^  with 
the  vertical  plane  will  be  the  line  required.  To  find  this. 
Knowing  the  length  dR  of  this  generatrix,  determine  the 
length  dr  of  its  elevation  as  in  the  figure ;  and  since  the 
-position  of  o'  one  extremity  of  it  is  fixed,  that  of  the  other, 
viz.  p',  in  xy,  may  be  found  as  indicated.  The  perpendi- 
cular to  xy  from  /'  is  a  locus  of/,  and  intersects  the  cir- 


STRAIGHT  LINE  AND  PLANE. 


51 


cular  arc  Rps,  which   is  another   locus  of  /,  which  point 
is  thus  determined.    Join  op^  o'p'  \  it  is  the  required  line. 
Fi-j.  10. 


It  is  obvious  that  there  are  four  such  lines  on  the 
uirface  of  the  cone,  and  therefore  there  are  four  solutions 
Df  the  problem,  when  00'  is  a  given  point  in  the  line. 

It  will  also  appear  from  the  above,  that  the  second 
ingle  <^  cannot  exceed  the  complement  of  6  or  (90-^) 
legrees. 

4—2 


52  SOLID   OR  DESCRIPTIVE  GEOMETRY. 

Problem  IV. 

To  draw  or  determine  the  traces  of  a  platie  inclined  6  de- 
grees and  perpejidicular  to  the  vertical  plane  of  projection. 
See  Fig.  5,  'Planes.' 

Draw  the  horizontal  trace  ql  of  the  required  plane  at 
right  angles  to  xy,  meeting  it  in  point  /;  and  through  / 
draw  hn!  making  the  angle  ylm'  equal  0 :  qh/i'  is  the  re- 
quired plane. 

It  is  obvious  that  Im  and  ly  are  the  two  perpendiculars 
to  ql,  the  horizontal  trace,  or  common  section  of  the 
planes,  at  the  point  /  in  it,  which  form  the  measuring-angle 
of  the  inclination  of  these  planes  to  each  other. 

The  plane  of  Im'  and  ly  having  been  turned 
into  the  horizontal  plane,  Im'  in  Fig.  is  no 
longer  perpendicular  to  ql. 

Similarly  (see  Fig.  5,  'Planes')  a  plane  n^o'  may  be 
drawn  at  right  angles  to  the  horizontal  and  making  an 
angle  <l>  with  the  vertical  plane  of  projection. 

Problem  V. 

To  determine  the  angles  6  and  ^  which  a  given  plane 
hav'  or  am,  mV  makes  with  the  planes  of  projection ;  and 
to  determine  the  angle  betweeti  its  traces. 

Since  a  dihedral  angle  is  measured  by  the  plane  angle 
formed  by  two  perpendiculars  drawn  one  in  each  plane 
to  the  same  point  in  their  common  section,  the  plane  of 
this  rectilineal  angle  is  perpendicular  to  the  'trace'  of  the 
given  plane.  '  Rabat '  the  plane  of  this  angle,  revolving 
it  about  that  line  of  it  which  is  in  the  plane  of  pro- 
jection in  question,  and  the  required  angle  of  inclination 
is  obtained. 


STRAIGHl^  LINE  AND  PLANE. 


53 


Thus  at  point  a  in  the  horizontal  trace  ha.  (Fig.   ii) 
two  such  perpendiculars  av  and  aV  were  found,  and   at 

Fig.  ir. 


p  )int  d'  in  the  vertical  trace  perpendiculars  dli  and  d'H, 
a  id  the  required  angles  shown. 

To  determine  the  magnitude  of  the  angle  ha.v'  between 
tl  e  traces  of  the  plane. 

Let  the  plane  be  'constructed'  or  rabatted  about  its 
h  )rizontal  trace  ha  until  it  coincides  with  the  horizontal 
p  ane. 


54  SOLID   OR  DESCRIPTIVE   GEOMETRY. 

Then  the  length  a  V  measured  from  a  along  av  pro 
duced   to    V^  gives  a  second  point  in  the  second  line  of 
the  required  angle.     Join  aFg,  and  ha.V^  is  the  angle  be-^ 
tween  the  traces  of  the  plane. 

Problem  VI. 

Given  the  horizontal  trace  ah.  of  a  plane  inclined  6  to 
determine  its  vertical  trace  av'. 

At  any  point  a  in  the  horizontal  trace  draw  av  at  right 
angles  to  it  and  meeting  xy  in  v.  Fig.  ii.  Draw  aV 
making  the  angle  z'aF  equal  to  the  given  angle  0.  From  v 
draw  vV  2i\.  right  angles  to  aVy  and  from  v  set  oflf  the  length 
vv'  at  right  angles  to  xy  and  equal  to  vK  Then  the 
straight  line  through  a  and  v  is  the  vertical  trace  required. 


Problem  VII. 

To  determine  the  traces  of  a  plane  making  an  angle  of 
B  degrees  with  the  horizontal  and  a?i  angle  of  ^  degrees  with 
the  vertical  plane. 

When  a  plane  touches  a  right  cone  it  is  inclined  at  the 
angle  which  the  generatrix  of  the  cone  makes  with  its  base. 

Determine  two  right  cones  (Fig.  1 2)  with  vertices  q'  and 
v  and  bases  tuw  and  sr'ni  in  the  horizontal  and  vertical 
planes  respectively;  the  generatrix  //  making  an  angle  6 
with  its  base  tuw,  and  generatrix  vs  an  angle  <^  with  its  base 
sr'm'.  The  common  tangent  plane  to  these  cones  is  the 
plane  required. 

The  surfaces  of  the  cones  have  a  common  point  // 
which  is  also  on  the  surface  of  a  common  sphere  enveloped 


STRAIGHT  LINE  AND  PLANE. 


55 


by  them  and  having  its  centre  o  in  xy.     See  Figure  13. 
Hence  the  plane  of  points  P,  Q,  and  Fis  that  required. 

Fig.  12.  .   ' 


\ 

V 

m 

/ 

^ 

t\ 

/ 

/ 

0 

/^ 

/^l 

\aI 

wX 

:  ?/ 

\ 

Sv 

\U 

// 

/\v 

V 

^ 

\)^  // 

w 

^ 

\ 

\ 

^^^^ 

/ 

/ y^ 

\ 

tr 

^ 

To  work  the  problem. 

Commence  with  one  of  the  vertices,  assuming  point  Q 

or  /  in  the  vertical  plane.    Fig.  12.     Draw^V  indefinitely  at 

ight  angles  to  xy^  meeting  it  ino;  and  draw  ^V  as  above 

lescribed,  faking  angle  d  with  xy,  on  which  let  fall  the 

)erpendicular  otf  from  0;  the  circle  described  with  centre  0 

.  .nd  radius  oc  gives  the  common  inscribed  sphere  of  the 


56 


SOLID   OR  DESCRIPTIVE  GEOMETRY. 


two  cones,  vs  drawn  as  above  mentioned,  making  the  angle 
^  with  xy,  and  touching  this  circle  in  d^  meets  <^o  produced 
in  the  point  v^  which  is  the  second  vertex.  Tangents  to 
the  bases  of  these  cones  through  v  and  /  meet  xy  in  point 
a  and  give  the  traces  of  the  required  plane  qav. 

Fig.  13. 


Limits.     The  sum  of  the  given  inclinations  or  5  +  ^  lies 
between  90"  and  180**. 


STRAIGHT  LINE  AND  PLANE. 


57 


Obs.     To  express  given  combinations  of  points,  lines, 
and  planes,  see  Problems  8  to  28. 

Problem  VIII. 
To  determine  a  straight  line  inclined  &^  to  lie  in  a  given 
plane  qb'm'.  , 

Fig.  14. 


5^  SOLID   OR  DESCRIPTIVE   GEOMETRY. 

Limits.  The  angle  ^  is  the  angle  of  inclination  of  the 
given  plane.  It  is  therefore  obvious  that  the  angle  of  incli- 
nation 0  of  the  given  line  must  be  not  greater  than  <f>. 
For  no  straight  line  can  lie  in  a  plane  and  have  a  greater 
inclination  than  that  of  the  plane. 

Construction.     Assume  point  aa'  in  the  given  plane. 

Make  the  angle  a'fia  equal  0. 

With  centre  a  and  radius  a^  describe  the  arc  ^ed  meet- 
ing the  trace  ^d'  in  d  or  d;  then  alf',  db'  or  ad^  a'd'  is  the 
required  line. 

Observe,  that  each  of  the  lines  AB  and  AD  satisfies 
the  conditions  of  the  problem. 

The  cone,  vertex  A  and  base  deb,  is  the  locus  of  all 
lines  through  A  inclined  6. 

The  given  plane  is  a  second  locus  of  the  required  line. 
And  these  loci  intersect  in  the  lines  AB  and  AD  only. 

Problem  IX  {converse  of  Problem  VIII). 

Through  the  given  straight  line  ab,  a'b'  {which  is  inclined 
6)  to  draw  a  plane  inclined  ^. 

Since  the  given  line  is  to  lie  in  the  required  plane,  its 
traces  must  lie  in  those  of  the  plane,  Theorem  X.  De- 
termine these  traces  bb',  vv'.     Problem  i. 

Determine  a  right  cone,  with  vertex  in  point  aa'  of  the 
given  line,  and  base  tuk  in  the  horizontal  plane,  its  gene- 
ratrix making  the  given  angle  <^  with  the  base.  Every 
tangent  plane  to  this  cone  is  inclined  <f>^,  Theorem  XL  ; 
and  the  straight  line  through  b  touching  the  base  of  the 


STRAIGHT  LINE  AND  PLANE. 
.      Fig.  15. 


59 


\«' 


\ 

X 

i\ 

/ 

I 

cone  in  /  is  the  horizontal  trace  of  such  plane,  Theorem 
XII.  By  producing  this  line  to  meet  xy  in  s  and  joining 
.  v',  we  obtain  the  vertical  trace  of  tht  required  plane. 

For  this  plane  tsv,  passing  through  points  A  and  B  in 
the  given  line,  contains  that  line;  and  touching  the  cone 
l.a5  the  required  inclination. 

Note :  That  the  radius  at  of  the  base  of  the  cone  is  taken 
(  qual  to  a^  in  figure. 

Also,  that  a  second  plane  bwv',  having  ub  for  its  hori- 
2  ontal  trace,  likewise  satisfies  the  conditions  of  the  problem. 

Generally,  two  planes  are  possible  as  long  as  </>"  exceeds 
l^ ;  when  «^''  is  equal  to  6°  only  one  plane  can  be  drawn, 
i  nd  when  <^'*  is  less  than  6"  the  problem  is  impossible. 


6o 


SOLID   OR  DESCRIPTIVE   GEOMETRY. 


Problem  X. 
To  determine  the  magnitude  of  the  angle  which  the  two 
given  straight  lines  OH,  OF  make  with  each  other. 

Fig.  I 6. 


<r\ 


Determine  the  horizontal  traces  ff'  and  hH  of  the  given 
lines,  then  (Theorem  X.)  the  horizontal  trace  //3  of  the 


STRAIGHT  LINE  AND  PLANE,  6 1 

plane  of  the  lines  passes  through  points  /  and  h,  and  its 
vertical  trace  ^v  through  ?/,  vv  being  the  vertical  trace 
oi  OF. 

'Rabat'  the  plane  j'^Z'',  revolving  it  about  its  tmce/ji 
into  the  horizontal  plane;  then  fOA  the  required  angle  is 
exhibited. 

The  indefinite  perpendicular  on  the  horizontal  trace 
through  the  point  <?  is  a  locus  of  O  (Theorem  XX.),  and  the 
length  Oa  is  made  equal  to  the  hypotenuse  of  the  right- 
angled  triangle,  of  which  oa  is  the  base  and  o'e  the  altitude. 

Cor.  To  bisect  the  angle  formed  by  two  given  straight 
lines ^  as  OY  and  OH. 

'Rabat'  the  plane  of  the  angle  as  above;  and  bisect 
the  angle  fOh  thus  found  by  a  straight  line  Os  meeting  the 
horizontal  trace  used  in  point  s;  obtain  s'  in  xy^  and  join  os, 
ds' ',  these  are  the  projections  of  the  required  bisecting  line. 

Problem  XI.     (See  Fig.  16.) 

To  draw  a  straight  line  oh,  o'h'  through  a  given  point 
pp'  to  make  a  given  angle  9  with  a  given  straight  line  of,  o'f . 

Determine  the  horizontal  trace  f^  of  the  plane  of  the 
Tiven  straight  line  OF  and  point  P  (Prob.  i.  Cor.  i) 
•Rabat'  this  plane,  and  through  P  so  'rabatted,'  draw 
;he  line  OP  making  the  given  angle  0  with  OF  in  the 
lorizontal  plane.  Determine  the  projections  00'  of  point  O, 
oin  op,  o'p',  they  are  the  projections  of  the  required  line. 

Note:  That  the  construction  of  this  problem  is  made 
nore  simple  by  using  a  second  plane  of  elevation  perpen- 
iicular  to  that  of  the  angle.     Theorem  VI.  (2). 


62 


SOLID  OR  DESCRIPTIVE  GEOMETRY. 


Problem  XII. 

To  determine  the  projections  of  the  intersection  of  two  given 
planes  hav'  and  h(3v'. 

Fig.  17.  "         .    •        ■ 


/'Sf 


The  planes  intersect  in  a  straight  line,  which  lying  in 
both  planes  has  its  traces  in  those  of  the  planes.  Hence 
z',  in  which  the  vertical  traces  meet,  is  the  vertical  trace  of 
the  line  of  intersection,  the  plan  v  of  this  point  is  in  xy^ 
and  is  found  by  a  perpendicular  to  it  (Theorem  IV.).    Simi- 


STRAIGHT  LINE  AND  PLANE.  63 

larly,  h  is  the  horizontal  trace  of  the  line  required,  and  //', 
its  elevation,  is  found  in  xy  by  a  perpendicular  to  that  line. 

Join  hVy  h'v',  these  are  the  projections  of  the  required 
line. 

Cor.  I.     To  determine  the  intersection  of  three  given  planes. 

Determine  the  intersection  ZTF  as  in  the  problem,  of 
planes  (i)  and  (2) :  then  the  intersection  AB  of  planes  (2) 
md  (3).  AB  will  meet  HV  in  a  point  P  which  is  the 
ntersection  required  and  is  the  vertex  of  the  solid  angle 
"ormed  by  the  three  given  planes. 

Cor.  2.  To  determine  a  straight  line  which  shall  pass 
^hrough  a  given  point  00'  and  meet  two  given  straight  lines 
■lb,  a'b',  and  de,  d'e'. 

Determine  the  traces  of  the  plane  of  the  point  O  and 
he  line  AB ;  likewise  those  of  the  plane  of  point  O  and 
'  he  straight  line  DE.     Then  the  intersection  of  these  two 
)lanes  is  the  line  required. 

Or,  find  the  intersection  of  one  of  the  given  lines  with 
he  plane  containing  the  other  and  the  given  point  O.  The 
ine  drawn  from  this  point  of  intersection  to  O  will  be  the 
ine  required. 

'        ■      Problem  XIII.      , 

To  determine  a  straight  line  at  right  angles  to  a  given 
j'lane  qlm'  which  shall  pass  through  a  given  point  pp'. 
:<ig.  18. 

Since  the  projections  of  a, perpendicular  to  a  plane  are 
]  erpendiculars  to  the  'traces'  of  the  plane,  Theorem  IX. ; 
.^  s  drawn  through  a  point  /  at  right  angles  to  ql,  and  /'/  at 
1  ght  angles  to  lm\  are  the  projections  of  the  required  line. 


64 


SOLID  OR  DESCRIPTIVE  GEOMETRY. 


Conversely.  To  determine  a  plane  at  right  angles  to  a 
given  straight  line  ps,  p's'  which  shall  pass  through  a  given 
point  ii'  in  the  same. 

Fis.  1 8. 


Draw  the  traces  ^a,  ea.  of  any  plane  da£\  by  the  pro- 
blem, at  right  angles  to  the  given  line. 


STRAIGHT  LINE  AND  PLANE.  6$ 

Determine  the  projections  of  any  line  lying  in  this  plane 
and  through  the  given  point  draw  a  parallel  ih,  i'h',  to  the 
line  determined,  and  find  its  trace  hh'. 

Through  h  draw  gl  at  right  angles  to  ps  meeting  xy  in  /, 
and  through  /  draw  Im'  at  right  angles  to  p's'  :  qlm'  is  the 
required  plane. 

Cor.  I.  To  determine  the  distance  of  a  given  point  pp' 
from  a  given  plane  qlm'. 

Before  reading  this  corollary  study  the  following  Pro- 
)lem  No.  14,  and  by  it  determine  point  //'  in  which  the 
])erpendicular  PS  to  the  plane  intersects  it. 

Then  by  Problem  1  determine  the  length  IP  of  the 
])erpendicular  of  which  ip,  i'p'  are  the  projections.  IP  is 
the  required  distance  of  the  given  point  /'from  the  plane. 

Cor.  2.  To  determine  two  parallel  planes  at  a  given  dis- 
iince  (n)  inches  from  each  other. 

Determine  a  straight  line  BC,  and  in  it  mark  off  the 
Imgth  PQ^  points  P  and  Q_  being  («)  inches  apart;  the 
planes  through  these  points  at  right  angles  to  the  line  BC 
are  those  required. 

Problem  XIV. 

To  determine  the  point  of  intersection  of  the  given  straight 
li'ie  ps,  p's',  with  the  given  plane.     Fig.  18. 

Every  plane  which  contains  the  given  line  meets  the 
g  ven  plane  in  a  straight  line  which  passes  through  the 
r(  quired  point  of  intersection  of  the  given  straight  line  and 
p  ane.  Assume  any  such  plane  pmm\  of  which  pm  and 
m  'n'  are  the  traces. 

Determine  the  line  of  intersection  hm^  Jim!  of  this  as- 
si  med  plane  with  the  given  one. 

E.  G.  5 


66 


SOLID   OR  DESCRIPTIVE   GEOMETRY. 


The  projections  ii'  of  the  Intersection  of  these  two  lines 
HM  and  PS  are  those  of  the  required  point. 

Another  mode  of  solving  this  problem  is  to  make  an 
elevation  of  the  given  line  and  plane  on  a  plane  at  right 
angles  to  the  latter :  point  /'  in  this  elevation  will  be  the 
intersection  of  the  elevation  of  the  line  with  the  vertical 
trace  of  the  plane :  whence  the  plan  /  may  be  obtained. 

Problem  XV. 
To  determine  the  angle  which  the  two  given  planes  qlm', 
qq'm'  make  with  each  other,  i.  e.  to  determine  their  ^profile' 
or  '■measuring  angle.'' 

Fig. 


STRAIGHT  LINE  AND  PLANE.  67 

This  plane-angle  is  formed  by  two  straight  lines  drawn 
one  in  each  plane  from  a  point  in  their  common  section 
QM  at  right  angles  to  that  line.  (Euc.  Bk.  xi.  def.  6.) 
The  plane  of  these  perpendiculars  is  therefore  (Euc.  Bk.  xi. 
\)  at  right  angles  to  the  line  QM:  hence  if  this  plane  be 
determined  and  'rabatted'  about  its  horizontal  trace, 
he  required  angle  wTr  is  shown. 

To  do  this,  assume  the  horizontal  trace  rw  of  the  plane 
of  required  angle,  at  right  angles  to  qnt^  meeting  it  in  j,  and 
1  he  horizontal  traces  of  the  given  planes  in  r  and  w. 

Fold  the  plane  about  rw  bringing  the  point  T  in  QM 
down  into  the  horizontal  plane  in  qm :  join  Tr,  Tw. 

The  distance  Ts  is  found  by  making  it  equal  to  Ts  on 
the  vertical  plane  (as  shown),  this  line  being  the  common 
section  of  the  'projecting  plane'  of  the  plan  qjn  with  the 
1  lane  of  the  measuring  angle. 

Conversely.  Given  Q  the  profile  or  measuring  angle  of 
t  le  inclination  of  two  planes  to  each  other,  to  determine  the 
t  aces  of  the  planes. 

Draw  the  plane  qq'm  and  determine  the  straight  line 
q  m,  q'ni  in  it. 

Draw  rs  at  right  angles  to  qm  and  determine  j 7"  from 
^T  as  before  and  join  rT:  then  make  the  angle  rTw  —  6^ 
a  id  point  w  is  determined,  through  which  draw  ql  the 
h  Drizontal  trace  of  the  second  plane  :  Im'  its  vertical  trace 
i^  drawn  through  the  vertical  trace  of  the  line  QM 
("heorem  X.). 


68 


SOLID   OR  DESCRIPTIVE   GEOMETRY. 


Problem  XVI. 

Through  a  given  point  aa'  to  draw  a  plane  parallel  to  a 
given  plane  qlm'. 

Fig.  20. 


Since  if  two  parallel  planes  be  cut  by  a  third  plane, 
their  sections  with  the  third  are  parallel  straight  lines  (Euc 
XI.  1 6),  therefore  the  traces  of  the  given  and  required  planes 
will  be  parallel  straight  lines. 


STRAIGHT  LINE  AND  PLANE.  69 

^  If  therefore  the  given  plane  be  not  parallel  to  xy,  one 
ioint  in  either  of  the  required  traces  being  found  both  those 
ines  can  be  obtained. 

Through  the  given  point  A  let  a  straight  line  be  drawn 
]  parallel  to  either  trace  of  the  given  plane;  the  trace  of  this 
line  supplies  the  required  point. 

Therefore  through  A  or  ad  draw  the  straight  line  A  V 
(>r  av,  a'v'  parallel  to  ql  which  is  a  horizontal  line;  the 
trace  -mf  of  this  line  gives  the  point  v'  in  the  vertical  trace 
/?z/  of  the  required  plane :  and  ph  drawn  parallel  to  ql 
t  irough  p  gives  the  horizontal  trace. 


Problem  XVII. 

Through  a  given  point  slr  to  draiv  a  straight  tine  which 
s,  mil  be  parallel  to  a  given  plane  qlm'  and  have  a  given  in- 
clination 6.     Fig.  20. 

Obs.  The  angle  6  cannot  exceed  the  angle  of  inclinar 
ti  m  of  the  given  plane. 

Through  the  given  point  ad  let  a  plane  hfiv'  be  drawn 
as  in  Problem  16,  parallel  to  the  given  plane.  And 
through  point  A  draw  the  straight  line  ar,  dr',  by  Problem 
8,  to  lie  in  the  plane  h^v\  and  have  an  inclination  of  0 
d(  grees. 

Another  mode  of  solving  the  problem  is,  to  determine 
in  the  given  plane  a  straight  line  having  the  required  incli- 
ne tion,  and  to  draw  through  the  given  point  A  a  straight 
lit  e  parallel  to  the  line  so  formed. 


70 


SOLID   OR  DESCRIPTIVE  GEOMETRY. 


Problem  XVIII. 

In  a  given  plane  bb'd'  to  determine  a  straight  line  which 
shall  be  perpendicular  to  a  given  straight  line  ba,  b'a'  lying 
in  that  plane. 

Fig.  11. 


STRAIGHT  LINE  AND  PLANE.  7 1 

'Rabat'  the  given  plane,  revolving  it  about  its  hori- 
zontal trace  bb'  into  the  horizontal  plane.  Then  AB  will 
lie  in  the  horizontal  plane  in  the  position  Ab.     Fig.  21. 

The  point  b  (or  B)  does  not  move  during  the  rotation 
Df  the  plane,  being  in  the  axis  or  trace  bb'  of  the  rotating 
plane.  The  perpendicular  to  bb'  through  point  a;  is  a  locus 
Df  A,  and  the  position  of  point  A  in  it  is  found  by  taking 
;he  length  b'd  along  the  vertical  trace  b'd'  of  the  plane 
ind  setting  it  off  from  the  line  bb'.  A  and  b  joined  gives 
he  line  Ab  or  AB  'rabatted.' 

Through  point  A  draw  the  indefinite  line  AD  perpen- 
dicular to  Ab^  and  determine  the  plan  d  of  any  point  D 
in  this  perpendicular.  To  do  this  : — Through  D  draw  a 
Jine  at  right  angles  to  bb'  for  one  locus  of  ^  (Theorem  XX.). 
vieasure  the  distance  of  point  D  from  the  line  bb'  and 
;  et  this  length  from  point  b',  along  the  vertical  trace  of  the 
])lane,  to  determine  d'.  A  perpendicular  from  d  to  xy  is 
;,  second  locus  of  d.  The  intersection  of  these  loci  of  d 
determine  that  point.  Join  a  and  d,  then  ad,  a'd'  are  the 
jirojections  of  the  required  perpendicular. 

Observe.  In  the  same  manner  a  straight  line  AD  may 
1  e  drawn  in  a  given  plane  to  make  any  given  angle  Q  with 
'<.  given  straight  line  AB  in  that  plane. 

N.B.  Theorems  VI.  and  VIII.  should  be  studied  in 
c  onnexion  with  this  problem  and  figure. 


72  SOLID  OR  DESCRIPTIVE   GEOMETRY. 


-Problem  XIX. 

To  determme  the  projections  of  a  given  plane  figure,  when 
the  inclination  of  its  plane  is  given  and  that  of  a  straight  line 
lying  in  the  plane  of  the  figure.     Fig.  21. 

I  St.  When  the  given  line  forms  part  of  the  perimeter 
of  the  figure. 

Let  the  given  figure  be  a  square,  its  plane  inclined 
0  degrees  and  one  side  AB  of  the  figure  incUned  ^  de- 
grees. 

Determine  the  plane  bb'd' :  in  it  place  the  line  ab, 
a'b'.  'Rabat'  the  plane  as  in  Problem  18,  and  on 
AB  describe  the  square  ABCD :  revolve  the  plane  back 
into  its  former  position  and  obtain  abed,  the  plan  of  the 
figure. 

2ndly.  When  the  given  straight  line  is  a  diagonal  of 
the  given  figure  or  its  bisecting  line  :  a  tangent,  a  chord,  or 
a  diameter  of  a  given  ellipse  or  circle. 

Determine  the  plane  bb'd'  of  the  figure  and  the  projec- 
tion of  the  given  line  in  it.  '  Rabat '  the  plane,  and  the 
line  as  lying  in  it ;  draw  the  given  figure  in  the  horizontal 
plane  having  the  required  position  with  regard  to  the  'ra- 
batted '  line :  and  from  thence  proceed  as  before  to  obtain 
the  projections  of  the  entire  figure  when  turned  back  into 
the  inclined  position. 


STRAIGHT  LINE  AND  PLANE. 


73 


Problem  XX. 

Given  two  straight  lines  AB,  AC  making  an  angle  BAG 
of  0  degrees  with  each  other;  to  determine  the  projections  of 
the  lines  when  AB  is  inclined  at  an  angle  of  a  degrees  and 
AC  inclined  /3. 

Fig.  22. 


3C. 


Observe  the  conditions  of  the  problem  are  impossible  if 
a+^  +  ^exceed  180° : 


74  SOLID   OR  DESCRIPTIVE   GEOMETRY. 

if  a  +  ^  +  e  equal  180" 
the  plans  of  AB  and  A  C  will  be  in  one  straight  line,  /.  e. 
the  plane  of  the  angle  BAC  will  be  a  perpendicular  one. 

The  case  in  which  a+^  +  d  is  less  than  180"  will  be 
that  here  considered. 

Draw  the  angle  BA  C  on  the  horizontal  plane.  It  will 
be  necessary  to  revolve  the  plane  of  the  angle  BAC  into 
the  inclined  position  it  will  occupy  when  its  containing 
lines  have  the  required  inclinations.  For  this  purpose  a 
horizontal  line  or  trace  BM  of  the  plane  must  be  disco- 
vered, about  which  the  plane  is  to  revolve.  Assume  a 
point  B  in  AB  and 

make  the  atigle  ABr  =  a"  and  , 
ArB  =  go\ 

Then  Br  will  be  the  length  of  the  plan  of  BA  when  BA 
is  in  the  required  position. 

Now  for  the  same  height  Ar{  =  A/),  by  the  aid  of  a 
tangent  fM  to  the  arc  rsf  determine 
the  angle  AMt=  ^  and 
...     ...     AtM=  90" :  and  point  M  is  found. 

The  straight  line  BM  is  the  required  horizontal  trace  of 
the  given  angle.  A  plane  of  elevation  having  its  xy  at  right 
angles  to  this  line  will  be  always  perpendicular  to  the 
plane  of  that  angle ;  and  the  elevation  of  the  angle  upon  it 
will  be  in  the  vertical  trace,  Theorem  VI.  2. 

The  elevation  of  the  path  of  point  A  is  the  circular 
arc  a"a',  in  which  a'  is  determined  by  the  parallel  to  xy 
at  the  distance  Ar. 

From  a  the  plan  a  is  found  in  AB  (Theorem  IV.). 


STRAIGHT  LINE  AND  PLANE.  75 

Then  BaM  is  the  required  plan,  and  8'a'  the  elevation 
and  vertical  trace  of  the  plane  of  the  given  angle.  «"SV  is 
the  angle  of  the  inclination  of  the  plane  of  the  lines. 

Cor.  To  determine  three  straight  lines  meeting  in  a  point, 
each  line  perpendicular  to  the  other  two. 

Proceed,  as  in  this  problem,  to  obtain  the  projections 
of  the  two  lines  AB  and  AC;  and  through  point  aa',  by 
Problem  13,  determine  the  projections  ae,  ae'  of  the  third 
line,  at  right  angles  to  the  plane  MM.  AB,  A  C,  AE  are 
the  required  lines. 

Problem  XXI. 

To  determine  the  projection  of  a  given  plane  figure  when 
the  heights  of  three  points  in  the  plane  of  the  figure  are  given. 

Limits.  When  the  given  heights  are  equal,  the  projec- 
tion of  the  figure  is  an  equal  and  similar  figure  to  the 
given  one. 

When  the  given  heights  are  unequal,  the  problem  is  a 
possible  one  only  when  the  difference  between  the  heights 
of  any  two  points  does  not  exceed  the  distance  between 
these  points. 

If  this  difference  is  equal  to  that  distance  the  plane  of 
:he  figure  is  vertical;  and  the  projection  required  will  be  a 
straight  line. 

Let  the  three  given  points  A,  B  and  C  be  at  the  angles 
)f  an  equilateral  triangle  of  m  inches  side,  and  the  heights 
)f  A,  B  and  C  be  /,  ^  and  r  inches  respectively  :  to  deter- 
nine  the  plan  abc  of  the  triangle.     Fig.  23. 

(i)     Draw  .^^C  in  the  horizontal  plane. 


7^  SOLID   OR  DESCRIPTIVE   GEOMETRY. 

Fig-  23. 


le^eZ 


^  ^^iU- 


w 


(2)  Determine  point  h  in  ^C  which  will  have  the  same 
height  as  B,  viz.  q  inches.  Then  hB  is  a  horizontal  line 
when  the  triangle  is  in  its  inclined  position. 

(3)  By  the  aid  of  an  elevation  at  right  angles  to  hB  as 
shown,  the  plan. a^^  is  determined. 


STRAIGHT  LINE  AND  PLANE.  .   77 

Problem  XXII. 

To  determine   two  parallel  planes  which  shall  contain 
the  given  straight  lines  AB  or  ab,  aV  and  CD  or  cd,  c'd'. 
ist.     If  the  given  lines  A£  and  CZ)  are  parallel. 

Determine  the  horizontal  traces  of  the  lines;  parallel 
lines  through  these  will  be  the  horizontal  traces  of  the 
parallel  planes  required,  whence  the  vertical  traces  can  be 
obtained. 

2ndly.  If  the  given  lines  AB  and  CD  be  neither 
parallel  nor  in  the  same  plane. 

Through  any  point  A  in  AB  draw  a  straight  line  AQ 
parallel  to  CD. 

Determine  the  horizontal  traces  of  the  lines  AB  and 
AQ;  the  straight  line  drawn  through  these  points  is  the 
horizontal  trace  of  one  of  the  required  planes. 

Determine  the  horizontal  trace  of  CD;  through  this 
ooint  a  parallel  line  to  the  trace  just  found  is  the  horizon- 
al  trace  of  the  second  plane  required :  and  the  vertical 
races  of  the  two  planes  can  be  obtained. 

3rdly.  If  the  given  lines  be  not  parallel  but  in  the 
;  ame  plane,  the  problem  is  impossible. 

Problem  XXIII. 

To  determine  a  straight  line  at  right  angles  to  each  of  two 
I  iven  straight  lines  AB  and  CD. 

I  St.     If  the  given  lines  be  parallel. 

Any  Straight  line  drawn  in  the  plane  of  the  parallels  per- 
1  endicular  to  one  of  the  lines  is  also  perpendicular  to  the 
c  ther. 


7^  SOLID   OR  DESCRIPTIVE   GEOMETRY. 

2ndly.  When  the  given  lines  are  neither  parallel  nor 
in  the  same  plane. 

From  any  point  P  in  AB  draw  a  line  PQ  parallel  to 
CD :  then  the  plane  of  AB  and  PQ  is  parallel  to  CD. 

From  any  point  C  in  CD  draw  CS  perpendicular  to 
the  plane  of  APQ,  meeting  it  in  .S*.  A  parallel  to  PQ  or 
CD  through  point  S  meets  AB  in  H.  Then  HT,  parallel 
to  CS  from  IT,  meets  CD  in  point  7^  and  is  the  required 
perpendicular  to  the  given  lines. 

Problem  XXIV. 

To  determine  the  angle  of  inclination  B  of  a  given  straight 
line  ab,  a'b'  to  a  given  plane  qlm'.     Fig.  24. 

In  this  problem  it  is  convenient  to  determine  the  com- 
plement of  the  angle  required. 

This  complementary  angle  is  the  angle  between  the 
given  straight  line  and  a  line  meeting  it  in  point  A  and  per- 
pendicular to  the  given  plane. 

Determine  this  perpendicular  ah,  a'h\  by  Problem  XIII., 
and  find  its  horizontal  trace  ha';  likewise  bb'  that  of  the 
given  line  ;  the  line  hb  through  these  points  is  the  horizon- 
tal trace  of  the  plane  of  the  lines.  'Rabat '  their  plane 
about  hb  bringing  A  into  the  horizontal  plane.  Ah,  Ab, 
when  drawn,  give  bAh  the  complementary  angle  or  90  —  6, 
and  6  is  the  angle  sought. 

Conversely : 

To  determine  a  straight  line  through  a  given  point  aa',  to 
make  a  given  angle  6  with  a  given  plane  qlm'. 

Through  ad  draw  ah,  a'h'  at  right  angles  to  the  given 
plane,  and  determine   its   horizontal   trace   hh'.     Through 


STRAIGHT  LINE  AND  PLANE. 
Fig.  24. 


79 


p  Dint  h  draw  in  any  direction  the  line  hh  as  the  horizontal 
t;  ace  of  the  plane  of  the  perpendicular  and  the  required 
li  le.  Fold  this  plane  about  hb  into  the  horizontal  one  and 
tlirough  point  A  thus  'rabatted'  let  Ab  making  an  angle 


So  SOLID  OR  DESCRIPTIVE  GEOMETRY. 

go-0  with  A/i  meet  the  trace  Ad  in  point  d.  Then  d'  being 
found  in  xy  and  joined  to  a,  the  required  line  ad,  a'b  is 
determined. 

Observe.  An  infinite  number  of  such  lines  ab,  a'b'  can  be 
determined  to  satisfy  the  given  conditions;  and  their  'locus' 
is  the  right-conical  surface  having  its  circular  base  in  the 
given  plane  and  its  generatrix  inclined  6  degrees  to  the 
same. 


Problem  XXV. 

To  determine  two  planes  which  shall  be  perpendicular  to 
each  other  and  inclined  at  angles  of  0  and  <f>  degrees  respectively 
to  the  horizontal  plane. 

It  will  be  obvious  that  the  angle  of  inclination  of  the 
second  plane  cannot  be  less  than  the  complement  of  that  of 
the  first ;  or  ^  not  less  than  90  -  ^.  - 

Therefore 

if  ^  is  greater  than  90  -  B,  there  are  two  solutions  : 
...<^  is  equal  to  90  -  Q,  there  is  one  solution ; 
...^  is  less  th^n  90  -  0,  there  is  no  solution. 
This  will  appear  by  what  follows. 

Draw  the  plane  qltrl  inclined  B  degrees;  also  the  straight 
]ine/w,/V;/  at  right  angles  to  it  (Problem  13),  and  find  the 
horizontal  trace  pp'  of  PM. 

Then  every  plane  which  contains  that  line  is  at  right 
angles  to  the  plane  B  (Eucl.  xi.  18).  Wherefore  if  such  a 
plane  revolve  about  PM,  as  an  axis,  until  it  makes  an  angle 
^  with  the  horizontal  plane,  it  is  the  plane  required. 


STRAIGHT  LINE  AND  PLANE. 
Fig.  25. 


81 


Let  it  therefore  be  made  to  touch  a  right  cone,  vertex 
ir  any  point  mm  in  PM  and  generatrix  m'^  making  an 
a:  igle  of  ^  degrees  with  its  base  gko.  The  horizontal  trace 
o  this  plane  must  touch  ^/^^  (Theorem  XL). 

E.  G.  6 


S2  SOLID   OR  DESCRIPTIVE   GEOMETRY. 

And  since  the  plane  is  to  contain  PM,  the  horizontal 
trace  rp  of  the  plane  must  pass  through  p  (Theorem  X.). 

Through  the  point  mm'  draw  the  horizontal  line  mn^ 
m'n  parallel  to  the  trace  rp^  and  determine  its  vertical 
trace  nn. 

Through  ii  and  s  draw  the  vertical  trace  of  the  plane. 

Observe  that  the  plane  rsn'  by  containing  PM  is  per- 
pendicular to  plane  B ;  and,  by  touching  the  cone  de- 
scribed, has  the  required  inclination  <^ ;  it  thus  satisfies  the 
conditions  of  the  problem. 

Note  also  that  the  inclination  of  PM  is  90  —  ^ ;  since  if 
a  straight  line  and  an  inclined  plane  be  perpendicular  to 
each  other  their  inclinations  are  complementary. 

Cor.  To  determine  three  planes  each  perpendicular  to  the 
other  two. 

For  the  first  and  second,  determine  planes  qhn  and  rsfi 
as  in  the  above  problem. 

Find  their  intersection  mq^  m'^  by  Problem  1 2. 

The  plane  which  is  perpendicular  to  MQ  is  perpendi- 
cular to  the  planes  which  meet  in  this  line  (Eucl.  xi.  19). 
Therefore  such  plane,  determined  by  Problem  13,  is  the 
third  plane  required. 

Problem  XXVI. 

To  determine  a  plane  inclined  at  an  angle  of  6  degrees  and 
making  a  given  angle  a  with  a  given  inclined  plane. 

Let  qlm'  be  the  given  inclined  plane. 

To  fulfil  both  conditions,  the  required  plane  must  touch 
two  right  cones  having  a  common  vertex  in  any  point  vtf 


STRAIGHT  LINE  AND  PLANE. 
Fig  25. 


83 


^i  \          /  1  \\ 

a,/ 

^       / 

7Z 

!     t/ 

>^tr        V 

f 

V 

\ 

\ 

V  ith  circular  bases  op  and  ^V  in  the  horizontal  and  inclined 

f  lanes  respectively* ;  the  generatrix  of  the  former  making  an 

*  b'  is  the  point  of  intersection  of  the  lines  z/V  and  lm\  and  is  not 
li  ttered  in  the  fig. :  also  the  plan  of  the  circular  base  b's'  is  not  shewn 
ii   the  fig. 

6—2 


§4  SOLID   OR  DESCRIPTIVE   GEOMETRY. 

angle  Q  with  its  base,  and  that  of  the  latter  an  angle  of 
a  degrees  with  the  inclined  plane. 

The  horizontal  trace  pi  of  the  common  tangent  plane  to 
the  cones  is  (Theorem  XI T.)  a  tangent  to  the  horizontal 
traces  of  the  two  conic  surfaces,  /.  e.  the  trace  //  touches  the 
circle  op  and  the  ellipse  rtu.  Since  four  such  lines  can  be 
drawn,  as  appears  by  the  figure,  it  is  obvious  that  in  the  case 
here  indicated  four  planes  can  be  determined  which  satisfy 
the  given  conditions  of  the  problem. 

Should  the  circle  op  fall  within  the  ellipse,  without 
touching  it,  the  given  conditions  are  impossible. 

Problem  XXVII. 

Given  three  lines  ag,  ak,  ar  meeting  in  a  point  a  ,•  as  the 
plans  of  the  straight  lines  AG,  AK,  AR,  each  of  which  is  at 
right  angles  to  the  plane  of  the  other  two ;  to  determine  an 
elevation  of  the  lines. 

Since  the  plane  of  lines  AK,  AR  is  perpendicular  to 
the  line  AG  (Eucl.  xi,  4),  the  horizontal  trace,  cd,  of  the 
plane  of  AK,  AR  is  perpendicular  to  ag,  the  plan  of  AG 
(Theorem  IX.). 

Therefore  draw  cd  at  right  angles  to  ga  produced,  and 
meeting  ah,  ar  in  points  c  and  d. 

ce  drawn  through  c  at  right  angles  to  ar,  and  de  drawn 
through  d  at  right  angles  to  ak,  will  meet  ag  in  e  and  be  the 
horizontal  traces  of  the  other  two  planes  (Theorem  IX.). 

Take  the  projecting  plane  of  the  plan  oi  AG  for  a  plane 
of  elevation,  and  let  it  revolve  about  the  line  ag  as  an  xy. 

This  plane  cuts  the  plane  of  AK  and  AR  in  AP  or  a'p, 
which  is  therefore  perpendicular  to  AG  or  a'g  in  the  plane 
of  elevation  (Eucl.  xi.  def.  3). 


STRAIGHT  LINE  AND  PLANE. 
Fig.  37. 


85 


^ —  r 


Therefore  on  ep  describe  the  semicircle  ec^p,  through  a 
d  aw  aa!  perpendicular  to  xy,  and  ea^  a'p  are  the  required 
elevations  of  the  three  lines  (Theorem  VI.  2). 

Problem  XXVIII. 

Given  two  points  B  and  C  at  unequal  distances  from  a  given 
pi  ine,  to  determine  a  point  Q  in  the  plane  at  which  the  straight 
Ih  es  QB  and  QC  shall  make  equal  angles  with  the  plane. 


86  SOLID  OR  DESCRIPTIVE  GEOMETRY. 

From  either  of  the  points,  as  B^  draw  the  straight  line 
BR  at  right  angles  to  the  given  plane  (Problem  13),  meeting 
it  in  S,  determined  by  Problem  14. 

Make  SR  -  SB,  point  R  being  on  the  opposite  side  of 
the  plane  to  B ;  join  R,  C,  cutting  the  given  plane  in  point 
Q  '•  Q.B,  QC  are  the  required  lines. 


Exercises  on  Chapter  III. 

1.  Determine  the  projections  of  four  points,  each  2 
inches  from  the  planes  of  projection,  viz.  one  in  each 
dihedral  angle. 

2.  Determine  four  points  each  in  a  plane  of  projection 
and  3  inches  from  xy ;  no  two  of  them  to  be  on  the  same 
side  of  that  line. 

Supposing  the  loci  or  projecting  lines  of  these  points 
(Theorem  IV.)  to  be  2  inches  apart,  determine  the  distance 
between  any  two  points. 

3.  A  point  is  2*5  inches  from  each  plane  of  projection, 
draw  two  lines  through  it  inclined  50°;  their  plans  making 
angles  of  35"  with  xy.     Problem  i,  converse  (2). 

Determine  the  traces  of  the  lines  :  also  the  angle  which 
the  lines  make  with  each  other.    Problem  10. 

4.  Through  a  point  2  inches  from  each  plane  of  pro- 
jection draw  lines  inclined  30",  making  angles  of  40°  with 
the  vertical  plane.     Problem  3  and  Theorem  IX. 


STRAIGHT  LINE  AND  PLANE.  8/ 

5.  A  straight  line  in  the  horizontal  plane  makes  an 
angle  of  45"  with  xy  and  is  a  trace  of  planes  which  are 
inclined  50";  find  their  vertical  traces.  Problem  3,  "tan- 
gent planes  to  cones,"  Chap.  viii. 

6.  A  regular  hexagon,  side  2  inches,  has  its  diameter 
and  adjacent  side  inclined  30"  and  45°  respectively :  draw 
its  plan  and  determine  the  incHnation  of  its  plane.  Pro- 
blem 20. 

7.  Draw  two  parallel  planes  2  inches  apart  and  inclined 
5o°.     Problems  i  and  13,  and  Problem  25,  note. 

8.  A  plane  is  inclined  40",  and  is  at  right  angles  to 
mother  which  is  inclined  70°:  draw  the  traces  of  the  planes. 
Problem  25. 

9.  The  horizontal  traces  of  two  planes,  inclined  45° 
md  30°  respectively,  make  an  angle  of  60":  determine  a 
hird  plane  at  right  angles  to  the  given  ones.     Problems  1 2 

and  13. 

10.  Two  planes  are  inclined  60"  and  70°  respectively 
;.nd  make  an  angle  of  50"  with  each  other:  determine 
1  heir  vertical  traces  on  a  plane  not  perpendicular  to  them, 
^'roblem  26.  •     . 

11.  A  regular  pentagon,  side  2*5  inches,  has  three  of 
i;s  angular  points  respectively  i,  2,  and  3  inches  high. 
Draw  its  plan.     Problem  21. 

12.  A  square,  side  3  inches,  lies  in  a  plane  inclined 
^5",  one  side  of  the  figure  makes  an  angle  of  40"  with 
t  le  horizontal  trace  of  its  plane.  Draw  its  plan.  Pro- 
l  lem  18. 


88  SOLID  OR  DESCRIPTIVE   GEOMETRY. 

13.  A  plane   is  inclined  80"  and   makes  an  angle   of 
60**  with  a  plane  which  is  inclined  50".     Draw  the  traces  of   ; 
the  planes.     Problem  26.  1 

14.  The  two  traces  of  a  plane  make  an  angle  of  50" 
with  each  other  and  make  equal  angles  with  xy :  determine 
these  latter  angles  and  the  inclination  of  the  plane. 

15.  Three  straight  lines  are  perpendicular  to  one 
another  and  two  of  them  are  inclined  25"  and  40**:  deter- 
mine the  inclination  of  the  third  line.    Problems  20  and  13. 

16.  The  horizontal  and  vertical  traces  of  a  plane 
make  angles  of  30"  and  55°  respectively  with  xy;  a  straight 
line  parallel  to  xy  is  2  inches  from  the  vertical  and  3  inches 
from  the  horizontal  plane  :  find  the  intersection  of  the  given 
line  and  plane.     Problem  14. 

17.  Two  planes  contain  a  right  angle;  one  of  them 
is  inclined  60"  and  their  intersection  is  inclined  50".  Draw 
the  traces  of  the  planes  and  find  the  inclination  of  the 
second.     Problems  8  and  13. 

18.  The  hypotenuse  of  a  right-angled  triangle  is  4 
inches  long,  and  horizontal;  the  plans  of  the  sides  make 
an  angle  of  115°  with  each  other:  determine  the  lengths 
and  inclinations  of  the  sides.  Can  more  than  one  set  of 
answers  be  given  ?     Eucl.  iii.  t,},,  and  Problem  20. 

19.  A  straight  line  inclined  40"  lies  in  a  plane  in- 
clined 60" ;  determine  a  plane  containing  the  given  line 
and  perpendicular  to  the  given  plane.     Problems  8  and  13. 

20.  Through  a  point  3  inches  high  draw  three  planes, 
inclinations  35°,  45",  and  60"  respectively;   forming  a  tri- 


STRAIGHT  LINE  AND  PLANE.  89 

hedral  angle  at  the  given  point.  Show  the  real  mag- 
nitude of  the  angle  of  each  face  of  the  solid  angle.  Pro- 
blems 12  and  10. 

21.  The  plans  of  two  lines  contain  an  angle  of  no". 
The  lines  themselves  are  at  right  angles,  and  one  is  in- 
clined 27".  Find  the  inclination  of  the  other.  Science 
Exam.  Hon.  1872. 

(i)  Draw  plans  of  the  lines  and  make  an  elevation  of 
3ne,  assumed  as  that  inclined  27°,  on  a  vertical  plane 
;aken  parallel  to  it. 

(2)  Determine  a  plane  perpendicular  to  this  line  andcon- 
;aining  the  other.  The  point  in  which  the  horizontal  trace  of 
his  plane  meets  the  plan  of  the  second  line  is  the  horizontal 
race  of  the  line  whose  angle  of  inclination  is  required. 

(3)  Determine  this  angle  by  Prob.  i. 

22.  Two  lines  are  inclined  at  30"  and  40".  They  are 
;  inches  apart  where  they  are  nearest  together,  and  this 
ine  of  2  inches  is  inclined  28".  Show  plan  and  elevation. 
Science  Exam.  Hon.  1873. 

(i)  Determine  a  line  inclined  28°,  and  mark  off  a 
•egment  of  this  line  2  inches  in  length.     Prob.  i. 

(2)  Draw  two  parallel  planes  at  right  angles  to  line  of 
;8"  passing  through  the  extremities  of  the  segment  of  2 
nches. 

(3)  From  the  point  where  line  of  28"  meets  one  plane 
flraw  a  line  lying  in  that  plane  and  inclined  at  40",  and 
irom  the  point  where  the  former  line  meets  the  parallel 
])lane   draw  another  line  lying  in  that  plane  and  inclined 


90  SOLID   OR  DESCRIPTIVE   GEOMETRY. 

The  two  lines  so  drawn  are  those  required. 

Note.  Four  pairs  of  Hnes  fulfilling  the  conditions  are 
possible. 

Another  solution  may  be  based  upon  Problems  20, 
r  (converse),  13  (converse)  and  8. 

23.  Given  a  plane  by  its  traces,  and  two  lines,  not 
passing  through  the  same  point,  by  their  projections.  The 
lines  to  be  neither  in  the  given  plane  nor  parallel  to  it. 
Find  the  projections  of  a  line  which  shall  meet  the  two 
given  lines,  be  parallel  to  the  given  plane,  and  at  a  given 
distance  from  it. 

Determine  a  plane  parallel  to  the  given  plane  at  a 
distance  from  it  equal  to  the  given  distance  of  the  parallel 
line  from  the  same  plane.     Problem  13,  Cor.  2. 

Find  the  points  of  intersection  of  the  two  given  lines 
with  this  plane.  Prob.  14.  The  line  joining  the  two  points 
thus  found  is  the  line  required. 

24.  Two  horizontal  lines  AB,  AC,  contain  an  angle 
of  56°,  a  plane  inclined  at  30"  contains  AB,  another  in- 
clined at  65"  contains  AC.  Draw  two  lines  passing  through 
A,  each  inclined  at  20"  and  lying  one  in  each  plane.  Deter- 
mine the  angle  between  these  two  lines.  Science  Exam. 
Hon.  1872.     Problems  4,  8  and  10. 

25.  A  hne  TV  3*3  inches  long  has  its  extremities  T 
and  Fin  the  horizontal  and  vertical  planes  respectively,  it 
is  inclined  to  the  horizontal  plane  at  40°,  and  has  its  trace 
T  2X  75  inch  from  the  ground  line.  Draw  its  plan  and 
elevation.     Science  JSxam.  1S6S. 


STRAIGHT  LINE  AND  PLANE.  9 1 

26.  An  indefinite  line  is  inclined  to  the  horizontal 
plane  at  40"  and  makes  an  angle  of  30'  with  the  vertical 
plane  of  projection,  the  distance  between  its  traces  being 
3  "5  inches.     Science  Exam.  Hon.  1869. 

27.  AB  is  a  line  parallel  to  the  horizontal  plane,  AC 
s  a  line  parallel  to  the  vertical  plane.  The  angles  bac  be- 
ween  the  plans  and  b'a'c'  between  the  elevations  are  120". 
Find  the  real  angle  between  the  lines.    Science  Exam.  187 1. 

28.  Determine  a  line  inclined  at  33"  lying  in  a  plane 
inclined  at  50".  This  is  the  orthographic  projection  on 
1  hat  plane  of  a  line  which  makes  an  angle  of  40"  with  it. 
Determine  the  projections  of  this  latter  line  on  the  co- 
ordinate planes  and  its  inclination.  Science  Exam.  Hon. 
:87i. 

29.  Find  the  traces  of  a  plane  which  contains  three 
assumed  points — one  in  each  plane  of  projection  and 
ttie  third  in  space. 

30.  Bisect  the  dihedral  angles  formed  by  the  three 
J  lanes  in  the  exercise  20  above. 

31.  Find  the  centre  of  a  sphere  of  given  radius  that 
s  lall  touch  three  assumed  planes. 


IV. 


SOLIDS  WITH  THE  INCLINATIONS  OF  THE 
PLANE  OF  ONE  FACE  AND  OF  ONE  EDGE 
OR   LINE   IN   THAT  FACE  GIVEN. 

Before  commencing  this  chapter  the  student  should 
refer  to  Chapter  iii.  Problems  i8  and  19. 

I.  Plan  and  elevation  of  a  pyramid,  4  inches  high,  with 
square  base  0/2'$  inches  side,  when  its  base  is  inclined  e^o'^, 
and  no  side  horizontal. 

(i)  Determine  by  its  traces  a  plane  50"  at  right  angles 
to  the  vertical  plane ;  /.  e.  draw  its  horizontal  trace  at  right 
angles  to,  and  its  vertical  trace  at  50"  with,  the  ground  line. 

(2)  'Rabat,'  that  is,  fold  this  plane  on  its  horizon- 
tal trace  into  the  horizontal  plane,  and  draw  the  square  as 
then  in  the  horizontal  plane;  i.e.  simply  draw  the  square, 
taking  care  that  no  side  is  parallel  to  xy. 

(3)  Lift  the  plane  back  to  the  required  position  and 
determine  required  plan  and  elevation  of  the  square  base. 
Remember  that  this  plane  being  at  right  angles  to  the  verti- 
cal plane  will  have  all  points  in  it  shown  in  this  elevation  /// 
the  vertical  trace  (Theorem  VI.  2).  The  square  base  will 
therefore  have  its  elevation  wholly  in  the  vertical  trace. 


SOLIDS   WITH  ONE  FACE   GIVEN.  93 

To  draw  this  elevation,  take  the  distances  from  the 
:orners  of  the  square  to  the  horizontal  trace  when  the  plane 
s  folded  down,  and  set  them  along  the  vertical  trace  from 
ts  junction  with  xy.  This  will  give  these  points  in  eleva- 
'ion.  Their  plans  will  lie  in  the  perpendiculars  to  xy  from 
\\&\r  elevations  (Theorem  IV.)  and  also  in  the  perpendiculars 

0  the    horizontal   trace  from   the   points    as    'rabatted' 
Theorem  XX.).    Thus  let  the  point  of  which  a'  is  the  eleva- 

lion  when  folded  down  or  'rabatted'  be  marked  A.  A 
]  )erpendicular  from  a  to  xy  contains  the  plan  a,  as  also 
does  the  perpendicular  from  A  to  the  horizontal  trace. 
The  plan  a  therefore  lies  in  the  intersection  of  these  per- 
]  )endiculars ;  in  other  words,  these  perpendiculars  are  loci 
( if  the  point  a,  and  their  intersection  gives  the  point  itself. 

(4)  The  axis  will  be  drawn  from  the  centre  of  the  base 
ii  plan  and  elevation  at  right  angles  to  the  traces,  and 
being  parallel  to  the  vertical  plane  will  be  set  in  true  length 
tn  elevation.  It  only  remains  to  join  the  proper  points,  and 
tie  projections  of  the  solid  are  completed. 

2.  Flan  and  elevation  of  a  cube  of  i-e^  inches  edge,  when 
one  face  is  inclined  60°,  and  no  side  horizontal. 

3.  Flan  and  elevation  of  a  pyramid,  4  inches  high,  with 
t'-gular  hexagonal  base  of  i'^  in.  side  inclined  70",  7io  side 
h  orizontal,  and  its  lowest  corner  i  inch  above  the  ground. 

4.  Flan  and  elevation  of  the  pyrajnid  in  the  last  problem 

1  'hen  the  base  is  inclined  70"  and  one  side  of  the  base  45". 

In  this  problem  there  is  given  not  only  the  inclina- 
t  on  of  the  plane  of  the  base,  but  also  of  one  edge  in  that 
tase. 


94  SOLID   OR  DESCRIPTIVE   GEOMETRY. 

Limits.  The  problem  is  therefore  to  draw  a  line  of 
given  inclination  in  a  plane  of  given  inclination,  and  this  is 
always  possible  if  the  inclination  of  the  line  is  not  greater  than 
that  of  the  plane. 

(i)  Determine  by  its  traces  the  required  plane  in- 
clined 70"  and  at  right  angles  to  the  vertical  plane.  Assume 
any  point  A  in  the  plane;  its  elevation  a'  is  by  the  position 
of  the  plane  in  the  vertical  trace;  its  plan  «  on  a  perpendi- 
cular to  xy  from  a',  and  for  convenience  it,  i.e.  the  plan  a, 
may  be  assumed  at  the  point  where  this  perpendicular 
meets  xy;  and  from  a'  draw  am  to  meet  the  ground  line 
at  the  given  angle  45°.  Rotate  the  triangle  thus  formed 
on  the  vertical  line  a'a,  the  projector  of  the  point  A,  until 
the  point  m  meets  the  horizontal  trace.  As  A  and  fn  are 
now  both  in  the  given  plane,  the  line  joining  a  and  m 
will  be  the  plan  of  Am  and  will  fulfil  the  conditions  by 
being  the  plan  of  a  line  wholly  in  plane  70°  and  itself 
incHned  45". 

(2)  *  Rabat'  plane  70°  on  its  horizontal  trace  into 
the  horizontal  plane.  On  the  line  Am,  thus  brought  into 
the  horizontal  plane,  describe  the  required  hexagon,  and 
complete  plan  and  elevation  of  the  whole  as  in  former 
problems. 

5.  Plan  and  elevation  of  a  cube  of  2'^  inches  edge  when 
the  plane  of  one  face  is  inclitied  65"  and  one  diagonal  of  that 
face  25*'. 

6.  Plan  and  elevation  of  a  tetrahedron  of  3  inches  edge 
with  one  face  70"  and  one  edge  in  that  face  40°.  Determine 
also  the  inclinations  of  the  other  edges  of  the  solid. 

7.  Plan  and  elevation  of  hexagonal  right  prism  when  the 
plane  of  one  of  its  faces  ABCD  is  inclined  50°  and  the  edge 
AB  of  the  base  inclined  35". 


SOLIDS    WITH  ONE  FACE    GIVEN.  95 

(i)  Determine  the  plan  of  rectangular  face  ABCD  as 
in  preceding  problems. 

(2)  It  will  next  be  necessary  to  determine  the  plane  of 
the  base,  which,  by  the  definition  of  the  solid,  is  perpendi- 
cular to  the  plane  of  the  face.  The  line  AB  already  deter- 
mined is  in  the  base,  being  common  to  both  planes,  and  its 
horizontal  trace  will  therefore  be  a  point  in  the  horizontal 
trace  of  the  required  plane  of  base.  And  as  the  projections 
of  a  perpendicular  to  a  plane  are  perpendicular  to  its  traces, 
the  horizontal  trace  of  the  plane  of  the  base  may  be  at 
once  drawn  through  this  point  at  right  angles  to  the  long 
edges  of  the  prism,  that  is,  in  this  case  to  be  or  ad. 

(3)  Take  a  ground  line  at  right  angles  to  the  horizon- 
tal trace  of  plane  of  base,  and  set  up  the  height  of  ^  or  ^ 
(both  known  points)  in  the  new  elevation.  By  the  position 
Df  the  new  plane  of  elevation,  the  vertical  trace  will  contain 
:he  elevations  of  both  A  and  B^  and  therefore  can  at  once 
DC  drawn. 

Fold  the  plane  of  the  base  about  its  horizontal  trace  to 
jring  AB  down  into  the  horizontal  plane.  On  the  AB 
has  rabatted  describe  hexagon  and  complete  as  in  former 
)roblems. 

8.  The  plane  containing  one  edge  of  a  tetrahedron  and 
(isecting  aiiother  is  inclined  at  50",  and  the  former  edge  is 
inclined  at  30".  Draw  plati  and  elevation  of  the  solid,  the 
idge  beingT,  inches.     Science  Exam.  Hon.  1870. 

(i)  Determine  plane  50°  containing  line  inclined  30", 
;  nd  '  rabat '  it. 

(2)  Measure  off  on  the  rabatted  line  3  inches,  and  on 
1  lis  segment  as  a  base  draw  an  isosceles  triangle  having  its 


96  SOLID   OR  DESCRIPTIVE   GEOMETRY. 

equal  sides  equal  to  the  altitude  of  an  equilateral  triangle  of  3 
inches  side.  The  figure  thus  drawn  is  the  true  form  of  the 
section  of  the  tetrahedron  made  by  the  given  plane. 

(3)  Rerabat  plane,  and  from  vertex  of  triangle  draw 
a  perpendicular  to  the  plane  and  produce  it  on  both 
sides  to  equal  distances  of  1*5  in.  On  joining  the 
extremities  of  this  perpendicular  with  the  extremities  of  the 
line  of  3  inches  measured  off  on  line  of  30°,  we  obtain  the 
projections  of  tetrahedron  required.  ^ 

9.  Plan  and  elevation  of  octahedron  3  in.  edge. 

a.  Plane  of  one  face  inclined  at  65"  and  one  edge  of  that 
face  at  30°. 

/8.  Plane  containing  two  axes  inclined  at  70"  and  one  edge 
of  the  solid  in  that  plane  at  2^^^. 

y.  When  two  of  its  axes  are  inclined  at  60°  and  20" 
respectively. 

10.  Pentagonal  pyramid,  side  of  base  i"5  in.,  axis  3*5  in., 
7vhen  the  plane  containing  the  axis  and  one  of  the  slant  edges 
is  inclined  at  50"  and  the  axis  at  2^. 

1 1.  Octagonal  right  prism,  side  of  base  75  in.,  axis  3  in., 
when  the  latter  is  inclined  at  30"  and  one  side  of  the  base  at  2  7". 


SOLIDS    WITH    THE    INCLINATIONS    OF    TWO 
ADJACENT  EDGES  OR  LINES  GIVEN. 

Limits.  It  must  be  remembered  that  the  sum  of  these 
inclinations  with  the  angle  contained  by  the  t7vo  edges  must  not 
exceed  i8o°. 

For  these  problems  refer  to  Chapter  in.,  Problem  20. 

I.  Plan  and  elevation  of  a  cube  when  the  inclinations  of 
two  adjacent  edges  as  BA,  BC,  20"  and  48",  are  given. 

(i)  Draw  the  square  A  BCD  to  represent  the  face 
rabatted  into  the  horizontal  plane.  Then  to  find  a 
horizontal  in  the  plane  of  this  face : — At  any  point  E  in 
BA,  or  BA  produced,  make  the  angle  BEF  20".  Then 
if  a  perpendicular  be  drawn  from  B  to  EF,  it  will  be  the 
length  of  the  line  expressing  the  height  to  which  B,  when 
in  required  position,  will  be  raised  above  horizontal  plane 
:ontaining  point  E.  From  ^  as  a  centre  with  this  perpendi- 
;ular  for  radius,  describe  a  circle.  The  tangent  to  this 
:ircle  to  make  an  angle  of  48°  {i.e.  the  inclination  of  the 
)ther  edge)  with  BC  will  give  a  point  H  where  it  meets 
BC  in  the  horizontal  required.  E  being  another  point  in  it, 
he  straight  line  through  H  and  E  will  be  the  horizontal 
lought  in  plane  of  face  ABCD. 

(2)    Assume  a  plane  of  elevation  at  right  angles  to  this 
E.  G.  -         7 


98  SOLID   OR  DESCRIPTIVE   GEOMETRY. 

inclined  plane  of  face  ABCD.  The  height  to  which  point 
B  is  to  be  raised  being  known,  a  line  of  level  that  distance 
above  the  xy  taken  will  contain  the  elevation  of  B.  The 
rest  of  the  work  is  similar  to  that  in  preceding  problems. 

Note.  To  understand  the  principles  of  this  construction, 
the  student  had  better  cut  a  card-board  model,  by  which  to 
illustrate  the  folding  up  and  down  of  the  plane  of  the 
angles  and  of  the  lines. 

2.  Plan  and  elevation  of  a  tetrahedron  when  two  of  the 
adjacent  edges  are  inclined  30"  and  45®. 

3.  Flan  and  elevation  of  hexagonal  right  prism  when  the 
two  edges  AB,  AD,  in  one  face  are  inclined  respectively  45° 
and  2  7",  AB  being  the  edge  of  the  base. 

Determine  the  plan  of  the  rectangular  face  ABCD  as  in 
Problem  i,  and  refer  to  explanations  of  Problem  7,  Chapter 
IV.,  to  find  the  plane  of  base  and  to  complete  the  projections 
of  the  solid. 

4.  Octagonal  prism,  3  in.  long,  side  of  base  i  in.,  when 
its  axis  is  inclined  at  40"  and  an  edge  of  the  base  «/  27". 

5.  Cube,  2*5  in.  edge,  when  two  diagonals  of  the  solid  are 
inclined  at  40"  and  35"  respectively. 

6.  Octahedron  3  in.  edge.  \ 
a.      When  two  adjacent  edges  in  one  face  are  inclined  at 

32"  and  40°. 

/8.  When  two  adjacent  edges  not  in  one  face  are  at  20° 
and  48". 

7.  A  regular  pentagonal  pyramid,  2  -5  in.  side,  axis  3  '5 
in.,  has  three  of  the  angular  points  of  its  base  taken  in  order, 
r  /;/.,  2  in.,  and  3  /;/.  high  respectively.     Plan  and  elevation. 


VI. 


SOLIDS  WITH  THE  INCLINATIONS  OF  TWO 
ADJACENT  FACES  GIVEN. 

For  these  problems  refer  to  Chapter  in.,  Problem  25. 

Limits.  Case  i.  If  the  given  planes  are  at  right 
angles  to  each  other,  their  inclination  being  6"  and  ^'*, 
6"  +  <f>^  must  not  be  less  than  90". 

Case  2.  If  the  given  planes  are  not  at  right  angles 
to  each  other,  refer  to  Chapter  iii..  Problem  26. 

I.  Plan  and  elevation  of  a  cube  of  3  inches  edge,  when 
the  inclinations,  45°  and  70°,  of  the  planes  of  two  adjacent 
faces  are  given. 

(i)  Determine  the  traces  of  plane  45°  assumed  at  right 
angles  to  the  vertical  plane. 

(2)  Determine  the  projections  of  a  perpendicular  to 
plane  45",  in  order  to  determine  plane  70",  since  every 
plane  containing  this  perpendicular  is  at  right  angles  to 
the  former  plane. 

'  (3)  Determine  horizontal  trace  of  plane  70".  To  do 
this,  make  any  point  in  the  perpendicular — e.g.  that  in  which 
it  pierces  the  plane  of  45° — the  vertex  of  a  right  cone,  genera- 
trix inclined  70°,  base  in  horizontal  plane.      Then,  since 

7—2 


100  SOLID  OR  DESCRIPTIVE   GEOMETRY. 

every  plane  touching  this  cone  is  inclined  70",  a  tangent  to 
the  circular  base  of  the  cone  will  be  the  horizontal  trace  of 
such  a  plane,  and  if  it  be  drawn  as  well  through  the  horizon- 
tal trace  of  the  perpendicular  to  plane  45**,  it  will  be  the 
horizontal  trace  required,  i.e.  of  a  plane  perpendicular  to 
plane  45**  and  inclined  itself  70°. 

(4)  Determine  the  plan  of  the  intersection  of  the  two 
planes.  One  point  in  this  plan  will  be  at  the  intersection 
of  the  horizontal  traces  of  the  two  planes,  and  a  second 
point  will  be  the  plan  of  the  point  in  which  the  perpendicular 
pierces  the  plane.  The  line  drawn  through  these  two  points 
will  be  the  plan  of  the  intersection  required.  Rabat  one  of 
the  planes  (say  that  at  45")  containing  this  line  into  the  hori- 
zontal plane,  draw  the  square  face  of  the  cube  on  the  line  so 
rabatted,  and  determine  plan  of  the  face  therefrom.  Complete 
plan  and  elevation  of  the  cube  as  in  former  problems. 

2.  A  prism  3  inches  long  has  an  equilateral  triangle  of 
2  "3  inches  side  for  its  base  ;  draw  its  plan  and  an  elevation  of  it 
7vhen  the  planes  of  its  base  and  one  face  are  equally  inclined  at 
60"  to  the  horizontal  plane.     Science  Exam.  Hon.  1869. 

3.  Plan  and  elevation  of  a  prism  with  base  a  regular 
hexagon.     Plane  of  one  face  inclined  48".     Base  68". 

(i)  Determine  the  plan  of  the  rectangular  face  of 
prism  as  for  the  face  of  the  cube  first  determined  in 
problem  i. 

(2)  On  a  vertical  plane  at  right  angles  to  plane  68", 
i.e.  with  its  ground  line  taken  at  right  angles  to  the 
horizontal  trace  of  plane  68",  determine  the  vertical  trace 
of  plane  68",  and  rabat  this  plane  into  the  horizontal 
plane.  On  AB,  the  edge  common  to  plane  of  face  and 
plane  of  base   so  rabatted,  describe  the  regular  hexagon. 


SOLIDS  WITH  TWO  ADJACENT  FACES   GIVEN.    10 1 

Determine  plan  therefrom  and  complete  problem  as  in  former 
cases. 

4.  Plan  and  elevation  of  a  tetrahedron.  Planes  of  adja- 
cent faces  6  and  <^. 

For  example,  let  6 - 45''  and  ^  =  75". 
(i)     Determine  the  angle  a  between  the  two  adjacent 
faces  of  the  solid. 

(2)  Draw  the  traces  of  plane  6,  taking  the  plane  at 
right  angles  to  the  vertical  plane  of  elevation. 

(3)  Determine  a  second  plane  inclined  «^  and  making 
an  angle  of  a  with  plane  6,  Chapter  in.  Problem  26.  The 
intersection  of  these  two  planes  B  and  ^  is  the  indefinite 
edge  of  the  solid.  Rabat  plane  0  or  ^,  and  on  the 
intersection  so  rabatted  mark  off  the  length  of  the  given 
edge  of  the  tetrahedron,  describe  the  face  upon  it,  and 
proceed  as  in  former  problems. 

5.  Plan  and  elevation  of  a  tetrahedron  with  one  face 
vertical  and  an  adjacent  face  inclined  50". 

Determine  the  angle  Q  between  adjacent  faces.  • 

Determine  a  plane  inclined  50°  to  the  horizontal  plane 
and  0  to  the  vertical  plane  by  Chapter  in.  Problem  7. 

On  the  vertical  trace  place  c^b'  the  real  length  of  an  edge 
of  the  tetrahedron,  and  on  it  describe  the  equilateral  triangle 
db'd.  Find  the  centre  v'.  a'b'c'v'  is  the  required  eleva- 
tion. 

In  plan  the  vertical  face  c^b'c'  will  be  a  straight  line  abc 
parallel  to  xy,  and  the  plan  v  of  the  vertex  will  be  a  point 
in  the  projector  drawn  from  v'  perpendicular  to  xy  at  a 
distance  in  front  of  abc  equal  in  length  to  the  axis  of  the 
tetrahedron.  Lines  joining  v  with  the  points  abc  of  the 
vertical  face  complete  the  plan  of  the  solid. 


102  SOLID   OR  DESCRIPTIVE   GEOMETRY. 

Note.  This  problem  should  also  be  worked  by  the 
general  method  as  given  in  Problem  4. 

6.  Substitute  the  octahedron  for  the  tetrahedron  in  Probs. 
4  and  5. 

7.  Pentagonal  pyramid  in  Prob.  10,  Chap.  iv. 

a.  When  the  planes  of  two  adjacent  faces  are  inclined  at 
6°  and  f^\ 

/8.  When  the  plane  of  the  base  is  inclined  at  6°  and  of  one 
face  at  ^. 

8.  Plan  and  elevation  of  right  prism,  regular  hexagonal 
base,  when  the  plane  of  ofie  face  is  inclined  at  50"  and  its  axis 
a/  35".     To  be  worked  by  the  method  given  for  this  Chapter. 


VII. 


PROPERTIES  OF  CURVED  SURFACES  AND 
PROBLEMS  BASED  THEREON. 

The  investigation  of  the  properties  of  curved  surfaces 
can  be  greatly  simplified  by  making  certain  assumptions  as 
to  the  mode  in  which  these  surfaces  have  been  generated  by 
the  motion  of  a  line,  straight  or  curved,  of  constant  or 
variable  magnitude,  according  to  a  definite  law. 

The  line  whose  motion  generates  the  surface  is  called 
l;he  generatrix.  Any  lines  which  limit  or  direct  the  motion 
jf  the  generatrix  are  called  directrices. 

All  surfaces  having  straight  lines  for  their  generatrices 
ire  called  Ruled  Surfaces. 

■Ruled  Surfaces  are  divisible  into  two  classes;  Develop- 
ible  Surfaces  and  Twisted  or  Skew  Surfaces. 

Any  two  consecutive  generatrices  of  a  Developable  Surface 
ie  in  one  plane.  This  is  not  true  of  Twisted  or  Skew 
Surfaces.  The  former  can  therefore  be  laid  flat,  or  de- 
veloped, without  extension  or  rupture  of  any  part,  by  turning 
iach  element  of  the  surface  successively,  about  the  consecu- 
ive  generatrices,  into  the  plane  of  development.  The  latter 
ire  not  developable. 


104  SOLID   OR  DESCRIPTIVE   GEOMETRY. 

The  simplest  of  all  Ruled  Surfaces  is  the  plane,  which  is 
generated  by  a  straight  line  moving  parallel  to  itself  along 
another  straight  line  fixed  in  position. 

To  understand  clearly  what  is  meant  by  a  Developable 
Surface,  observe  that  since  the  consecutive  generatrices  lie 
in  one  plane  they  must  either  meet  or  be  parallel.  Let  A, 
B,  C,  D,  &c.  be  a  series  of  consecutive  generatrices  of  a 
Ruled  Surface,  and  let  B  meet  A  in  the  point  i,  C  meet  B 
in  2,  D  meet  C  in  3  and  so  on.  The  limit  of  the  points  i, 
2,  3,...??,  will  be  a  curve,  generally  of  double  curvature, 
called  the  Edge  of  Regression,  to  which  every  generatrix  of 
the  surface  is  a  tangent.  The  parts  of  the  surface  generated 
by  the  tangent  line  to  the  edge  of  regression  on  each  side  of 
the  point  of  contact,  form  what  are  called  the  two  sheets  of 
the  surface. 

In  the  cone  the  points  i,  2,  3,  &c.  coincide,  and  the 
edge  of  regression  becomes,  therefore,  a  point — i.e.  the  vertex 
of  the  cone. 

In  the  cylinder,  the  generatrices  being  parallel,  the  edge 
of  regression  is  at  an  infinite  distance. 

From  the  mode  of  generation,  it  is  evident  that  through 
every  point  on  a  Ruled  Surface  one  straight  line,  at  least, 
can  be  drawn  coinciding  throughout  its  whole  length  with 
the  surface. 

Some  surfaces  may  be  conceived  as  generated  in  a 
variety  of  ways,  but  there  is  one  class  which  it  is  most 
convenient  to  regard  as  generated  by  the  motion  of  a  line 
about  a  fixed  axis  in  such  a  manner  that  every  point  in  the 
generating  line  is  always  at  the  same  distance  from  two 
fixed  points  in  the  axis.  These  are  called  Surfaces  of 
Revolution:     The  generating  line  may  be  straight,  or  of 


CURVED  SURFACES— THE  SPHERE.  10$ 

single  or  double  curvature.  The  treatment  of  these  surfaces 
is  generally  facilitated  by  assuming  the  generatrix  to  be  the 
line  in  which  a  plane  containing  the  axis  would  cut  the 
surface.     Such  a  section  is  called  a  meridian  section. 

The  limits  of  this  book  do  not  admit  of  an  exhaustive 
treatment  of  the  properties  of  curved  surfaces,  but  the 
student  who  wishes  to  pursue  the  subject,  by  the  aid  of 
Descriptive  Geometry,  into  its  higher  developments,  is 
strongly  advised  to  do  so  on  the  lines  indicated  in  the 
foregoing  general  remarks. 

PROBLEMS  ON  THE  PROJECTION  OF  CURVED 
SURFACES. 

The  construction  of  special  generatrices  of  curved 
surfaces  is  always  a  problem  of  importance,  and,  generally, 
the  projection  of  a  curved  surface  is  determined  by  the 
projection  of  a  sufficient  number  of  its  generatrices. 

The  Sphere. 
Def.     a  sphere  is  the  surface  generated  by  the  revolu- 
tion of  a  semicircle  about  its  diameter,  which  remains  fixed 
in  position  during  the  motion. 

A  sphere  may,  therefore,  be  regarded  as  the  locus  of  all 
points  in  space  which  are  equidistant  from  a  fixed  point. 

Theorem  I.  Every  orthographic  projection  of  a  sphere 
is  a  circle  equal  in  radius  to  the  radius  of  the  sphere. 

Theorem  II.  Every  plane  section  of  a  sphere  is  a 
circle.  That  cut  by  a  plane  passing  through  the  centre  is  a 
circle  equal  in  radius  to  the  radius  of  the  sphere,  hence  called 
a  ^^great  circle.'"     Other  plane  sections  are  small  circles. 

I .    To  determine  the  centre  and  radius  of  a  sphere  which  shall 


I06  SOLID   OR  DESCRIPTIVE  GEOMETRY. 

pass  through  four  given  points,  A,  B,  C,  D,  not  in  the  same 
plane,  and  no  three  whatever  of  which  are  in  the  satne  straight 
lifie. 

Theorem.     One  sphere  and  one  only  is  possible. 

First  method.  If  the  line  joining  any  two  of  the 
points  be  bisected  by  a  plane  perpendicular  to  it,  since  every 
point  in  that  plane  is  equidistant  from  the  extremities  of  the 
line  which  it  bisects,  the  plane  must  contain  the  centre 
of  the  sphere  (Theorem  XVIII.  page  ii).  For  the  same 
reason  if  two  other  planes  be  assumed  similarly  bisecting 
any  other  two  of  the  lines  joining  the  given  points,  three 
planes  will  have  been  determined,  each  containing  the 
required  centre.  This  will  therefore  be  the  point  which  is 
the  common  intersection  of  the  three  planes. 

Second  method.  Determine  the  centre  F  of  the  circle 
which  passes  through  any  three  of  the  points,  as  A,  B,  C, 
and  through  the  point  F  draw  a  perpendicular  to  the  plane 
of  these  three  points.  This  perpendicular  will  contain  the 
centre  of  the  sphere.  Similarly,  determine  the  centre  Q  of 
the  circle  which  shall  pass  through  the  fourth  point  F)  and 
two  of  the  others,  as  A,  B.  The  perpendicular  through  Q 
to  the  plane  of  A,  B,  F>,  will  also  contain  the  centre  of  the 
sphere.  The  point  O  in  which  the  perpendiculars  meet  will 
therefore  be  the  centre  of  the  sphere  required. 

The  radius  of  the  sphere  is  the  true  length  of  a  line 
from  O  to  any  one  of  the  given  points.     Prob.  i.  Chap.  iii. 

Note.  To  simpHfy  the  problem,  three  of  the  points  may 
be  taken  in  one  of  the  planes  of  projection. 

2.  Three  spheres,  A,  B,  and  C,  of  i'6  in.,  '8  in.,  and  -4 
in.  radii  respectively,  lie  on  the  horizontal  plane,  each  touching 
the  other  two.    Plan  and  elevation. 


CURVED  SURFACES— THE  SPHERE.  10/ 

Begin  with  an  elevation  of  the  spheres  A  and  ^  on  a 
vertical  plane  assumed  parallel  to  a  vertical  plane  containing 
their  centres.  In  this  position  the  elevations  will  be  two 
circles  resting  upon  xy  and  touching  each  other.  Draw  two 
similar  elevations  of  C,  one  touching  A  and  the  other 
touching  B.  Take  the  horizontal  distances  between  the 
elevations  of  the  centres  of  A  and  C,  and  of  B  and  C,  and 
from  the  plans  of  the  centres  of  A  and  B  with  these 
distances  for  radii  respectively,  describe  arcs.  The  inter- 
section of  these  arcs  will  determine  the  plan  of  the  centre  of  C, 
the  third  sphere.    Its  elevation  can  then  easily  be  determined. 

3.  To  determine  the  projections,  pp',  of  a  point,  P,  on  the 
surface  of  a  given  sphere. 

Let  00'  be  the  projections  of  the  centre  O  of  the  sphere, 
and  /  the  plan  of  the  given  point.  The  plan  /  must  of 
course  be  assumed  somewhere  within  the  circle  which  is  the 
plan  of  the  sphere.  The  elevation  /'  of  P  will  be  in  the 
projecting  line  drawn  from  /  at  right  angles  to  xy,  and 
its  height  may  be  found  by  taking  an  auxiliary  vertical 
section  plane  through  the  point  P  and  the  centre  O,  and 
rabatting  it  into  one  of  the  planes  of  projection. 

4.  A  sphere  of  i  in.  radius  revolves  uniformly  about  its 
fixed  vertical  diameter.  A  tracing  point  is  carried  wiiformly 
over  its  surface  in  a  fixed  vertical  plane  from  the  highest  to  the 
^owest  point  of  the  sphere,  during  the  time  in  which  the  latter 
makes  one  complete  revolution.  Show  the  projections  of  the 
■urve.     Centre  of  sphe^-e  2  in.  high. 

(i)  Beginning  at  the  highest  point,  divide  one  half  of  a 
/ertical  great  circle  of  the  sphere  into  any  number  of  equal 
)arts.     Through   these  points   take  a  series   of  auxiliary 


I08  SOLID   OR  DESCRIPTIVE  GEOMETRY. 

horizontal  planes  cutting  the  sphere.     The  sections  in  plan 
will  be  circles,  and  in  elevation,  straight  lines  parallel  to  xy. 

(2)  Divide  the  complete  circle,  that  is  the  plan  of  the 
sphere,  into  the  same  number  of  equal  parts  as  the  half  of 
the  great  circle  above  is  divided  into,  and  draw  the  radii  of 
the  circle  from  each  of  these  points.  Starting  from  the 
highest  point  of  the  surface,  construct  the  plan  of  the  curve 
through  the  points  where  the  first  circle  is  cut  by  a  radius, 
the  second  circle  by  the  next  radius,  and  so  on  to  the 
circumference  of  the  plan,  and  then  back  again  to  the  lowest 
point. 

(3)  For  the  elevation,  project  the  points  through  which 
the  plan  of  the  curve  is  drawn  to  the  elevations  of  the 
horizontal  sections  in  which  they  lie. 

The  case  should  also  be  worked  in  which  the  velocity  of 
the  revolving  sphere  is  doubled  and  the  vertical  velocity  of 
the  tracing  point  constant. 

5.  The  plane  of  a  circle  2*5  in.  diameter  is  inclined 
at  25".  Its  centre  is  2*8  in.  above  the  horizontal  platie. 
Determine  the  sphere  which  resting  on  the  horizontal  plane 
has  this  circle  on  its  surface.  Science  Exam.  Hon.  1873. 

(i)  Determine  plane  of  25°  and  the  projections  of  the 
circle  lying  therein. 

■  A  straight  line  drawn  from  the  centre  of  the  given  circle 
at  right  angles  to  its  plane  will  pass  through  the  centre  of 
the  required  sphere. 

(2)  Draw  projections  of  a  line  at  right  angles  to  given 
plane  from  centre  of  given  circle  and  find  the  horizontal 
trace  of  this  line. 

(3)  Draw  a  vertical  plane  containing  the  line  deter- 
mined in  (2).     This  plane  will  be  perpendicular  to  plane  25" 


CURVED  SURFACES— THE  SPHERE.  I09 

and  its  horizontal  trace  will  pass  through  the  horizontal  trace 
of  the  perpendicular  to  this  plane  (2)  and  will  also  be  at 
right  angles  to  the  horizontal  trace  of  the  given  plane. 

The  vertical  plane  will  cut  the  given  circle  at  two  points 
which  are  the  extremities  of  a  diameter,  shown  in  plan  as  the 
minor  axis  of  the  ellipse  which  is  the  plan  of  the  circle. 
These  points  are,  moreover,  two  points  in  a  great  circle  of 
the  sphere  which  has  the  given  circle  on  its  circumference 
and  which  rests  on  the  horizontal  plane  at  a  point  in  that 
great  circle. 

The  plane  of  this  great  circle  is  vertical  and  coincides 
therefore  with  the  vertical  plane  just  drawn.  The  horizontal 
trace  of  this  plane  is  a  tangent  at  the  point  of  the  great 
circle  mentioned  above  as  that  in  which  the  sphere  touches 
the  horizontal  plane. 

(4)  Rabat  this  vertical  plane  and  with  it  the  two 
points  into  the  H.P. 

(5)  Determine  a  circle  passing  through  these  points 
and  touching  the  horizontal  trace  of  the  vertical  plane  (3). 
The  centre  of  this  circle  is  that  of  the  sphere  constructed, 
and  its  radius  that  of  the  sphere  required. 

Find  plan  and  elevation  of  the  centre  and  describe 
circles  therefrom  with  the  radius  found. 

The  circles  so  drawn  are  the  required  projections  of  the 
sphere. 

6.  Determine  three  points^  A,  B,  C,  at  unequal  distances 
from  an  assumed  oblique  pla7ie,  and  construct  the  sphere  which 
shall  touch  the  oblique  plane  and  pass  through  the  three  points. 

Note.  The  three  points  must  be  on  the  same  side  of 
the  plane  and  must  not  lie  in  the  same  straight  line. 


no         SOLID  OR  DESCRIPTIVE   GEOMETRY. 

(i)  Find  the  projections  a,  )8,  7,  of  the  three  points 
A,  B,  C,  upon  the  oblique  plane. 

(2)  Determine  a  plane  passing  through  A,  B,  C,  and 
find  its  intersection  with  the  same  plane. 

(3)  Rabat  the  oblique  plane  with  its  contained  points, 
a,  /3,  y,  and  the  intersection  found  in  (2),  into  the  horizontal 
plane. 

Then,  since  the  lengths  of  the  projectors,  aA,  (3B,  yC, 
are  known,  the  points  A,  B,  C,  relatively  to  the  rabatted 
oblique  plane,  can  be  set  up,  the  inclination  of  their  plane 
determined,  and  a  circle  described  through  them. 

(4)  Find  the  centre  and  radius  of  the  sphere  that 
touches  the  rabatted  oblique  plane  and  has  this  circle  on  its 
surface,  as  in  the  preceding  problem,  and  lift  the  oblique 
plane,  and  the  centre  of  the  sphere,  back  into  position. 

For  another  construction  see  Bradley's  Ektnents  of 
Geometrical  Drawings  Part  11.  Plate  xxxii. 

7.  Three  spheres^  A,  B,  C,  of  2-3,  1*9,  and  1*3  itiches 
diameter^  rest  upon  the  horizontal  plane,  each  touching  the  other 
tivo.  Determijie  plan  and  elevation  of  a  fourth  sphere,  75 
inch  radius,  touching  all  three. 

(i)  Draw  plan  and  elevation  of  the  three  spheres,  A,  B, 
C,  as  in  Problem  2  above. 

(2)  Determine  an  elevation  of  two  of  the  spheres,  as 
A,  B,  on  a  vertical  plane  parallel  to  the  line  joining  their 
centres.  The  elevation  will  be  two  circles  touching  at  a 
point  on  their  circumferences  which  is  a  projection  of  the 
point  in  which  the  spheres  themselves  touch. 

(3)  Draw  a  third  circle  of  75  inch  radius  touching 
each  of  the  two  circles  determined  above.     Consider  the 


CURVED  SURFACES— THE  SPHERE.  HI 

circle  last  drawn  as  a  projection  of  the  sphere  of  75  inch 
radius  which  is  required  to  touch  all  three  given  spheres. 
From  the  drawing  it  will  be  apparent  that  the  elevation 
drawn  is  that  of  a  sphere  75  inch  radius  touching  two  only 
of  these  spheres,  viz.  A  and  B.  Conceive  the  sphere  of  75 
inch  radius  to  roll  round  the  two  spheres  A,  B,  and  preserve 
its  contact  with  each  during  the  revolution.  The  centre  of 
the  revolving  sphere  will  describe  a  circle  in  a  plane  at  right 
angles  to  the  line  AB,  which  circle,  in  elevation,  will  be  a 
straight  line  drawn  from  the  centre  of  the  circle  of  75  inch 
radius  perpendicular  to  a'b'  and  produced  to  an  equal 
distance  on  the  opposite  side  of  this  line. 

Note.  The  circle  described  by  the  centre  of  the  re- 
volving sphere  is  the  locus  of  the  centres  of  all  spheres  of 
75  inch  radii  which  touch  each  of  the  spheres  A,  B. 

(4)  Determine  plan  of  this  circle.  The  plan  will  be 
an  ellipse  with  its  major  axis  at  right  angles  to  ab. 

(5)  Proceed  similarly  with  one  of  the  two  spheres 
taken  above  and  the  third,  C,  and  determine  the  projections 
of  the  locus  of  the  centre  of  a  sphere  of  75  inch  radius 
in  simultaneous  rolHng  contact  with  both.  The  plan  of  this 
locus  will  likewise  be  an  elHpse,  and  the  two  points  where 

.  the  loci  intersect  will  be  shown  in  plan  by  the  intersection 
of  the  ellipses. 

The  points  thus  determined  are  the  centres  of  spheres 
of  75  inch  radii  which  satisfy  the  required  conditions. 

For  practice  another  example  may  be  taken  in  which 
the  three  spheres,  A,  B,  C,  are  at  different  heights  above  the 
horizontal  plane  and  not  in  contact,  taking  care  not  to 
remove  the  centres  so  far  apart  as  to  render  the  problem 
impossible. 


112  SOLID    OR  DESCRIPTIVE   GEOMETRY. 

Note.  After  working  Chapter  x.  the  student  will  see 
that  these  points  are  also  the  two  intersections  common  to 
three  spheres  described  from  the  centres  of  the  three 
given  spheres  A^  B,  C,  with  radii  equal  to  that  of  .(4  +  75, 
B+"}$,  ^+75,  from  the  centres  oi A,  B,  C,  respectively. 

8.  Detertnine  two  points  on  the  surface  of  a  given  sphere^ 
and  find  the  projections  of  the  shortest  line  that  can  be  traced 
on  the  sphere  between  them. 

Note.  It  will  be  a  segment  of  the  great  circle  passing 
through  the  points. 

The  Cylinder. 
Def.     a  cylinder  is  the  surface  generated  by  a  straight 
line  which  moves  parallel  to  itself  and  always  passes  through 
a  given  curved  line. 

A  right  cylindrical  surface  is  one  whose  plane  section 
at  right  angles  to  its  generatrices  is  a  circle. 

The  traces  of  a  cylinder  are  the  curves  in  which  it  inter- 
sects the  horizontal  and  vertical  planes  of  projection. 

The  directrix  of  a  cylindrical  surface  may  be  a  line  of 
single  or  double  curvature,  but  it  can  in  eveiy  case  be 
reduced  to  a  plane  curve — its  trace^  for  example.  Generally, 
any  curved  line  whatever  can  be  regarded  as  the  intersection 
of  two  cylindrical  surfaces  whose  generatrices  are  respec- 
tively perpendicular  to  the  horizontal  and  vertical  planes 
of  projection,  and  directrices  plane  curves  lying  in  those 
planes.  Conversely,  the  projection  of  any  curved  line 
whatever  on  a  plane  is  the  trace,  on  that  plane,  of  a 
cylindrical  surface  whose  generatrices  are  perpendicular 
to  the  plane  of  projection,  and  directrix  the  given  curved 
line. 


CURVED  SURFACES— THE  CYLINDER.  113 

"     A  cylinder  is   given  when   one  trace  and  a  generatrix 
are  given. 

1.  Given  ab,  the  major  axis  of  the  ellipse  forming  the 
section  of  a  right  cylindrical  surface  by  the  horizontal  plane 
when  the  axis  of  the  surface  is  inclined  6'^ ;  determine  the 
base  and  the  plan  of  the  frustum. 

Note.  The  minor  axis  of  the  ellipse  is  equal  to  the 
diameter  of  the  required  base. 

The  vertical  plane  through  ab  contains  the  axis  of 
the  cylinder  and  is  at  right  angles  to  its  base.  Rabat 
this  plane  into  the  horizontal.  Bisect  ab  in  o,  and  draw  op' 
making  the  angle  bop'  equal  to  ff'.  Through  p'  draw  I'p'm' 
at  right  angles  to  op' ,  and  meeting  parallels  to  that  line 
through  a  and  b  in  /  and  m'.  Then  I'm'  gives  the  length 
of  the  required  minor  axis  cd.  From  these  data  the  curve 
may  be  readily  drawn.       '  " 

Through  p'  a  perpendicular  to  ab  produced  meets  that 
line  in  /,  which  is  the  centre  of  the  plan  of  the  base  of  the 
cylinder.  The  plan  of  this  base  is  likewise  an  ellipse, 
having  its  major  axis  nq  equal  and  parallel  to  the  minor  of 
the  former,  its  own  minor  Im  being  determined  by  perpen- 
diculars on  ab  drawn  from  /'  and  m' .  Tangents  to  these 
:urves  parallel  to  the  plan  of  the  axis  complete  the  plan 
)f  the  frustum  of  the  cylinder. 

2.  Given  an  ellipse  as  the  horizontal  trace  of  a  right 
cylindrical  surface.  Find  its  plan,  and  an  elevation  on  a 
■  'ertical  plane  making  an  angle  of  -^d^  with  the  plan  of  the 
i  'xis. 

(i)  Describe  a  semicircle  on  the  major  axis  ab,  and 
Irom  one  extremity,  as  a,  set  off  the  length  of  the  minor 

E.  G.  '8 


114  SOLID   OR  DESCRIPTIVE  GEOMETRY. 

axis,  cd,  as  a  chord  of  the  semicircle  to  m.  The  line  drawn 
from  0,  the  centre  of  ab,  at  right  angles  to  am,  is  an 
auxiliary  elevation  of  the  axis  of  the  required  cyUnder  on 
a  vertical  plane  passing  through  ab. 

Parallels  to  the  elevation  of  this  axis  from  a  and  b 
complete  the  auxiliary  elevation  of  the  indefinite  cylinder. 
Tangents  to  the  ellipse,  parallel  to  its  major  axis,  complete 
the  indefinite  plan. 

{2)  Draw  xy  making  an  angle  of  30°  with  the  plan 
of  the  axis,  for  the  new  elevation.  Assume  any  point  P 
in  the  auxiliary  elevation  of  the  axis,  and  find  its  plan  p. 
From  plan  p  draw  a  projecting  line  at  right  angles  to  xy, 
arid  measure  off  on  this  line  above  xy  the  height  of  P, — 
the  point  thus  found  will  be  the  elevation,  /',  on  the 
new  vertical  plane,  of  the  point  P  in  the  axis.  A  line 
from  /'  to  the  point  0'  where  the  perpendicular  from  0 
meets  xy,  will  be  the  elevation  of  the  axis  on  the  new 
vertical  plane. 

The  elevation  of  the  cylinder  may  be  now  completed 
by  drawing  two  lines  parallel  to  the  elevation  of  the  axis, 
one  on  each  side,  at  distances  equal  to  the  semi-minor  axis 
of  the  ellipse. 

3.  To  determine  the  projections^  pp',  of  a  point ,  P,  on  the 
surface  of  a  cylinder. 

Let  /  be  the  plan  of  a  given  point,  P,  on  the  surface 
of  the  given  cylinder.  Through  /  draw  the  plan  of  a 
generatrix  of  the  surface,  and  find  its  elevation.  The  point 
/',  in  which  the  projecting  line  from  the  plan  /  cuts  the 
elevation  of  the  generatrix,  will  be  the  elevation  of  P. 


CURVED  SURFACES— THE   CYLINDER.  11$ 

4.  Draw  the  cylinder  which  would  envelope  a  sphere 
of  I'i,  in.  radius  and  centre  2  "5  iti,  above  the  ground.  Axis 
of  cylifider  inclined  50",  and  making  in  plan  an  angle  ^30° 
with  xy.  Show  the  circle  of  contact  and  the  horizontal  trace 
of  the  cylinder. 

(i)  Draw  projections  of  the  sphere  and  plan  of  the 
axis  of  the  cylinder  according  to  the  conditions. 

(2)  Use  the  plan  of  the  axis  as  the  ground  line  for 
an  auxiliary  V.P.,  in  which  draw  a  second  elevation  of  the 
sphere.  A  line  drawn  through  the  centre  of  this  elevation 
of  the  sphere,  to  meet  the  new  ground  line  at  the  given 
inclination,  will  be  the  elevation  of  the  axis  of  the  required 
cylinder,  and  parallels  to  it,  tangential  to  the  circle  which 
represents  the  sphere,  will  determine  the  major  axis  of 
the  ellipse  in  which  the  cylinder  cuts  the  horizontal  plane. 
The  minor  axis  of  the  curve  will  be  marked  off  by  parallels 
to  the  axis  of  the  cylinder  in  plan,  tangential  to  the  circle 
which  is  the  plan  of  the  sphere.  From  the  new  or  auxiliary 
elevation,  an  elevation  on  the  original  vertical  plane  can 
now  be  easily  determined. 

(3)  The  circle  of  contact  is  shown  in  the  auxiliary 
elevation  as  a  straight  line,  and  from  this  can  readily  be 
determined  in  plan,  and  from  thence  on  the  original  vertical 
plane. 

Note  i.  The  horizontal  trace  of  the  cylinder  might 
also  be  found  by  means  of  generatrices  drawn  through  a 
number  of  assumed  points  in  the  circle  of  contact. 

Note  2.  The  trace  of  this  cylinder  is  the  boundary 
of  the  shadow  of  the  sphere,  cast  upon  the  horizontal 
plane  by  rays  of  light  parallel  to  the  generatrices   of  tha 


Il6  SOLID   OR  DESCRIPTIVE   GEOMETRY. 

cylinder;  and  the  "circle  of  contact"  marks  the  boundary 
of  light  and  shade  on  the  sphere.     See  Shadows,  Chap,  ix, 

5.  The  axes  of  two  equal  cylindrical  surfaces  are  inclined 
in  opposite  directions  at  25°,  50°,  the  line  perpendicular  to  both 
these  axes  is  3  inches  long  and  inclined  at  30",  the  two  surfaces 
touch  each  other  i?i  one  point  only.  Draw  plan  of  cylinders 
showing  point  of  contact.     Elevation  at  pleasure. 

Science  Exam.  Hon.  1869. 
(i)     Determine    projections    of    the    axes    and    their 
common    perpendicular    by    Problem    22,    Exercises    on 
Chap.  III. 

(2)  Bisect  the  plan  and  elevation  of  the  line  of  3  inches 
which  is  perpendicular  to  the  axes.  The  points  of  bisection 
are  the  projections  of  the  point  in  which  the  cylinders 
touch. 

(3)  The  plans  of  the  cylinders  can  be  completed  by 
making  auxiliary  elevations  of  the  axes  on  vertical  planes 
assumed  containing  these  axes.  Lines  parallel  to  the 
elevations  of  the  axes  and  in  the  same  vertical  planes 
with  them,  at  distances  of  i'5  in.  on  each  side,  meet 
the  horizontal  plane  in  points,  on  the  plans  of  the  axes 
produced,  which  are  the  extremities  of  the  major  axes 
of  the  elliptic  traces  of  the  cylinders  on  the  horizontal  plane. 
The  semi-minor  axes  of  the  same  sections  are  equal  to 
1*5  in.,  the  radius  of  the  cylinders,  and  hence  the  ellipses 
can  be  drawn  and  the  plans  and  elevations  completed. 

6.  The  centre,  O,  of  a  sphere  of  i  in.  radius  is  i  "5  in. 
above  the  horizontal  plane  and  2*5  in.  ifi  front  of  the  vertical 
plane.  A  straight  line  in  the  horizontal  plane  2*25  in.  from 
the  plan,  o,  of  the  centre  of  the  sphere  and  making  an  angle  of 
30'  with  xy,  is  the  plan  of  the  indefinite  axis,  inclined  at  50*, 


CURVED  SURFACES— THE   CYLINDER.  H/ 

of  a  right  cylindrical  surface  touching  the  sphere.  Determine 
the  traces  of  the  surface  and  the  point  of  contact  with  the  sphere, 
wJun  tfie  latter  is  without  the  cylinder. 

(i)  Determine  a  plane  containing  the  centre  O  of 
the  sphere  and  the  given  axis.  This  plane  cuts  the  sphere 
in  a  great  circle,  and  the  touching  cylinder  in  a  generatrix 
which  is  a  tangent  to  this  circle  at  a  distance  from  the 
axis  equal  to  the  semi-diameter  of  the  cylinder  required. 

(2)  Rabat  this  plane,  with  the  great  circle  and  given 
axis,  into  the  horizontal  plane.  A  line  touching  the  rabatted 
great  circle  parallel  to  the  rabatted  axis  will  give  the  point 
F  of  contact,  from  which  the  projections,  //',  may  be  readily 
obtained.  The  distance  between  these  parallels  will  give 
the  semi-diameter  of  the  cylinder,  whence  the  traces  and  the 
projections  of  the  surface  may  be  determined. 

Note.  The  student  will  understand  the  methods  by 
which  touching  surfaces  are  determined  better  after  he 
has  worked  systematically  through  the  problems  on  tangent- 
planes  to  curved  surfaces  given  later  on. 

Two  surfaces  are  said  to  touch  at  a  point  when  they 
have  a  common  tangent  plane  at  that  point.  A  given 
straight  line  may,  in  general,  be  the  axis  of  two  right  cylinders 
touching  a  given  sphere.  When  the  given  axis  passes 
through  the  centre  of  the  sphere  one  cylinder  only  is 
possible,  and  it  in  this  case  envelopes  the  former  surface. 
When  the  axis  is  a  tangent  to  the  sphere  one  cylinder 
only  is  possible,  and  it  contains  the  sphere.  If  the  axis 
be  entirely  free  from  the  sphere  two  cylinders — one  includ- 
ing and  the  other  excluding  the  sphere — can  be  drawn. 
When  the  axis  meets  the  sphere  in  two  points  and  does 
not  pass  through  its  centre,  two  tangent  cylinders — one  con- 
taining and  one  intersecting  the  sphere — are  determinable. 


Il8  SOLID  OR  DESCRIPTIVE  GEOMETRY. 

7.  A  right  cylindrical  surface  of  2  in.  diameter  rawlves 
uniformly  about  its  axis  fixed  in  position,  and  a  line  is  traced 
upon  it  by  a  point  which  moves  over  the  surface  parallel 
to  its  axis  at  a  uniform  velocity.  Draw  the  projections 
of  the  curve  formed  when  the  tracing  point  moves  i  ///. 
forward  during  the  time  of  each  complete  revolution  of  the 
cylinder. 

Work  two  cases,  (i)  when  the  axis  is  vertical,  (2)  when 
the  axis  is  oblique,  and  a  developme7it. 

Note.  This  curve  is  the  well-known  helix  or  screw- 
thread.  One  of  its  properties  is  that  the  tangent  line 
to  the  curve  at  any  point  is  inclined  to  the  generatrix 
of  the  surface  through  that  point  at  an  angle  constant  for 
every  part  of  the  curve.  Hence  when  the  cylinder  is 
developed  the  curve  becomes  a  straight  line  inclined  at  a 
determinable  angle,  a",  to  the  line  which  is  the  developed 
circular  base  of  the  cylinder.  This  angle  is  called  the 
angle  of  the  screw,  and  is  equal  to  that  which  the  tangent 
line  at  any  part  of  the  curve  makes  with  the  plane  of 
the  base  of  the  cylinder.  The  distance,  P,  through  which 
the  tracing  point  moves  while  the  cylinder  makes  one 
revolution,  is  called  the  pitch  of  the  helix.  Obviously 
P=  2Trr  tana. 

To  work  the  problem,  divide  a  circular  base  of  the 
cylinder  into  any  number  of  equal  parts  and  draw  genera- 
trices through  the  points  of  division.  Let  iV"be  the  number 
of  parts  into  which  the  circular  base  is  divided.  Starting 
from  one  of  the  points  of  division,  measure  off  from  the 
base,  on  the  successive  generatrices,  distances  successively 

P       P       P  P 

equal  to  ■^,  2-^,   3-^ A^— .     P  in  this  case  being 

I  in< 


CURVED  SURFACES— THE   CONE.  HQ 

Join  the  projections  of  these  points  for  the  projections  of 
the  helical  curve. 

The  development  presents  no  difiSculty. 

8.  Assume  any  two  points  on  the  surface  of  the  cylinder 
in  the  last  problem  not  in  the  same  generatrix,  and  show 
the  projections  of  the  shortest  line  that  can  be  traced  on  its 
surface  betiveen  them. 

Develop  the  surface  with  the  two  contained  points, 
and  join  them  by  a  straight  line.  The  points  in  the 
development  in  which  this  line  meets  certain  of  the  genera- 
trices can  then  be  easily  transferred  to  the  corresponding 
projections  of  these  generatrices,  and  the  points  so  found 
be  joined  for  the  projections  of  the  required  line,  which, 
in  this  case,  is  clearly  a  portion  of  a  helix. 

Note.  This  problem  may  be  taken  as  a  type  of  the 
method  by  which  the  shortest  line  between  any  two  points 
on  any  developable  surface  whatever  may  be  found,  or  any 
given  line  laid  upon  the  surface.  See  Prob.  7  on  "The 
Cone." 

The  Cone. 

Def.  a  cone  is  the  surface  generated  by  the  motion 
of  a  straight  line  which  always  passes  through  a  fixed  point 
and  a  given  curved  line. 

The  fixed  point  is  called  the  vertex  of  the  cone. 

The  directrix  may  be  of  single  or  double  curvature,  but 
every  conic  surface  may  be  represented  by  a  plane  directrix, 
a  fixed  point,  and  a  rectilineal  generatrix. 

If  the  generatrices  extend  beyond  the  vertex,  a  second 
similar  cone  is  generated.  These  simultaneously  generated 
cones  are  called  the  two  sheets  of  the  surface. 


I20 


SOLID   OR  DESCRIPTIVE   GEOMETRY. 


The  curves  in  which  the  cones  intersect  the  planes  of 
projection  are  called  its  traces. 

A  Right  Conic  surface  is  one  which  can  be  represented 
by  a  circle  as  its  plane  directrix,  and  a  point  for  its  vertex 
in  the  perpendicular  to  the  plane  of  the  circle  at  its 
centre. 

A  Cone  is  given  when  its  vertex  and  a  trace  are  given. 
.  I.     Given  ab,  the  major  axis  of  the  ellipse  which  forms  the 
oblique  section  of  a  right  cone  by  the  horizontal  plane,  and  vv', 
the  projections  of  the  vertex,  to  draw  the  plan  of  the  frustum. 
Fig.  28. 

Fig.  28. 


Since  the  plan  v  in  ab  produced  is  given,  it  will  be 
obvious  that  when  the  ellipse  is  obtained,  the  required  plan 
will  be  completed  by  drawing  tangents  to  the  ellipse  through 
V.     Hence  the  problem  consists  chiefly  in  finding  cd^  the 


CURVED  SURFACES—THE   CONE.  121 

minor  axis  of  the  curve.  This  line  is  the  horizontal  trace  of 
the  plane  of  a  circular  section  of  the  cone  made  through 
point  o\  the  bisection  of  ab^  and  forms  a  chord  of  that  circle. 

This  circle  being  perpendicular  to  the  axis,  VP,  of  the 
cone,  is  inclined  to  the  horizontal  plane  at  the  complement 
of  the  angle  of  inclination  of  that  axis.  The  circle  must 
therefore  be  rabatted  and  its  chord  shown. 

Take  a  vertical  plane  through  Fand  ab;  and  on  it  draw 
v'a,  v'b,  the  traces  of  the  conic  surface.  Through  o'  draw 
k'l'  making  equal  angles  with  these  lines ;  this  is  the  vertical 
trace,  and  elevation  of  the  circular  base. 

Rabat  this  circle  into  the  vertical  plane  as  shown  in 
Fig.  28,  about  k'l\  its  vertical  trace  and  diameter.  Draw 
through  0'  the  chord  CD  at  right  angles  to  k'T.  CD  gives 
the  length  cd  of  the  required  minor  axis. 

2.  To  determine  the  projections,  pp',  of  a  point,  P,  on  the 
surface  of  a  cone. 

Assume  /,  the  plan  of  the  point,  and  through  p  draw  a 
plan  of  the  generatrix  of  the  cone  and  show  its  elevation. 
The  point  /',  in  which  the  projecting  line  from  the  plan  / 
cuts  the  elevation  of  the  generatrix,  will  be  the  elevation  oiP. 

3.  A  cone  with  vertex  5  iji.  high,  axis  inclined  50",  and 
in  plan  making  an  angle  ^30"  with  the  ground  line,  envelops 
a  sphere  of  i  inch  radius  and  centre  2  in.  high.  Determine 
horizontal  trace  of  cone  and  circle  of  contact. 

As  in  Problem  4  on  "  The  CyHnder,"  use  the  plan  of  the 
axis  as  the  ground  Hne  for  an  auxiliary  elevation.  In  this 
auxiliary  plane  set  up  the  elevation  of  the  axis  by  drawing  a 
line  inclined  50°  with  xy  through  the  point  which  is  the 
elevation  of  the  centre  of  the  sphere,  and  determine  vv\  the 


122  SOLID   OR  DESCRIPTIVE   GEOMETRY. 

vertex  of  the  cone,  5  in.  high.  Tangents  from  if  to  the  circle 
representing  the  elevation  of  the  sphere  will  mark  off  on  xy 
the  line  which  is  the  elevation  of  the  horizontal  trace  of  the 
cone.  This  same  line  is  also  the  length  of  the  major  axis  of 
the  ellipse  which  is  the  horizontal  trace  of  the  cone. 

To  find  the  minor  axis  of  the  same  ellipse,  the  point 
must  be  determined,  on  the  circle  of  contact,  through  which 
a  line  from  the  extremity  of  the  minor  axis  to  the  vertex 
would  pass.  In  the  auxiliary  elevation  this  line  will  be 
shown  by  joining  the  vertex  and  the  centre  of  the  major 
axis.  The  intersection  of  this  line  with  the  straight  line 
representing  the  elevation  of  the  circle  of  contact  gives  the 
elevation  of  the  point  required,  through  the  plan  of  which 
point  a  line  drawn  from  the  plan  of  the  vertex  marks  off  the 
extremity  of  the  minor  axis  sought. 

See  also  "Note"  i,  Prob.  4,  on  "The  Cylinder." 
Note.  The  straight  hne  which,  in  the  auxiliary  eleva- 
tion, will  be  the  elevation  of  the  circle  of  contact,  will  not 
pass,  as  in  the  case  of  the  enveloping  cylinder,  through  the 
elevation  of  the  centre  of  the  sphere.  It  is  the  line  joining 
the  tangent  points  before  determined. 

The  plan  and  new  elevation  of  the  circle  of  contact  can 
now  be  readily  determined.  The  student  should  show  the 
traces  of  the  plane  of  the  circle  of  contact. 

The  horizontal  trace  of  the  cone  would  represent  the 
shadow  of  the  sphere,  cast  upon  the  H.P.  by  rays  of  light 
proceeding  from  the  fixed  point  V;  and  the  circles  of  contact 
would  mark  the  boundary  of  the  light  and  shade. 

4.  The  centre,  O,  of  a  sphere  of  i  in.  radius,  is  i  '5  in. 
above  the  horizontal  plane,  and  2*5  ;«.  in  front  of  the  vertical 
plane.    A  straight  line  in  the  /wrizontal plane,  2-2^  in.  from 


CURVED  SURFACES— THE   CONE.  123 

the  plan,  o,  of  the  centre  of  the  sphere,  mid  making  an  angle  of 
30"  with  xy,  is  the  plan  of  the  axis,  inclined  at  50°,  of  a  right 
cone  whose  convex  surface  touches  the  sphere.  The  vertex  of 
the  cone  is  5  in.  high,  and  its  circular  base  touches  the  hori- 
zontal plane.  Determine  its  plan  and  elevation  and  the  point 
of  contact  of  the  surfaces. 

(i)     Find  the  projections  vi/  of  the  vertex  V. 

(2)  Determine  a  plane  containing  the  centre  O  of  the 
sphere  and  the  axis,  as  in  Prob.  6  on  "  The  Cylinder." 

(3)  Rabat  this  plane  with  its  contained  great  circle  and 
axis.  Let  V  be  the  rabatted  vertex.  From  V  draw  a  Hne 
touching  the  rabatted  great  circle  in  P'.  The  angle  between 
the  rabatted  axis  and  this  tangent  line  is  the  semi-vertical 
angle  of  the  cone.  When  this  angle  is  found,  the  projections 
of  the  cone  are  easily  determined  by  the  aid  of  an  auxiliary 
vertical  plane  of  elevation  containing  the  axis. 

(4)  The  projections//'  of  the  point  of  contact  P  are  at 
once  determinable  from  the  rabatted  point  P'. 

The  "Note"  to  Prob.  6  on  "The  Cylinder"  applies, 
mutatis  mutandis,  to  this. 

5.  Refer  to  Prob.  4  on  ^^The  Sphere.^ ^  A  point  I'sin. 
dista?it  in  plan  from  the  centre  of  the  sphere  and  4*5  in.  high,  is 
the  vertex,  Y,ofa  conic  surf  ace  of  which  the  traced  curve  is  the 
directrix;  find  its  horizontal  trace. 

It  is  only  necessary  to  draw  a  number  of  generatrices 
from  V  through  points  in  the  curve,  the  line  joining  the 
horizontal  traces  of  these  generatrices  will  be  the  required 
trace  of  the  conic  surface. 

6.  A  right  cone  revolves  uniformly  about  its  axis  fixed  in 
position.,  and  a  curve  is  traced  upon  its  surface  by  a  point 


1-24  SOLID   OR  DESCRIPTIVE   GEOMETRY. 

moving  with  uniform  velocity  in  a  straight  line  from  the  vertex 
to  the  base  of  the  cone  in  a  fixed  plane  containing  the  axis. 
Determine  the  projection  of  the  curve  when  the  rotation  of  the 
cone  and  the  motion  of  the  point  are  so  timed,  that  the  point 
moves  from  the  vertex  to  the  base  while  the  cone  is  makifig  one 
revolution.     Radius  of  base  i'5  in.;  axis  3  "5. 

The  student  after  working  Prob.  4  on  "The  Sphere,"  and 
Prob.  7  on  "  The  CyHnder,"  will  find  this  problem  and  its 
development  an  easy  exercise.  It  may  be  interesting  to 
note  that  this  curve  is  Hogarth's  famous  "Hne  of  beauty." 

The  gradient  of  this  spiral  depends  entirely  upon  the 
relative  velocity  of  the  tracing  point  and  the  cone. 

7.  A  semicircle  is  described  upon  a  generatrix  of  the  cone 
in  the  last  problejti  as  a  diameter,  and  the  plane  cofitaining  it  is 
wrapped  round  the  conic  surface.  Draw  the  projections  of  the 
resulting  curve. 

Develop  the  cone  and  describe  the  semicircle  on  a 
generatrix  in  the  development.  If  the  segment  of  the  circle 
which  is  the  development  of  the  base  of  the  cone  be  divided 
into  any  number  of  equal  parts,  and  the  circular  base  of  the 
solid  be  divided  into  the  same  number  of  equal  parts,  a  set 
of  generatrices  in  the  development  may  be  drawn  of  which 
the  corresponding  projections  are  easily  obtainable.  The 
points  in  which  certain  of  the  generatrices  in  the  develop- 
ment cut  the  semicircle  may  now  have  their  corresponding 
projections  found,  in  the  projections  of  these  generatrices,  by 
proportionate  division,  and  the  curve  be  drawn  through  them 
in  plan  and  elevation.  Similarly  any  plane  curve  may  be 
wrapped  round  any  developable  surface  whatever,  or  the 
shortest  line  between  any  two  given  points  be  traced  on  the 
surface.  For  an  example  of  the  latter  problem,  see  No.  8 
on  "The  Cylinder." 


TANGENT-PLANES  AND  NORMALS.  12$ 

Tangent-planes  and  Normals  to  Curved  Surfaces. 
General  Remarks. 
Def.  The  tangent  line  to  a  curve  at  any  point  P  is  the 
limiting  position  a  straight  line  passing  through  P,  and 
cutting  the  curve  in  another  point  (2,  assumes  when  the 
said  straight  line  is  turned  about  P  in  such  a  manner  that  <2 
continually  approaches  P  and  ultimately  coincides  with  it. 

The  tangent  line  to  a  curve  at  any  point  is  also  a 
tangent,  at  the  same  point,  to  any  surface  on  which  the 
curve  is  traced. 

Def.  The  tangent-plane  to  a  curved  surface  at  any 
point  P  is  the  plane  containing  two  tangent  lines,  at  the 
same  point  P,  to  any  two  curves  traced  upon  the  surface 
through  that  point. 

Theorem,  The  tangent-plane  to  a  curved  surface  at  any 
point  P  contains  all  the  tangent  lines  drawn  at  the  same 
point  to  all  the  lines  that  can  be  traced  upon  the  surface  at 
that  point. 

Cor.  If  one,  or  more,  straight  lines  can  be  traced  on  a 
curved  surface  at  a  point  /*,  the  tangent-plane  to  the  surface, 
at  that  point,  meets  the  surface  in  the  said  line  or  lines. 

Thus,  the  tangent-plane  to  a  Ruled  Surface  at  any  point 
meets  the  surface  along  a  generatrix  drawn  through  that 
point.  When  the  surface  is  Developable,  e.g.  the  cone  and 
cylinder — the  tangent-plane  at  a  point  is  also  a  tangent- 
plane  along  a  generatrix. 

Def.  The  normal  to  a  curved  surface  at  any  point  P  is 
the  perpendicular  drawn  through  P  to  the  tangent-plane  to 
the  surface  at  that  point. 


126  SOLID   OR  DESCRIPTIVE  GEOMETRY. 

Only  one  normal  can  be  drawn  to  a  surface  at  the  same 
point. 

All  the  normals  to  a  cone  or  a  cylinder  (or  other  Develop- 
able surface)  along  a  generatrix  lie  in  one  plane  perpendicular 
to  the  tangent-plane  which  touches  the  surface  along  the 
same  generatrix.  This  is  not  true  of  Twisted  or  Skew  surfaces. 

Two  curved  surfaces  touch  at  a  point  when  the  tangent- 
plane  to  one  of  the  surfaces  at  that  point  is  also  a  tangent- 
plane  to  the  other. 


Two  lines  that  meet  completely  determine  the  plane 
containing  them.  From  this  and  the  definition  of  a  tangent- 
plane  given  above,  it  follows  that,  in  general,  the  problem  of 
constructing  a  tangent-plane  to  a  surface  is  reducible  to 
that  of  drawing  two  tangent  lines  to  any  two  lines  that  can 
be  conveniently  traced  on  the  surface  at  the  point  of  contact 
of  the  tangent-plane.  When  the  surface  is  a  Rtded  one  the 
problem  is  further  simplified  by  one  at  least  of  the  lines 
traceable  on  the  surface  through  the  point  of  contact  being 
straight.  Sometimes,  as  for  example  in  the  hyperboloid  of 
one  sheet  and  the  hyperbolic  paraboloid,  two  such  straight 
lines  can  be  drawn. 

When  the  normal  to  the  required  tangent-plane  is  easily 
determinable,  it  may  be  first  drawn  and  the  tangent-plane 
subsequently  taken  at  right  angles  to  it.  Occasionally, 
auxiliary  surfaces,  having  the  same  tangent-plane  as  the 
given  surface,  may  be  advantageously  assumed.  Thus  the 
following  theorem  is  frequently  required: — 

Theorem.  The  tangent-plane  to  a  cone  or  cylindrical 
surface  is  a  tangent-plane  to  the  spheres  or  surfaces  of 
revolution  enveloped  by  it. 


TANGENT-PLANES  TO   CONES.    •  12? 

It  was  pointed  out  when  treating  of  the  cylinder,  that 
any  curved  line  in  space  might  be  regarded  as  the  inter- 
section of  two  surfaces,  consequently  the  line  of  intersection 
of  two  tangent-planes,  one  to  each  surface,  at  the  common 
point,  P,  would  be  a  tangent  line  to  the  curve  at  the  same 
point.  This  tangent  line  is  obviously,  therefore,  perpen- 
dicular to  the  plane  containing  the  two  normals  to  the 
surfaces.  Thus,  if  P  were  a  point  in  the  intersection  of 
two  spheres  of  centres  X  and  Y,  a  line  through  P  perpen- 
dicular to  the  plane  of  XYP  would  be  a  tangent  line  to 
the  curve  of  intersection  at  the  point  P. 

PROBLEMS  ON  TANGENT-PLANES  TO  CURVED 
SURFACES. 
In  the  foregoing  problems  on  curved  surfaces  it  was 
found  convenient  to  treat  the  sphere  first,  and  to  follow  with 
the  cylinder  and  cone.  In  the  treatment  of  Tangent-Planes 
to  these  surfaces  this  order  can  be  inverted  with  consider- 
able advantage. 

Tangent-Planes  to  Cones. 

I.  A  right  cone,  base  2  in,  diameter,  and  axis  4  in.  long,  is 
so  placed  that  the  circular  base  touches  the  horizontal  plane  ; 
the  axis  is  inclined  at  50",  and  its  plan  makes  an  angle  0/^0" 
with  xy.  Determine  a  point  P  on  the  conic  swface  and  draiv 
a  tangent-plane  to  the  cone  at  that  point. 

Find  the  horizontal  trace  of  the  cone.  Draw  the  genera- 
trix VP,  and  find  its  horizontal  trace.  This  will  be  a  point 
in  the  ellipse  which  is  the  horizontal  trace  of  the  surface. 
Through  this  point  draw  a  tangent  line  to  the  ellipse  for  the 
H.T.  of  the  tangent-plane. 

The  point  in  which  the  H.T.  meets  xy  will  be  one 
point  in  the  V.T. ;  another  will  be  the  vertical  trace  of  a 


128  SOLID  OR  DESCRIPTIVE  GEOMETRY. 

line  drawn  from  V  parallel  to  H.T.  Through  these  two 
points  the  vertical  trace  (V.T.)  of  the  tangent-plane  can 
be  drawn. 

Note.  The  plane  containing  the  generatrix  VP  and  the 
axis  of  the  cone,  contains  all  the  normals  to  the  tangent - 
plane,  and  is  at  right  angles  to  it.  Therefore,  if  a  line  be 
drawn  from  any  point  in  the  generatrix  (as  V)  at  right 
angles  to  the  plane  of  the  normals  its  horizontal  trace  will 
be  a  point  in  the  H.T,  of  the  tangent-plane. 

This  enables  us  to  find  the  H.T.  of  the  tangent-plane 
without  depending  on  the  plane  geometrical  problem  for 
drawing  a  tahgent  line  to  a  conic  section. 

2.  Take  the  same  cone  as  in  the  last  problem,  and  assume 
a  point  P  outside  the  conic  surface.  Draw  tangent-plane  as 
before,  and  the  line  of  contact. 

a.  Find  the  horizontal  trace  of  the  conic  surface,  and 
the  horizontal  trace  of  the  line  KP.  The  H.T.  of  the 
tangent-plane  will  be  the  line  drawn  from  the  horizontal 
trace  of  the  line  VP  to  touch  the  horizontal  trace  of  the 
conic  surface.  Two  such  lines  may  be  drawn,  i.e.  two 
tangent-planes,  each  satisfying  the  conditions,  are  possible. 
The  V.T.s  may  be  found  as  in  the  last  problem, 

(3.  When  the  horizontal  trace  of  VP  is  inaccessible, 
the  point  P,  or  any  convenient  point  in  the  line  VP,  may  be 
made  the  vertex  of  a  similar  cone,  and  the  two  H.T.s  drawn 
touching  the  two  horizontal  traces  of  the  conic  surfaces.  In 
the  particular  case  in  which  the  line  VP  is  parallel  to  the 
horizontal  plane,  the  H.T.s  will  be  tangents  drawn  to  the 
horizontal  trace  of  the  conic  surface  parallel  to  the  plan  of 
the  line  VP. 


TANGENT-PLANES  TO  CONES.  1^9 

The  lines  of  contact  will  be  the  generatrices  of  the  cone 
drawn  from  the  points  in  which  the  H.T.s  of  the  tangent- 
planes  touch  the  horizontal  trace  of  the  surface. 

3.  Take  the  same  cone  as  before  a  fid  any  line  inclined  at 
6"  and  <^°  to  the  horizotital  and  vertical  planes  respectively. 
Draw  a  tangentplane  to  the  cone  parallel  to  this  line. 

Draw  a  line  from  F parallel  to  the  given  line  and  find  its 
horizontal  trace.  The  H.T.s  of  the  tangent-planes  which 
satisfy  the  conditions  can  then  be  drawn  from  this  point 
tangents  to  the  horizontal  trace  of  the  surface. 

Note  the  limits.  When  the  line  from  the  vertex  parallel 
to  the  given  line  falls  inside  the  conic  surface  the  problem  is 
impossible. 

The  above  solution  enables  us  to  draw  a  tangent-plane 
to  a  cone  perpendicular  to  a  given  plane  :  such  a  tangent- 
plane  being  of  course  parallel  to  any  line  taken  at  right 
angles  to  the  given  plane. 

4.  To  the  same  cone  as  before  draxv  a  tangent-plane  which 
shall  be  inclined  at  70"  to  the  horizontal  plane. 

Theorem.  A  common  tangent-plane  can  always  be 
drawn  to  two  cones  having  the  same  vertex,  unless  one  of 
the  sheets  of  one  of  the  conic  surfaces  lies  wholly  within  the 
surface  of  the  other  without  touching  it. 

The  number  of  tangent-planes  possible  depends  upon 
the  relation  between  the  two  cones,  and  may  be  one,  two, 
three,  or  four. 

To  work  the  problem,  make  V,  the  vertex  of  the  given 
cone,  the  vertex  also  of  a  right  cone  standing  on  the 
horizontal  plane,  with  generatrices  inclined  at  70". 

E.  G.  -  Q 


130         SOLID  OR  DESCRIPTIVE  GEOMETRY, 

Any  line  that  can  be  drawn  a  tangent  to  both  the 
horizontal  traces  of  the  cones  will  be  a  H.T.  of  a  tangent- 
plane  satisfying  the  requirements  of  the  question ;  its  V.T. 
can  be  found  as  before. 

Limits.  The  given  angle  of  inclination  of  the  tangent- 
plane  must  lie  between  90"  and  a",  where  a"  is  the  angle 
which  the  generatrix  of  minimum  inclination  makes  with 
the  horizontal  plane. 

5.  Given  an  oblique  plane  and  a  straight  line  VO  meeting 
it  in  the  point  V,  determine  a  right  cone  that  shall  touch  the 
given  plane  and  have  the  straight  line  NO  for  its  axis. 

(i)  Construct  the  plane  of  the  base  at  right  angles  to 
the  line  VO  through  the  point  O,  and  find  the  intersection 
of  this  plane  with  the  given  plane. 

(2)  Rabat  the  plane  of  the  base,  with  the  point  O  and 
the  line  of  intersection,  into  one  of  the  planes  of  projection, 
and  determine  the  radius  of  the  circle  having  O  for  its 
centre  and  the  intersection  line  for  its  tangent.  This  will 
be  the  radius  of  the  base  of  the  cone. 

(3)  Take  an  auxiliary  vertical  plane  containing  VO, 
and  thence  complete  the  projections  of  the  cone. 

6.  Given  a  right  cone,  axis  oblique  to  both  planes  of 
projection,  and  a  point  P  in  space.  Determine  a  second  cone 
having  the  point  P  for  its  vertex  and  a  given  straight  line 
passing  through  V  for  its  axis  that  shall  touch  tlu  given  cojie. 

The  two  cones  will  have  for  their  common  tangent-plane, 
a  plane  passing  through  their  vertices. 

Draw,  therefore,  a  tangent-plane  to  the  given  cone 
through  the  given  point  F,  and  determine  by  the  last 


TANGENT-PLANES   TO  CYLINDERS.  I3I 

problem  a  right  cone  touching  this  plane  and  having  the 
given  straight  line  for  its  axis. 

Note.  Two  tangent-planes  can  be  drawn  to  the  given 
cone  through  the  given  point,  hence  two  cones  can  be 
found  satisfying  the  conditions  of  the  problem. 

7.  General  Case.  To  draw  a  normal  to  a  conic 
surface  through  a  given  point  not  in  the  surface. 

Find  the  curve  which  is  the  locus  of  the  feet  of  all  the 
perpendiculars  let  fall  from  the  given  point  upon  all  the 
tangent-planes  to  the  cone.  The  point,  or  points,  in  which 
this  curve  meets  the  surface  will  determine  the  normal  or 
normals. 

To  find  the  intersection  of  this  curve  with  the  given 
surface  a  second  cone  may  be  taken,  having  the  same  vertex 
as  the  first  and  the  curve  for  its  directrix.  Lines  from  the 
point,  or  points,  in  which  the  horizontal  traces  of  these  cones 
meet,  to  the  common  vertex,  will  determine  the  intersection 
of  the  curve  and  the  given  surface. 

8.  General  Case.  To  draw  a  normal  to  a  given  cotie 
parallel  to  a  given  plane. 

Draw  a  tangent-plane  to  the  given  cone  perpendicular  to 
the  given  plane  (see  Prob.  3).  Any  perpendicular  to  this 
tangent-plane  from  a  point  in  its  line  of  contact  Avith  the 
surface  will  be  normal  to  the  given  cone,  and  parallel  to  the 
given  plane. 

Tangent-Planes  to  Cylinders. 

I .  A  right  cylinder y  base  two  inches  diameter  and  axis  four 
inches  long,  is  so  placed  that  the  circular  base  touches  the  hori- 
zontal plane;  the  axis  is  inclined  at  50°  and  its  plan  makes  an 


132  SOLID   OR  DESCRIPTIVE   GEOMETRY. 

angle  of  30"  with  xy.  Determine  a  point,  P,  on  its  surface, 
and  draw  a  tangent-plane  to  the  cylinder  at  that  point. 

Draw  a  generatrix  of  the  cylinder  through  P,  and  find  its 
horizontal  trace  in  that  of  the  cylinder.  The  tangent  to 
the  horizontal  trace  of  the  cylinder  at  the  point  which  is  the 
horizontal  trace  of  the  generatrix  will  be  the  H.T,  of  the 
required  tangent-plane,  the  V.T.  of  which  can  then  be 
found. 

The  generatrix  through  P  is  the  line  of  contact  of  the 
tangent-plane  with  the  surface.  The  plane  through  this  and 
the  axis  contains  all  the  normals  to  the  tangent-plane. 
Hence  the  latter  might  be  determined  at  right  angles  to  the 
plane  of  the  normals  as  in  the  parallel  case  of  the  right  cone. 

2.  Same  cylinder  as  in  the  last  probletn,  and  a  point, 
P,  without  the  surface.  Draw  tangent-plane  and  show  line 
of  contact. 

Through  P  draw  a  line  parallel  to  the  generatrices  of 
the  cylinder.  The  H.T.  of  the  tangent-plane  can  be  drawn 
through  the  horizontal  trace  of  this  line  to  touch  the 
horizontal  trace  of  the  surface.  Find  V.T.  and  the  line 
of  contact  as  in  other  problems  and  compare  with  the 
parallel  problem  on  the  cone. 

3.  Sajne  cylinder  as  before^  and  a  line  inclined  6°  and  ^° 
to  the  horizontal  and  vertical  planes  respectively.  Draw 
tangent-plane  parallel  to  the  given  line. 

Determine  a  plane  containing  the  given  line  and  pa- 
rallel to  a  generatrix  of  the  cylinder.  A  straight  line, 
touching  the  horizontal  trace  of  the  cylinder,  and  parallel 
to  the  horizontal  trace  of  the  plane  determined  above,  will 
be  the  H.T.  of  the  required  tangent-plane,  whence  the  line 
of  contact  and  the  V.T.  may  be  found. 


TANGENT-PLANES  TO   CYLINDERS.  133 

Note  the  limits.  The  angle  6"  must  lie  between  90" 
and  the  angle  of  inclination  of  the  generatrices  of  the 
cylinder. 

This  problem  enables  us  to  determine  a  tangent-plane 
to  a  cylinder  perpendicular  to  a  given  plane. 

4.  Draw  a  plane  inclined  70"  to  the  horizontal  plane  and 
touching  the  cylinder  used  in  the  foregoing  problenis. 

Determine  any  right  cone  standing  on  its  circular  base 
having  its  generatrices  inclined  at  70". 

Find  the  horizontal  trace  of  a  tangent-plane  to  this  cone 
parallel  to  a  generatrix  of  the  given  cylinder.  A  line  pa- 
rallel to  this  horizontal  trace  touching  the  horizontal  trace  of 
the  cylinder  will  be  the  H.T.  of  the  required  tangent-plane. 
Complete  by  drawing  line  of  contact  and  V.T. 

Limits.  The  given  angle  must  lie  between  90"  and 
the  angle  of  inclination  of  the  generatrices  of  the  given 
cylinder. 

5.  Given  two  poi?its,  B  and  C,  at  equal  distances  from 
a  given  plane,  as  the  foci  of  an  elliptic  section  of  a  right 
cylinder  which  touches  the  plane:  determine  the  cylinder  atid  its 
line  of  contact  with  the  plane. 

The  plane  of  the  ellipse  is  perpendicular  to  the  given 
plane,  and  the  curve  touches  the  given  plane  at  the  ex- 
tremity, (2,  of  its  minor  axis.  This  point  may  be  determined 
by  a  perpendicular  to  the  plane  through  (9,  the  point  of 
bisection  of  BC.  The  line  BC  being  parallel  to  the  given 
plane,  the  foot  of  this  perpendicular  from  O  to  the  plane  will 
be  (2- 

The  major  axis  will  be  equal  to  BQ-¥  CQ,  whence  the 
ellipse  is  completely  determined. 


134         SOLID   OR  DESCRIPTIVE  GEOMETRY. 

The  position  of  a  circular  section  of  the  cylinder  per- 
pendicular to  the  given  plane,  with  radius  equal  to  OQ,  can 
be  determined,  with  one  extremity  of  a  diameter  coinciding 
with  an  extremity  of  the  major  axis  of  the  ellipse.  See  the 
construction  given  in  Prob.  i  on  "The  Cylinder." 

The  straight  line  through  Q  and  the  point  in  which  the 
circle  touches  the  plane,  is  the  generatrix  in  which  the 
cylinder  touches  the  plane. 

Work  the  problem : 

(i)     When  the  given  plane  is  horizontal. 
(2)        „  „  „        oblique. 

6,  To  determine  a  right  cylinder  which  shall  have  a  given 
straight  line  for  its  axis  and  shall  touch  a  given  right  cylinder. 

It  is  obvious  that  the  two  cylindrical  surfaces  will  have 
a  common  tangent-plane  at  their  point  of  contact.  This 
point,  which  lies  in  a  straight  line  at  right  angles  to  both 
axes,  must  therefore  be  determined. 

Let  AB  be  the  axis  of  the  given  cylinder,  and  CD  that 
of  the  required  one :  AB  not  lying  in  the  same  plane  with 
CD.  Determine  FQ  the  common  perpendicular  to  these 
axes  by  Problem  23,  Chap,  iii.,  meeting  AB  in  point  Pj 
and  CD  in  Q. 

Find  T'the  point  in  which  FQ  pierces  the  surface  of  the 
given  cylinder.  T  is  the  point  of  contact,  and  QT  the 
radius  of  the  circular  section  of  the  required  cylinder, 
which  may  hence  be  constructed.  See  Problem  5  on  "The 
Cylinder  "  for  special  exercise. 

Another  Method. 

Assume  any  point,  F,  and  draw  through  it  parallels  to 
the  two  given  axes.     Determine  the  horizontal  traces  of 


TANGENT-PLANES  TO   CYLINDERS.  135 

these  lines  and  join  them  for  the  horizontal  trace  of  the 
plane  containing  them.  This  plane  will  be  parallel  to  the 
generatrices  of  both  cylinders. 

Another  plane  parallel  to  this,  and  touching  the  given 
cylinder,  will  be  the  common  tangent-plane  of  the  two 
cylinders,  and  its  horizontal  trace  will,  therefore,  be  a  tangent 
line  to  the  horizontal  traces  of  both  surfaces. 

The  horizontal  trace  of  the  required  cylinder  may  be 
drawn  in  either  of  the  following  ways  : — 

(i)  Find  the  horizontal  trace  of  its  given  axis  and  the 
real  distance  from  any  point  M  in  this  axis  to  the  tangent 
plane.  This  will  be  the  radius  of  the  contained  sphere. 
The  cylinder  enveloping  this  sphere  and  having  the  given 
axis  can  be  drawn  by  Problem  4  on  "  The  Cylinder"  and  its 
horizontal  trace  determined.     Or^ 

(2)  Draw  from  any  point  in  the  given  axis  a  perpen- 
dicular to  the  tangent-plane,  and  find  the  point  in  which 
the  perpendicular  meets  the  plane.  A  line  through  this 
point  parallel  to  the  given  axis  will  be  the  line  of  contact  of 
the  tangent-plane  with  the  required  cylinder,  and  the  H.T. 
of  this  line  will  be  a  point  in  the  H.T.  of  the  cylinder. 
The  true  length  of  the  perpendicular  to  the  plane  will  be 
the  semi-minor  axis  of  the  elliptical  trace.  On  drawing  the 
minor  axis  of  the  ellipse  through  the  horizontal  trace  of  the 
given  axis  of  the  cylinder  at  right  angles  to  the  plan  there- 
of, we  have  one  axis  and  a  point  in  the  curve,  from  which 
data  the  ellipse  can  be  drawn,  by  means  of  a  strip  of  paper, 
on  the  well-known  principle  of  the  trammel 

The  generatrices  drawn  from  the  two  points  in  which 
the  ellipses  are  touched  by  the  horizontal  trace  of  the 


13^  SOLID   OR  DESCRIPTIVE   GEOMETRY. 

tangent  plane  will  meet  in  T^  the  point  of  contact  of  the 
cylinders. 

7.  Given  a  right  cylinder,  axis  oblique,  a  point  in  space 
and  a  straight  line  passi?ig  through  it;  determine  a  right  cone 
that  shall  have  the  given  line  for  its  axis,  the  point  for  its 
vertex  and  touch  the  given  cylitider. 

Refer  to  the  parallel  problem  on  "Tangent-Planes  to 
Cones,"  Problem  6,  for  method. 

8.  General  Case.  To  draw  a  non?tal  to  a  cylinder 
through  a  point  not  in  the  surface. 

Construct  the  curve  which  is  the  locus  of  the  feet  of  all 
the  perpendiculars  let  fall  from  the  given  point  upon  all  the 
tangent-planes  to  the  cylinder,  as  for  the  parallel  case  on 
the  cone.  The  intersection  of  this  curve  with  the  cylinder 
will  determine  the  normal  or  normals. 

This  intersection  may  be  found  by  making  the  curve 
the  directrix  of  a  new  cylinder  whose  generatrices  are  paral- 
lel to  those  of  the  given  cylinder.  The  common  genera- 
trices of  these  surfaces,  drawn  from  the  points  in  which  their 
horizontal  traces  meet,  will  determine  the  intersections  of 
the  curve  with  the  given  surface,  and,  therefore,  the  normals, 
if  there  are  more  than  one. 

9.  General  Case.  To  draw  a  normal  to  a  given 
cylinder,  parallel  to  a  given  plane. 

Draw  a  tangent-plane  to  the  given  cylinder  perpen- 
dicular to  the  given  plane  (Prob.  3). 

Any  perpendicular  to  this  tangent-plane  from  a  point  in 
its  line  of  contact  with  the  cylinder  will  be  normal  to  the 
given  surface,  and  parallel  to  the  given  plane. 

1 0.  Determine  a  helix  of  \'<^  in.  pitch  on  the  surface  of 
a  right  cylinder^  axis  vertical,  diameter  of  base  2  in.,  as  in 


TANGENT-PLANES   TO  SPHERES.  137 

Problem  7  on  '■^The  Cylinder"  and  draw  the  curve  in  which 
the  surface  generated  by  the  motion  of  the  tangent  line  to  the 
helix  intersects  the  horizontal  plane. 

Let  P  be  any  point  on  the  helix.  Its  elevation  p'  will 
be  a  point  in  the  elevation  of  that  curve,  and  its  plan  p  a 
point  in  the  circumference  of  the  circle  which  is  the  plan  of 
the  cylinder. 

A  tangent  line  to  this  circle  at  the  point  /  will  be  the 
plan  of  the  tangent  line  to  the  helix  at  the  point  P  in  the 
curve,  and  as  this  line  is  inclined  at  a"  (the  ajigle  of  the 
screw)  to  the  horizontal  plane,  its  horizontal  trace  may  be 
found  for  one  point  in  the  required  curve. 

Similarly  other  points  may  be  found,  and  the  curve, 
which  is  the  involute  of  the  circle  forming  the  base  of  the 
cylinder,  drawn  through  them. 

Note.  This  surface  is  a  "Developable  Helixoid"  of 
which  the  helix  is  the  Edge  of  Regression. 

Tangent-Planes  to  Spheres. 

I.  A  point,  O,  2  in.,  and  i'^  in.,  from  the  horizontal 
and  vertical  planes  respectively,  is  the  ccfitre  of  a  sphere  of  i  i?i. 
radius.  Draw  the  sphere,  and  a  tangent-plane  to  it  at  a  point 
P  on  its  surface,  2-3  in.  from  H.P.,  and  i-8  in.  from  V.P. 

To  find  P,  take  an  auxiliary  vertical  plane  parallel  to  the 
vertical  plane  of  projection,  i*8in.  from  it,  cutting  the  sphere. 
The  elevation  of  the  section  of  the  sphere  by  this  plane  will 
be  a  circle  shown  in  true  shape.  A  point  in  the  elevation 
of  the  circle  2-3  in.  above  xy,  will  be/'  the  elevation  of  the 
point  P,  whence  the  plan  p  can  be  readily  found. 

If  any  two  planes  be  assumed  passing  through  P  and 
the  centre  O  of  the  sphere,  they  will   cut  the   spherical 


138  SOLID  OR  DESCRIPTIVE   GEOMETRY. 

surface  in  two  great  circles  having  a  common  radius  OP^ 
and  since  the  tangent  lines  at  P  to  both  of  these  circles  are 
at  right  angles  to  this  radius,  the  tangent-plane  to  the  sphere 
at  the  same  point  is  also  perpendicular  to  OP ;  i.  e.  OP  is 
the  normal  to  the  tangent-plane.     Hence, 

To  draw  a  tangent-plane  to  the  sphere  at  the  point  -P, 
it  is  only  necessary  to  draw  a  plane  at  right  angles  to  OP 
to  pass  through  the  point  P,  by  Prob.  13,  Chap.  iii. 
Converse. 

Note.  The  normal  to  a  sphere  from  any  point  whatever 
is  obviously  the  straight  line  drawn  from  the  point  to  the 
centre  of  the  sphere. 

2.  Determine  a  tangent-plane  to  the  sphere  in  the  last 
prohlem  containing  a  given  point,  P,  without  the  sphtrical 
surface.  Let  P  be  in  the  vertical  plane,  5  in.  high  and  375  in. 
from  the  elevation  o'  of  the  centre  of  the  sphere. 

Make  P  the  vertex  of  a  cone  enveloping  the  sphere 
^Prob.  3  on  "  The  Cone  " ).  Any  tangent  line  to  the  hori- 
zontal trace  of  the  cone  will  be  the  horizontal  trace  of  a 
plane  fulfilling  the  required  conditions. 

In  cases  in  which  the  horizontal  trace  of  the  enveloping 
cone  is  inaccessible  the  circle  of  contact  may  be  determined, 
and  the  tangent-plane  to  the  sphere  drawn  at  any  assumed 
point  in  this  circle,  by  the  last  problem. 

Note.  There  is  no  limit  to  the  number  of  solutions 
possible. 

3.  Take  the  sphere  and  point  P  in  Prob.  2,  and  assume 
another  point,  Q,  also  without  the  spherical  surface.  Deter- 
mine a  tangent-plane  to  the  sphere  that  shall  pass  through 
P  and  Q. 


TANGENT-PLANES  TO  SPHERES.  139 

Note.  Since  P  and  Q  lie  in  the  tangent-plane,  the 
straight  line  PQ  indefinitely  produced  will  also  lie  in  the 
tangent-plane.  The  tangent-plane  to  the  sphere  through  P 
and  Q  will  therefore  be  a  tangent-plane  to  the  sphere 
through  every  point  in  the  straight  line  PQ.  produced ;  and 
if  this  line  do  not  meet  the  spherical  surface  two  planes 
touching  the  sphere  can  be  drawn  through  it;  if  the  line 
touch  the  sphere  one  only  can  be  drawn,  and  if  it  pass 
through  the  spherical  surface  the  problem  is  impossible. 

First  Method.  Make  P  and  (2,  or  any  two  other  con- 
venient points  in  the  line  PQ,  produced,  the  vertices  of  two 
cones  enveloping  the  sphere.  If  the  horizontal  traces  of 
these  cones  are  determinable,  a  line  touching  them  may  be 
taken  for  the  horizontal  trace  of  the  tangent-plane  required ; 
but  if  not,  determine  the  circles  of  contact  of  the  two  cones 
and  the  points  R  and  6"  in  which  they  intersect,  and  draw 
the  traces  of  the  planes  passing  through  the  points  PQR 
and  PQS,  for  those  of  the  tangent-planes  satisfying  the 
conditions.  Or,  tangent-planes  to  the  sphere  at  the  points 
P  and  S  may  be  drawn  as  in  Prob.  i. 

To  shorten  the  work,  one  cone  may  be  assumed  with  its 
vertex  in  PQ  at  the  same  level  as  the  centre,  (7,  of  the 
sphere,  the  plan  of  the  circle  of  contact  being  in  this  case 
a  straight  line. 

Second  Method.  Take  a  plane  through  the  centre,  O, 
of  the  sphere  at  right  angles  to  the  straight  line  joining  the 
points  P  and  Q.  Let  /  be  the  intersection  of  this  line  and 
plane.  Straight  lines,  through  the  point  7,  touching  the 
great  circle  in  which  this  plane  cuts  the  sphere,  give  the 
points  R  and  S,  whence  the  plane  may  be  determined 
as  before. 


140  SOUD   OR  DESCRIPTIVE   GEOMETRY. 

Third  Method.  Find  the  horizontal  trace  of  the  line 
PQ.t  produced  if  necessary,  and  the  horizontal  trace  of 
a  cone  having  its  vertex  in  this  line.  The  two  tangent 
lines  to  the  conic  trace,  through  the  H.T.  of  the  line  PQ, 
will  be  the  horizontal  traces  of  the  two  tangent-planes. 
The  vertical  trace  of  the  line  will  also  be  a  point  in  the 
vertical  traces  of  the  planes,  which  may  be  thus  determined. 

Fourth  Method.  Draw  a  cylinder  enveloping  the  sphere 
with  its  axis  parallel  to  PQ.  The  two  tangent-lines  to  the 
horizontal  trace  of  the  enveloping  cylinder,  through  the  H.T. 
of  the  line  PQ,  will  be  the  horizontal  traces  of  the  tangent 
planes. 

4.  Take  the  sphere,  and  poijit  P,  as  in  Prob.  2,  and 
determine  a  tangent-plane  to  the  sphere  that  shall  pass  through 
P  and  be  inclined  to  the  horizontal  plane  at  an  angle  of  'j^^. 

First  Method.  Make  the  given  point  P  the  vertex 
of  a  cone  enveloping  the  sphere,  and  draw  a  tangent-plane 
to  this  cone  at  75"  to  the  horizontal  plane,  by  Prob.  4  on 
"Tangent-Planes  to  Cones." 

Second  Method.  Envelop  the  sphere  by  a  cone  with  its 
axis  vertical  and  generatrix  inclined  at  75", 

A  plane  containing  P  and  touching  the  cone  will  satisfy 
the  conditions.     "  Tangent- Planes  to  Cones,"  Prob.  2. 

5.  The  centre,  O,ofa  sphere  of  1  in.  radius  is  i"5  in.  from 
eachpla?ie  of  projection.  Draw  the  traces  of  a  plane  inclined 
at  75"  and  60°  to  the  horizontal  and  vertical  planes  respectively, 
and  touching  the  sphere. 

Determine  a  right  cone,  generatrix  inclined  75"  to  the 
horizontal  plane,  enveloping  the  sphere. 

Similarly,  find  the  vertex,  V,  of  another  right  cone,  genera- 
trix inclined  60"  to  the  vertical  plane,  enveloping  the  sphere. 


TANGENT-PLANES   TO  SPHERES.  I4I 

The  tangent-plane  to  the  first  Cone  through  the  point  F, 
the  vertex  of  the  latter,  satisfies  the  given  conditions. 

Note.  Four  planes — two  parallel  pairs — are  possible. 
Compare  this  problem  with  Prob.  7,  Chap.  in. 

6.  Determine  the  sphere,  and  the  point  P,  as  in  Prob.  2. 
Draw  the  traces  of  a  tangent-plane  to  the  sphere  that  shall 
pass  through  P  and  be  parallel  to  a  line  inclined  at  60°  to  the 
horizontal  plane  and  30"  to  the  vertical. 

Envelop  the  sphere  by  a  cone  having  P  iox  its  vertex, 
and  determine  a  tangent-plane  to  this  cone  parallel  to  the 
given  line.     "Tangent-Planes  to  Cone,"  Prob.  3. 

Note.  Refer  to  Problem  3,  "Tangent-Planes  to  Sphere," 
and  compare. 

7.  Determine  a  tangent-plane  to  two  spheres  of  miequal 
sizes,  unequal  heights,  and  free  from  one  another,  which  shall 

(a)     contain  a  given  point  P, 

(fi)     be  parallel  to  a  giveji  straight  line. 

Envelop  the  two  spheres  by  a  cone. 

Note.  Two  enveloping  cones  are  possible.  In  one  case 
both  spheres  will  be  enveloped  by  the  same  sheet  of  the 
conic  surface,  and  in  the  other  case,  one  sphere  will  be 
enveloped  by  one  sheet  of  the  cone  and  the  other  sphere  by 
the  other  sheet. 

For  (a).  Determine  a  tangent-plane  to  the  enveloping 
cone  that  shall  contain  P.  Or,  Make  P  the  common  vertex 
of  two  cones — one  enveloping  one  sphere  and  the  other 
the  other  sphere — and  determine  the  tangent-plane  to  them. 

For  (^).  Determine  a  tangent-plane  to  the  enveloping 
cone  parallel  to  the  given  line. 


142  SOLID  OR  DESCRIPTIVE  GEOMETRY, 

8.  Three  equal  spheres,  diameters  2  in.  each,  have  their 
centres  at  the  angles  of  a  vertical  equilateral  triangle  of  4  in. 
side,  with  one  side  horizontal.  Draw  a  tangent-plane  to  the 
three  spheres : — 

(a)  To  pass  above  the  two  lower  and  under  the  upper 
sphere. 

(fi)  To  pass  above  one  of  the  lower  and  under  the  other 
two  spheres. 

For  (a).  Draw  an  auxiliary  elevation  on  a  vertical  plane 
at  right  angles  to  the  plane  of  the  triangle.  A  tangent 
to  the  two  circles  which  are  the  elevations  of  the  three 
spheres  can  be  drawn  for  the  auxiliary  vertical  trace  of 
the  required  plane.  The  H.T.  will  of  course  be  perpendi- 
cular to  the  auxiliary  xy. 

For  (/8).  Make  an  elevation  on  a  vertical  plane  parallel 
to  the  plane  of  the  triangle. 

Determine  a  cylinder  containing  the  upper  and  one 
of  the  lower  spheres,  and  a  second  cylinder  parallel  to  the 
first  cylinder  and  containing  the  third  sphere. 

The  horizontal  trace  of  the  required  plane  can  now 
be  drawn  as  a  tangent  to  the  two  ellipses  which  are  the 
horizontal  traces  of  the  two  cylinders. 

The  tangent  lines  on  the  cylinders  can  be  shown  by 
drawing  generatrices  from  the  tangent  points  of  the  ellipses. 

Find  the  points  of  contact  with  the  spheres  by  drawing 
perpendiculars  to  the  tangent-plane  from  their  centres. 
These  points  are  fixed  where  the  projections  of  the  perpen- 
diculars cut  the  generatrices  which  are  the  lines  of  contact 
with  the  cylinders. 

9.  Draw  the  three  spheres  touching  in  Problem  2  on 
**The  Sphere"  and  determine  a  tangent-plane  to  the  three. 


TANGENT-PLANES  TO  SPHERES.  143 

Note.  The  horizontal  plane  of  projection  is  one 
tangent-plane. 

To  find  the  other.  Determine  plan  and  elevation  of  the 
axes  of  two  cones,  one  enveloping  A  and  B,  and  the  other 
enveloping  A  and  C.  Lines  through  the  centres  of  the 
spheres  give  these  axes.  The  horizontal  traces  of  these  axes 
will  be  the  vertices  of  the  enveloping  cones,  and  will  also  be 
points  in  the  H.T.  of  the  tangent-plane  required.  The 
vertical  trace  will  be  most  readily  found  by  the  aid  of 
an  auxiliary  vertical  plane  taken  at  right  angles  to  the 
H.T.  of  the  tangent-plane. 

10.  Determine  the  tangent-planes  to  three  spheres  A,  B,  C, 
of  unequal  sizes,  unequal  heights,  and  free  from  one  another. 

Determine  two  cones  enveloping  A  and  B,  and  A  and  C, 
respectively.  A  tangent-plane  to  one  of  these  cones  con- 
taining the  vertex  of  the  other  will  satisfy  the  conditions.  Or, 
the  vertex  of  one  of  the  cones  enveloping  two  of  the  spheres 
may  be  made  the  vertex  of  a  second  cone  enveloping 
the  third  sphere,  and  the  tangent  plane  to  these  cones 
determined  for  the  tangent-plane  required.  The  student 
will  here  see  that  this  affords  a  method  of  determining  a 
tangent-plane  to  a  given  cone  and  a  given  sphere. 

Refer  to  "Note,"  Problem  7  above,  and  observe  that 
eight  tangent-planes  to  the  three  spheres  can  be  found. 

The  student  can  exercise  himself  by  showing  that  the 
vertices  of  the  six  possible  enveloping  cones  are  in  four 
straight  lines — three  vertices  in  each  line. 

11.  Determine  the  cone,  Prob.  i.  ^'^  Tangent- Planes  to 
Cones,"  and  a  sphere  75  in.  radius  touching  the  horizontal 

plane  in  a  point,  P,  assumed  at  pleasure.     Show  the  traces  of 
a  tangent-plane  to  the  sphere  and  the  cone. 


144  SOLID   OR  DESCRIPTIVE   GEOMETRY. 

One  solution  of  this  problem  was  referred  to  in  No.  lo 
above. 

Another  Method.  Envelop  the  sphere  by  a  cone 
similar  to  the  given  one,  and  determine  a  tangent-plane 
to  one  of  these  cones  and  containing  the  vertex  of  the 
other.     This  will  be  the  plane  required. 

1 2.  To  draw  a  normal  to  a  sphere  parallel  to  a  given 
straight  line. 

A  line  through  the  centre  of  the  sphere  parallel  to 
the  given  line  will  be  the  normal  required. 

13.  To  draw  a  tangent-plane  to  a  sphere  parallel  to  a 
given  pla7ie. 

Draw  a  perpendicular  from  the  centre  of  the  sphere 
to  the  given  plane.  This  will  be  the  normal  to  the  required 
tangent-plane,  which  may  at  once  be  drawn  at  right  angles 
to  the  normal  through  the  point  in  which  the  latter  meets 
the  surface. 

Problems  on  Surfaces  of  Revolution. 

Having  already  defined  a  surface  of  revolution  in  the 
general  remarks  at  the  head  of  this  chapter,  it  will  be  only 
needful  here  to  summarise  the  chief  properties  in  the  follow- 
ing theorems : — 

Theorem  I.  All  sections  of  a  surface  of  revolution  by 
planes  at  right  angles  to  the  axis,  are  circles  having  for  their 
centres  the  points  in  which  the  axis  meets  the  section  plane. 

Theorem  II.  All  meridian  sections  of  a  surface  of 
revolution  are  similar  and  equal  curves. 

Theorem  III.  All  the  tangent  lines  drawn  from  a 
fixed  point  in  the  produced  axis  of  a  surface  of  revolution 


SURFACES  OF  REVOLUTION.  145 

to  meridian  sections,  touch  the  surface  in  points  which  lie 
on  the  circumference  of  a  circle  whose  plane  is  at  right 
angles  to  the  axis.  That  is,  the  locus  of  all  tangent  lines 
to  a  surface  of  revolution  drawn  from  a  fixed  point  in  the 
produced  axis,  is  a  right  cone. 

Theorem  IV.  The  tangent  line  to  any  circular  section 
of  a  surface  of  revolution  at  a  point  in  it,  is  perpendicular 
to  the  meridian  section  of  the  surface  through  that  point. 

Cor.  I.  The  tangent-plane  to  the  surface  at  any  point 
is  perpendicular  to  the  meridian  plane  through  the  same 
point. 

Cor.  2.  The  normal  to  a  surface  of  revolution  at  any 
point,  lies  wholly  in  the  meridian  plane  passing  through  that 
point,  and  is  normal  to  the  meridian  section  at  the  same 
point. 

Cor.  3.  The  normal  from  a  given  point  to  a  surface  of 
revolution  is  the  normal  to  the  section  of  the  surface  made 
by  a  meridian  plane  containing  the  given  point. 

Cor.  4.  The  tangent-planes  to  a  surface  of  revolution 
at  different  points  in  the  same  circular  section  pass  through 
the  same  point  in  the  axis. 

Cor.  5.  The  normals  to  a  surface  of  revolution  at 
different  points  in  the  same  circular  section  meet  the  axis  at 
the  same  point. 

A  surface  of  revolution  is  given  when  its  axis  and  its 
plane  generatrix  (meridian  section)  are  given.  But,  what- 
ever line  be  given  as  the  generatrix,  it  can  always  be  reduced 
to  an  equivalent  plane  generator. 

ll\^Q..right  cone,  and  the  right  cylinder,  may  be  regarded  as 
surfaces  of  revolution,  but  in  the  general  treatment  of  conic 

£.  G.  10 


146  SOLID   OR  DESCRIPTIVE   GEOMETRY. 

and  cylindrical  surfaces  it  is  necessary  to  consider  them  in 
another  point  of  view.  This  has  been  already  done  as  far 
as  the  limits  of  this  book  allow.  The  sphere,  which  is 
essentially  a  Surface  of  Revolution,  has  also  for  convenience 
been  already  separately  considered. 

The  surface  generated  by  an  ellipse  revolving  about 
either  of  its  axes  is  called  a  Spheroid.  It  is  also  a  particular 
case  of  an  ellipsoid,  namely  an  Ellipsoid  of  Revolution.  It  is 
prolate  or  oblate  according  as  it  revolves  about  the  major  or 
minor  axis. 

A  few  problems  of  a  general  character  here  follow: — 

I.  An  ellipse,  major  axis  275  in.^  minor  axis  2  in., 
generates  a  surface  by  revolving  about  its  major  axis.  Show 
plan  and  elevation  of  the  surface  when  the  axis  of  revolution 
is  vertical,  and  find  the  projections  of  a  point,  P,  on  it. 

The  plan  of  the  surface  will  be  simply  a  circle  of  2  in. 
diameter,  and  its  elevation,  an  ellipse  of  ^he  dimensions 
given  in  the  question. 

To  find  pp',  the  projections  of  the  point  P.  If  the  plan 
p  be  given,  assume  a  meridian  section  to  be  made  through 
F  and  to  be  turned  round,  with  the  point  I*  on  it,  till  paral- 
lel to  the  vertical  plane.  The  height  of  F  may  then  be 
found  by  drawing  a  projecting  line  from  the  new  position  of 
p  in  plan  to  cut  the  ellipse  which  is  the  elevation  of  the 
surface.  Either  of  the  two  points  in  which,  in  general,  the 
projecting  line  meets  the  ellipse,  may  be  the  height  of  /*, 
according  as  we  assume  the  point  to  be  on  the  upper  or 
lower  part  of  the  surface. 

If/'  be  given,  p  may  be  found  by  taking  a  horizontal 
section  through  F,  the  plan  of  which  section  will  be  a  circle 


SURFACES  OF  REVOLUTION:  HZ 

passing  through  the  plan  /.  This  point  may  then  be  readily 
determined  by  drawing  a  projecting  line  from  /'  to  meet  the 
circle. 

2,  To  determine  a  cone  enveloping  the  spheroid  in  the  last 
problem  f  when  the  vertex  is  a  point  in  the  produced  axis  of  the 
surface. 

Let  v'  be  the  elevation  of  the  vertex.  Draw  from  v'  a 
tangent  line  to  the  ellipse  which  is  the  elevation  of  the 
spheroid  and  call  the  point  of  contact  f.  The  line  v't'  is 
the  elevation  of  a  generatrix  of  the  enveloping  cone,  and  its 
plan  will  be  a  line  drawn  from  v  parallel  to  xy.  The  circle 
of  contact  (the  directrix)  will  be  the  circle  in  which  a  hori- 
zontal section  plane  through  T  cuts  the  surface,  and  the 
horizontal  trace,  or  base  of  the  cone,  will  be  a  circle  passing 
through  the  H.T.  of  the  generatrix.  VT,  and  concentric 
with  the  circle  which  is  the  plan  of  the  spheroid. 

In  a  similar  manner,  any  number  of  enveloping  cones 
can  be  drawn  by  taking  other  points  in  the  axis  for  vertices. 

3.  Determine  the  spheroid  as  before,  and  assume  a  point, 
P,  in  space,  without  the  surface,  and  not  in  the  axis.  Find  the 
cone  which  has  P  for  its  vertex  and  erivelops  the  spheroid. 

(i)  Determine  any  cone,  as  in  the  last  problem,  envelop- 
ing the  spheroid,  and  having  its  vertex  in  the  axis  of 
revolution  produced. 

(2)  Draw  a  tangent-plane  to  this  cone  passing  through 
P.  This  will  also  be  a  tangent-plane  to  the  spheroid,  and 
the  point  in  which  the  circle  of  contact  of  the  cone  with 
the  spheroid  meets  the  line  of  contact  of  the  tangent-plane 
with  the  same  cone,  will  be  the  point  of  contact  of  the 
tangent-plane  with  the  spheroid.     Two  tangent-planes  being 


14^  SOLID  OR  DESCRIPTIVE  GEOMETRY. 

possible  to  the  enveloping  cone,  there  will,  of  course,  be  two 
such  points. 

(3)  Through  these  points  of  contact  draw  two  lines  to 
the  point  P.  Since  these  lines  lie  in  tangent-planes  to  the 
spheroid,  and  contain  the  points  of  contact  of  the  tangent- 
planes,  they  are  tangents  to  its  surface,  and,  inasmuch  as  they 
pass  through  P,  they  are  generatrices  of  the  required  en- 
veloping cone. 

Similarly,  by  taking  a  series  of  other  points  in  the  pro- 
duced axis  for  the  vertices  of  new  cones,  any  required 
number  of  pairs  of  generatrices  may  be  found,  and  their 
horizontal  traces  joined  by  a  curve  for  the  H.T.  of  the 
enveloping  cone.  The  curve  joining  the  points  of  contact 
of  the  tangent-planes  will  give  the  curve  of  contact  of  the., 
enveloping  cone  with  the  spheroid- 

4.  Determine  a  cylinder  enveloping  the  spheroid  in  Pro- 
blem I,  when  the  generatrices  are  parallel  to  a  given  line  inclined 
at  60"  to  the  horizojital  plane,  and  itt  plan  making  an  angle 
of  2)0°  with  xy. 

(i)  Envelop  the  spheroid  by  any  cone,  as  in  Problem 
2,  having  its  vertex  in  the  produced  axis, 

(2)  Draw  a  tangent-plane  to  this  cone  parallel  to  the 
given  line.  This  will  also  be  a  tangent-plane  to  the  spheroid, 
and  its  point  of  contact  may  be  found  as  in  the  last 
problem.  Here  too,  since  there  are  two  tangent-planes, 
there  will  be  a  pair  of  points  of  contact. 

(3)  Through  this  pair  of  points,  two  lines,  parallel  to 
the  given  line,  may  be  drawn  for  a  pair  of  generatrices  of 
the  enveloping  cylinder. 


1 


SURFACES  OF  REVOLUTION:  149 

Similarly,  any  number  of  pairs  of  generatrices  can  be 
found,  and  the  enveloping  cylinder,  with  its  curve  of  contact, 
completely  determined. 

Note.  The  last  two  problems  might  be  worked  by  the 
aid  of  auxiUary  sections  of  the  surface  by  planes  which  pass, 
in  Prob.  3,  through  the  given  point  F,  and,  in  Prob.  4,  by 
planes  taken  parallel  to  the  given  line.  Tangent  lines  to  the 
sections  give  pairs  of  generatrices  of  the  enveloping  surfaces. 

5.  A  surface  is  generated  by  the  revolution  of  a  circle  of 
I  in.  diameter  about  a  fixed  straight  line  in  the  plane  of  the 
circle  and  1*5  in.  from  its  centre.  Detennine  a  poifit,  P,  on  its 
surface f  and  the  enveloping  cones  and  cylinder  as  in  Probs.  i, 
2,3,  and  4  above. 

This  surface,  which  is  an  anchor  ring,  or  annulus  of  a 
circular  section,  will  afford  the  student  an  additional  exercise 
in  the  application  of  the  methods  used  in  the  solution 
of  the  problems  mentioned,  without  presenting  any  special 
features  of  difficulty. 

Tangen'I*-Planes  and  Normals  to  Surfaces  of 
Revolution. 

After  what  has  been  already  given,  it  will  be"  only 
necessary  to  speak  of  the  following  problems  in  the  most 
general  terms. 

1.  To  determine  a  tangent-plane  to  a  surface  of  revolution 
at  a  given  point  on  it. 

Draw  the  normal  (Theorem  IV.  Cor.  2,  above)  and  the 
tangent-plane  at  right  angles  to  it,  through  the  given  point. 

2.  To  determine' a  tangent-plane  to  a  surface  of  revolution 
that  shall  coiitain  a  given  straight  line  or  its  equivalent^  tivo 
points  in  spaa. . 


ISO  SOLID  OR  DESCRIPTIVE   GEOMETRY. 

The  student  on  referring  to  the  parallel  problem,  "  Tan- 
gent-Planes to  Spheres,"  will  not  be  at  a  loss  for  several 
possible  solutions,  the  easiest  of  which  is  to  envelop  the 
surface  by  a  cylinder  with  generatrices  parallel  to  the  given 
line.  A  tangent-plane  to  this  cyhnder  through  any  point 
in  the  line  will  satisfy  the  conditions. 

3.  Determine  a  tangent-plane  to  a  surface  of  rez>olution 
that  shall  make  an  angle  6°  with  H.P.  and  contain  a  given 
point,  P,  in  space. 

Make  P  the  vertex  of  a  cone  enveloping  the  surface  of 
revolution,  and  determine  a  tangent-plane  to  this  cone 
that  shall  make  an  angle  of  ff'  with  H.P.  This  will  be 
the  plane  required. 

4.  To  draw  a  normal  to  a  surface  of  revolution  parallel 
to  a  given  straight  line. 

Determine  a  meridian  section  of  the  surface  by  a  plane 
parallel  to  the  given  straight  line,  and  a  tangent  line  to  the 
curve  at  right  angles  to  the  projection  of  this  line  upon  the 
plane.  A  perpendicular  to  the  tangent  line,  through  the 
point  of  contact,  will  be  the  required  normal. 

5.  To  draw  a  tangent-plane  to  a  given  surface  of  revo- 
lution parallel  to  a  given  plane. 

Draw  a  perpendicular  to  the  given  plane  and  a  normal 
parallel  to  this  line.  The  required  tangent-plane  may  then 
be  determined  at  right  angles  to  the  normal  through  the 
point  in  which  it  meets  the  surface. 

The  Hyperboloid  of  Revolution  of  one  Sheet. 

This  surface  is  a  particular  case  of  a  surface  of  greater 
generality  known  as  the  Hyperboloid  of  one  Sheet.  It  is 
an  undevelopable  Ruled  surface  admitting  of  several  modes 


HYPERBOL  01  D  OF  RE  VOL  UTION.  1 5  I 

of  generation,  one  of  which  is,  by  the  revolution   of  an 
hyperbola  about  its  conjugate  axis,  whence  the  name. 

It  will  be  treated  here  as  a  Twisted  Surface  of  Revolution, 
generated  by  a  straight  line  revolving  about  an  axis  not 
in  the  same  plane  with  it;  and  the  conjugate  hyperbolas, 
which  form  the  meridian  section  of  the  surface,  will  be 
determined  as  a  type  of  the  method  by  which  any  line 
whatever,  given  as  a  generatrix,  may  be  reduced  to  an 
equivalent  generator  in  a  meridian  plane. 

I.  A  straight  line  inclined  at  50"  to  the  horizontal  plane 
is  the  generatrix  of  a  surface  of  revolution,  the  axis  of  which  is 
vertical.  The  nearest  point  in  the  generatrix  is  '5  in.  from  the 
axis.  Draw  the  elevation  of  a  meridian  section  on  a  parallel 
vertical  plane. 

Since  every  point  in  the  generatrix  describes  a  circle 
about  the  axis  in  a  plane  perpendicular  to  it,  if  a  plane  be 
taken  through  any  point,  P,  in  the  generatrix  perpendicular  to 
the  axis,  meeting  the  latter  in  point  A,  and  intersecting  the 
assumed  meridian  plane  in  a  straight  line  through  A,  the 
distance  AP  can  be  cut  off  on  this  line  from  A,  both  sides 
of  the  axis,  for  two  points,  /"  and  P^,  in  the  required 
meridian  section.     Similarly  other  points  may  be  found. 

To  draw  the  problem,  begin  by  describing  a  circle  of 
•5  in.  radius  in  the  horizontal  plane  for  the  plan  of  the  throat, 
or  collar,  of  the  hyperboloid,  and  show  the  straight  line, 
which  is  its  elevation,  two  or  three  inches  above  xy. 

Draw  the  given  generatrix  parallel  to  the  vertical  plane. 
In  plan  this  will  be  a  straight  line  touching  the  circle  and 
parallel  to  xy.  In  elevation  it  will  be  a  straight  line  drawn 
through  the  middle  point  of  the  elevation  of  the  collar,  and 
making  an  angle  of  50°  with  xy. 


152  SOLID   OR  DESCRIPTIVE   GEOMETRY. 

Let  0  be  the  centre  of  the  circle  which  is  the  plan  of 
the  collar,  then  o  is  also  the  plan  of  the  axis.  Determine 
the  projections  pp'  of  any  point  P  in  the  generatrix.  As- 
sume a  horizontal  plane  passing  through  P  to  cut  the 
surface ;  the  plan  of  the  section  will  be  a  circle  of  centre  o 
and  radius  op ;  the  elevation,  a  straight  line  parallel  to  xy 
through/'.  The  extremities  of  this  elevation  are  two  points 
in  the  required  elevation  of  the  meridian  section.  Others 
may  be  found  in  a  similar  manner,  and  the  curve  put  in  by 
hand. 

Observations.  The  student  should  note  well  the  fol- 
lowing facts :— ^ 

(i)  The  plan  of  the  generatrix  is,  in  every  position, 
a  tangent  to  the  collar. 

(2)  The  surface  has  two  systems  of  generatrices  inclined 
at  equal  angles  in  opposite  directions.  Hence  two  straight 
lines,  coinciding  with  the  surface  throughout  their  whole 
length,  may  be  drawn  through  any  point  on  it,  and,  there- 
fore, two  generatrices  of  different  systems  are  always  in  the 
same  plane. 

(3)  No  two  generatrices  belonging  to  the  same  system 
are  in  one  plane,  and  no  three  can  be  parallel  to  the  same 
plane.     Hence  the  surface  cannot  be  developed. 

(4)  If  any  two  generatrices  of  the  same  system  be 
determined,  and  a  third  belonging  to  the  other  system  be 
drawn  from  a  point  in  one  of  them  and  produced,  it  will 
meet  the  other.  Similarly,  if  three  such  generatrices  be 
determined,  and  a  fourth  belonging  to  another  system  be 
drawn  through  any  point  in  one  of  them,  it  will,  if  produced, 
meet  the  other  two.    Hence^  we  see  that  if  three  generatrices 


HYPERBOLOID   OF  REVOLUTION.  153 

of  the  surface  be  assumed  as  directrices,  it  might  be  generated 
by  a  fourth  straight  line  which  moves  so  as  ahvays  to  pass 
through  them.  That  is,  the  Hyperboloid  of  Revolution  is  a 
particular  case  of  a  surface  called  the  Hyperboloid  of  One 
Sheet,  which  may  be  defined  as  the  surface  generated  by  a 
straight  line  which  moves  so  as  always  to  meet  three  straight 
lines  not  in  the  same  plane. 

(5)  The  locus  of  all  lines  drawn  through  the  centre  of 
the  collar  parallel  to  the  generatrices  is  a  right  conic  sur- 
face called  the  asymptotic  cone.  A  section  plane  passing 
through  the  vertex  of  this  cone  and  not  at  right  angles 
to  the  axis,  cuts  the  Hyperboloid  in  an  ellipse,  parabola,  or 
hyperbola,  according  as  it  meets  the  cone  in  a  point,  a 
straight  line,  or  two  straight  lines.  Parallel  sections  are 
similar  curves. 

Note.  If  the  hyperbolas  which  form  the  meridian 
section  of  this  surface  were  rotated  about  the  transverse 
axis,  two  distinct  surfaces  would  be  produced— the  Hyper- 
boloid of  Revolution  of  Two  Sheets.  This  however  is  not 
a  Ruled  surface. 

2.  Given  one  projection  (p  or  p')  of  a  point,  P,  on  the 
surface  of  the  hyperboloid  of  revolution  in  the  last  problem,  to 
find  the  other  projection. 

If  p  be  given,  draw  a  tangent  to  the  plan  of  the  collar 
through/.  This  will  be  the  plan  of  a  generatrix.  Find  its 
elevation  and  the  point/'  in  it. 

If  /'  be  given,  a  horizontal  section  of  the  surface 
through  P,  as  in  the  parallel  problem  on  the  sphere,  will 
determine/. 

3.  To  draw  a  tangent-plane  to  the  surface  at  a  point,  P, 
on  it.  - 


154  SOLID   OR  DESCRIPTIVE   GEOMETRY. 

Draw  the  two  generatrices  through  P.  The  plane  con- 
taining these  will  be  the  plane  required. 

Note.  This  is  a  tangent-plane  at  a  point  only,  and  not 
along  a  generatrix. 

Undevelopable  Ruled  Surfaces  Generally. 

The  Hyperboloid  of  One  Sheet  has  been  already  defined 
as  the  surface  produced  by  a  straight  Une  which  moves 
so  as  always  to  meet  three  given  straight  lines  not  in  the 
same  plane. 

To  determine  the  surface,  then,  it  is  only  necessary  to 
draw  a  number  of  straight  lines  from  different  points  in  one 
of  the  given  rectilineal  directrices  to  meet  the  other  two. 
(Prob.  XII.,  Cor.  2,  Chap,  in.) 

Theorem.  If  any  three  generatrices  be  found  by  this 
construction,  a  straight  line  which  moves  so  as  always 
to  pass  through  these  three  lines  will  generate  the  same 
surface. 

That  is,  the  Hyperboloid  of  One  Sheet  has  two  distinct 
systems  of  rectilineal  generatrices,  a  fact  which  was  pointed 
out  when  treating  of  the  particular  case  of  the  Ruled  Hyper- 
boloid of  Revolution. 

Every  plane  section  of  the  surface  is  a  conic,  and  it  is  by 
a  comparison  of  the  character  of  the  sections  made  by  planes 
cutting  the  surface  in  particular  directions  that  the  student 
will  best  get  a  mental  picture  of  its  form.  Turning  to  the 
Hyperboloid  of  Revolution  of  One  Sheet,  it  is  seen  that  the 
sections  by  planes  at  right  angles  to  the  axis  are  circles. 
Now,  if  these  sections  were  ellipses,  the  surface  would  be  the 
general  case  of  the  Hyperboloid  of  One  Sheet  we  are  here 
considering,  and,  from  this  point  of  view,  might  be  called  the 


RULED  SURFACES  OF  REVOLUTION.  155 

Elliptic-Hyperboloid.  Several  other  modes  of  generation  are 
obvious,  such  as  by  the  motion  of  a  variable  ellipse,  or  a 
variable  hyperbola,  under  particular  restrictions,  and  by  the 
motion  of  a  straight  line,  always  touching  the  elliptic  collar, 
in  a  plane  perpendicular  to  the  plane  of  the  collar  and  in- 
clined at  a  constant  angle  to  the  same  plane. 

A  number  of  interesting  problems  might  be  discussed  on 
surfaces  generated  when  the  three  directrices  are  not  recti- 
lineal, but  the  limits  of  this  book  exclude  them. 

Another  group  of  surfaces,  generated  by  a  straight  line 
which  moves  parallel  to  a  given  plane  and  always  passes 
through  two  given  lines  not  in  one  plane,  may  just  be 
mentioned. 

When  both  the  directrices  are  straight  lines,  a  surface 
called  the  Hyperbolic-Paraboloid  is  produced. 

Like  the  Hyperboloid  of  One  Sheet,  this  surface  can  be 
generated  by  the  motion  of  a  straight  line  in  two  ways. 
Thus,  the  surface  may  be  generated  by  a  straight  line 
moving  along  two  generatrices '  for  new  directrices,  and 
always  parallel  to  the  plane  to  which  the  two  original 
directrices  are  parallel. 

To  picture  the  form  of  this  surface  in  the  mind,  the 
student  should  turn  once  more  to  the  Hyperboloid  of 
Revolution  of  One  Sheet.  Let  /*  be  a  point  on  the  collar, 
and  assume  that  the  meridian  section  through  /*  is  a 
parabola,  instead  of  an  hyperbola,  of  which  P  is  the  vertex. 
If  now  the  section  by  the  plane  of  the  collar  be  also  a 
parabola,  with  P  for  its  vertex,  instead  of  a  circle,  and 
the  first  parabola  move  with  its  vertex  along  the  curve 
of  the  second  in  a  plane  always  parallel  to  itself,  it  will 
generate  the  surface  in  question. 


156  SOLID  OR  DESCRIPTIVE   GEOMETRY. 

Since  through  every  point  in  these  two  surfaces  two 
rectilineal  generatrices  can  be  drawn  a  tangent-plane  at 
a  point  on  either  surface  is  easily  determinable. 

When  one  directrix  is  a  straight  line  and  the  other  a 
curve,  the  surface  generated  is  called  a  Conoid. 

The  only  case  that  can  be  touched  upon  here  is  that 
in  which  the  axis  of  a  right  cylinder  and  a  helix  traced 
on  its  surface  are  the  directrices,  and  the  plane  of  the 
circular  base  that  to  which  the  generatrix  is  always  parallel. 
This  is  the  well-known  surface  of  the 

Square- Threaded  Screw. 

I.  Draw  two  helices,  '5  in.  apart  and  i  '5  in.  pitch,  on  the 
surface  of  a  right  cylinder  2  in.  diameter  and  vertical  axis.  A 
rectangle,  '^in.  by  'T$in.,  moves  between  these  helices  in  a 
vertical  plane  which  always  passes  through  the  axis,  thus 
generating  a  square-threaded  screw.  Show  several  turns  of 
the  thread  and  a  tangent-plane  at  a  point  on  the  screw 
surface. 

Noting  that  the  two  remote  angles  of  the  generating 
rectangle  trace  helices  of  the  same  pitch,  and  same  dis- 
tance apart,  as  those  given,  on  the  surface  of  a  right 
cylinder  of  radius  175  in.  having  the  same  axis  as  the 
given  cylinder,  these  helices  may  be  drawn  as  in  Prob.  7 
on  "The  Cylinder"  and  the  drawing  completed  to  show  the 
screw  and  a  sectional  elevation  of  a  nut  with  the  same 
thread. 

To  work  the  tangent-plane :  Let  P  be  the  point  taken 
on  the  helix,  through  P  draw  the  rectilineal  generatrix 
and  a  tangent  line  to  the  helix  at  that  point.  The  plane 
containing  the  generatrix  and  tangent  line  is  the  required 
tangent-plane  to  the  surface  .at  P. 


RULED  SURFACES   OF  REVOLUTION.  157 

Before  dismissing  the  subject  of  curved  surfaces,  there  is 
one  other  Ruled  Surface  of  Revolution  of  importance  in  the 
arts  which  may  be  left  to  the  ingenuity  of  the  student  to 
work  out  for  himself. 

2.  An  angular-threaded  screw  is  generated  by  an  isosceles 
triangle  whose  vertex  is  constrained  to  inove  in  a  helix  while 
its  base  moves  along  the  axis  of  the  right  cylinder  on  which  the 
helix  is  traced.  Show  the  screw  and  nut,  and  determine  a 
tangent-plane  to  the  surface  at  a  given  point  on  it. 

Two  examples  of  this  surface  will  be  found  in  Bradley's 
Elements  of  Geometrical  Drawing,  Part  II.  Plate  xxxvii. 
Figs.  I  and  2. 

The  student  who  wishes  for  other  exercises  can  deter- 
mine tapering,  square,  and  V-threaded  screws  on  a  cone. 


VIIL 

SECTIONS  BY  OBLIQUE  PLANES. 

Preliminary, 

An  Oblique  Plane  may  be  given  in  various  ways  of 
which  the  following  are  some  of  the  most  important : — 

1.  By  the  angles  a"  and  ^  which  the  vertical  and 
horizontal  traces  respectively  make  with  xy.   Fig.  29. 

2.  By  the  angles  6"  and  <^"  which  the  plane  makes  with 
the  horizontal  and  vertical  planes  of  projection  respectively. 
Prob.  VIL  Chap.  iir. 

3.  By  the  angles  a"  and  ^'. 

4.  By  the  angles  a"  and  <^". 

5.  By  the  angles  ^  and  ^. 

6.  By  the  angles  /S"  and  i^\ 

7.  By  three  given  points,  or  their  equivalents,  as;  (i) 
Two  straight  lines  parallel  or  meeting,  and  (2)  One  straight 
line  and  a  point.     Prob.  I,  (3),  Cor.  I.  Chap.  III.,  p.  46. 

The  student  should  solve  each  of  these  cases  before 
going  on  with  the  work  of  this  Chapter. 

As  an  example,  3  and  $  (which  are  also  types  of  4  and  6) 
are  solved  in  Fig.  29. 


SECTIONS  BY  OBLIQUE  PLANES. 
Fig.  ag. 


159 


v^> 


it* 


(^* 


'V 


A  point  ^  {aa')  in  the  vertical  plane  is  made  the  vertex 
of  a  right  cone  standing  on  its  base,  cef,  in  H.P.,  with  its 
generatrices  inclined  at  6°.  For  3,  draw  ct  tangent  to  cefaX 
c,  meeting  xy  at  t,  and  making  the  angle  xtc  equal  to  a";  join 
a't.  Then  ct  is  the  horizontal  trace,  and  td  the  vertical  trace, 
of  the  required  plane.  For  5,  draw  a't  making  the  angle 
dtx  equal  to  jS",  and  from  t  draw  tc  touching  the  circle  cef. 
This  determines  the  plane  as  before. 

Nos.  4  and  6  are  worked  similarly,  the  axis  of  the  cone, 
however,  being  in  the  H.P.  instead  of  the  V.P, 


l60  SOLID   OR  DESCRIPTIVE   GEOMETRY. 

General  Remarks  on  the  Method  of  Working 
Sections  by  Oblique  JPlanes. 

When  the  section  is  of  a  form  bounded  by  plane  faces, 
the  general  solution  is  effected  by  finding  the  intersection  of 
the  given  section  plane  with  the  indefinite  planes  of  each  of 
the  faces.  The  parts  of  the  indefinite  intersections  that  are 
common  to  the  faces  and  the  section  plane,  are  lines  of  the 
section  required.  Sometimes  the  work  is  simplified  by 
finding  the  points  in  which  the  edges  of  the  form  pierce  the 
section  plane,  by  the  aid  of  auxiliary  planes  containing  these 
edges. 

When  the  section'  is  of  a  curved  surface,  the  general 
method  is  to  assume  some  mode  of  generation  of  the 
surface  and  construct  the  intersection  of  each  generatrix 
with  the  plane. 

Here  also  recourse  is  frequently  had  to  auxiliary  planes 
cutting  the  surface  in  generatrices  and  the  section  plane  in 
straight  lines.  Sometimes,  however,  the  auxiliary  planes  are 
made  to  cut  the  surface  in  some  easily  determinable  curve 
other  than  a  generatrix. 

When  convenient,  auxiliary  planes  should  be  taken 
parallel  or  perpendicular  to  one  of  the  planes  of  projection, 
as  in  these  cases  one  projection  of  the  auxiliary  section  is 
always  a  straight  line. 

For  the  most  part  the  practical  solution  of  the  problems 
in  this  chapter  is  greatly  facilitated  by  assuming  an  auxiliary 
plane  of  projection  at  right  angles  to  one  of  the  traces 
of  the  section  plane.  The  projection  of  the  section  on  this 
auxiliary  plane  being  a  straight  line,  the  other  projections, 
true  shape  of  section,  and  projection  of  frustum   on   the 


.     SECTIONS  BY  OBLIQUE  PLANES.  l6l 

plane  of  section,  may  be  easily  deduced.  The  simplest 
practical  solution  of  a  problem  is,  however,  not  necessarily 
the  one  most  meet  for  the  purposes  of  the  student,  whose 
aim  should  always  be  to  develop  a  grasp  of  the  method 
and  principles  of  the  subject,  and  a  knowledge  of  the 
properties  of  Form  on  which  they  are  based. 

Problems. 

I.  A  right  prism  standing  on  a  square  base  of  2  in.  side, 
one  side  of  the  square  making  an  angle  of  T^d^  with  xy,  is  cut  by 
an  oblique  plane,  inclined  at  50",  which  passes  through  a  point 
in  the  axis  of  the pris7n  2  in.  above  the  base,  and  has  for  its 
horizontal  trace  a  line  inclined  to  xy  at  an  angle  of  40". 
Show  the  projections  of  the  frustum,  the  true  shape  of  the 
section,  the  development,  and  a  new  projection  of  the  frustum 
on  the  plane  of  section. 

Determine  plan  and  elevation  of  the  prism  and  the 
traces  of  the  section  plane. 

To  work  the  projection  of  the  section. 

First  Method: — Assume  auxiliary  vertical  planes  con- 
taining the  edges  of  the  prism  and  cutting  the  given  plane. 
The  lines  in  which  the  assumed  vertical  planes  cut  the  given 
section  plane,  will  meet  the  edges  through  which  the  auxiliary 
vertical  planes  are  taken  in  required  points  of  the  section. 

If  the  horizontal  traces  of  these  auxiliary  vertical  planes 
be  taken  parallel  to  the  horizontal  trace  of  the  given  oblique 
section  plane,  the  lines  of  intersection  will  be  horizontal — 
their  plans  coinciding  with  the  horizontal  traces  of  the 
auxiliary  planes,  and  their  elevations  being  lines,  parallel  to 
xy,  drawn  from  the  points  in  which  the  vertical  traces  of 
these  planes  cut  the  V.T.  of  the  given  section  plane. 

E.  G.  II 


1 62  SOLID   OR  DESCRIPTIVE   GEOMETRY. 

The  auxiliary  planes  may  however  be  taken  so  as  to 
contain  two  edges  and  thus  find  two  points  in  the  section 
simultaneously,  a  particular  case  of  which  is  the  following : — 

Second  Method: — Determine  the  intersection  of  the  plane 
of  each  of  the  faces  with  the  given  plane  of  section.  The 
elevations  of  these  intersections  will  cut  the  elevations  of 
the  edges  of  the  prism  in  the  required  points  of  section,  and 
the  segments  of  the  lines  of  intersection  between  the  points 
of  section  will  be  the  sides  of  the  section  by  the  given  plane. 

Third  Method: — Take  an  auxiliary  vertical  plane  of 
projection  at  right  angles  to  the  horizontal  trace  of  the  given 
plane  of  section.  The  projection  of  the  section  on  this  plane 
will  lie  in  the  vertical  trace  of  the  section  plane  on  the  same 
plane.  Hence,  the  heights  may  be  determined  iil  this 
auxiliary  elevation  and  transferred. 

To  find  the  true  shape  of  the  section  by  rabatting  the 
plane  of  section  into  one  of  the  planes  of  projection. 

Let  -f  be  a  point  in  the  section  of  which/,/'  are  the  pro- 
jections. Through  the  plan,  /,  draw/w  perpendicular  to  the 
horizontal  trace  and  meeting  it  in  m.  Measure  the  hypo- 
tenuse of  the  right-angled  triangle  which  has  pm  for  its 
base,  and//*,  the  height  of  P^  for  its  perpendicular,  along 
mp  produced.  This  will  give  the  point  P  rabatted  into  the 
horizontal  plane  about  the  H.T.  of  the  section  plane. 

If  the  other  points  of  the  section  be  similarly  rabatted, 
they  may  be  joined  up  for  the  true  shape  of  the  section. 

Note.  If  the  auxiliary  vertical  plane  of  projection,  given 
in  the  third  method  above,  be  used,  the  hypotenuses  of  these 
right-angled  triangles  may  be  measured  at  once  from  the 
auxiliary  vertical  trace.  For  this,  and  the  development  of 
the  frustum,  refer  to  similar  work  in  Chap.  i. 


SECTIONS  BY  OBLIQUE  PLANES.  163 

To  draw  the  projection  of  the  frustum  on  the  plane  of 
section. 

Let  F,  Q,  R,  S,  be  the  points  in  which  the  vertical 
edges  of  the  prism,  drawn  from  the  points  A,  B,  C,  B,  of 
the  base  respectively,  meet  the  section  plane,  i.e.  the  points 
of  the  section. 

The  feet  T,  U,  V,  W,  of  the  perpendiculars  to  the 
plane  of  section  from  the  points  A,  B,  C,  D,  will  determine 
the  projection  of  the  base  of  the  frustum  on  the  plane. 
Lines  joining  the  feet  of  the  perpendiculars  with  the  cor- 
responding points  of  the  section  and  with  one  another  will 
complete  the  projection  of  the  frustum  on  the  plane  of 
section,  and  the  latter  must  now  be  rabatted  about  one  of 
its  traces  to  show  the  projection. 

For  example,  to  rabat  the  plane  about  its  vertical  trace, 
draw  from  /',  and  other  points  g,  r,  s',  of  the  elevation  of 
the  section,  perpendiculars  to  that  trace,  and  mark  off  from 
it,  on  these  perpendiculars,  the  actual  distances  of  the  points 
F,  Q,  jR,  S,  from  the  vertical  trace,  i.e.  the  hypotenuses 
of  the  right-angled  triangles  of  which  perpendiculars  from 
/',  q',  r,  /,  to  the  vertical  trace,  and  from  /,  q,  r,  s,  to  xy, 
are  the  sides. 

Join  these  in  order  for  the  rabatted  section. 

Then  proceed  in  the  same  manner  for  the  rabatment  of 
the  points  ll,  U,  V,  W,  which  determine  the  projection 
of  the  base  A,  B,  C,  D,  upon  the  plane.  The  rabatted 
points  T,  U,  V,  W,  may  then  be  joined  with  F,  Q,  F,  S, 
respectively,  for  the  complete  projection  of  the  frustum. 

Note.  When  one  point,  as  T,  in  the  rabatted  projection 
of  the  base  upon  the  plane  of  section  is  found,  work  may  be 
saved  by  joining  TF,  and  drawing  the  projections  of  the 
other  edges  from  the  points  Q,  F,  S,  parallel  to  TF  to  meet 

II — 2 


164  SOLID   OR  DESCRIPTIVE  GEOMETRY. 

the  perpendiculars  from  b\  ^,  d,  to  the  vertical  trace  in  the 
points  U,  V,  W. 

Another  Method : — If  the  auxiliary  vertical  trace,  in  the 
third  method  of  working  the  section,  be  treated  as  a  new  xy, 
the  plan  of  the  frustum  on  the  plane  of  section  can  be 
deduced  without  any  difficulty.  The  student  should  work 
the  problem  both  ways. 

2.  A  solid  is  formed  by  truncating  a  right  prism,  which 
stands  upon  a  regular  pentagonal  base  of  I'e^  in.  side  with  the 
nearest  face  of  the  prism  parallel  to  the  vertical  plane,  by  two 
oblique  planes  inclined  in  opposite  directions  on  either  side ; 
their  horizontal  traces  equally  inclined  to  xy  at  angles  of 
40",  equally  distant  175  in.  from  the  centre  of  the  plan  of  the 
base,  and  the  line  in  which  they  intersect  inclined  at  60°  and  in 
the  same  plane  with  the  axis  of  the  prism. 

Draw  the  plan  of  the  prism  and  from  the  centre  describe 
a  circle  of  175  in.  radius.  Two  tangents  to  this  circle 
making  angles  of  40"  with  xy  will  be  the  horizontal  traces  of 
the  oblique  planes;  and  a  line  from  the  point  in  which 
these  horizontal  traces  intersect,  through  the  axis  and 
inclined  at  the  given  angle  60°,  will  be  the  common  section 
of  the  two  given  planes,  the  vertical  traces  of  which  can  be 
drawn  through  the  vertical  trace  of  this  line.,  ■ 

The  rest  of  the  work  is  the  same  as  in  the  last  problem. 

Develop  the  form  and  determine  its  projection  on  one  of 
the  planes  of  section.  Work  also  a  new  plan,  the  xy  for 
which  is  to  be  taken  at  30"  with  the  elevation  of  the  axis. 

3.  A  square,  2  in.  side,  in  the  horizontal  plane,  one  side 
inclined  to  xy  at  30",  is  t/ie  base  of  an  oblique  prism,  the 
long  edges  of  which  are  inclined  at  60"  and  in  plan  parallel  td 


SECTIONS  BY  OBLIQUE   PLANES.  1 6$ 

a  diagonal  of  the  square.  Distance  between  the  planes  of 
the  bases  3*5  in.  Determine  the  section  made  by  an  oblique 
plane  inclined  in  the  same  direction  as  the  edges  of  the  prism 
at  an  angle  of  ^o"  and  passing  through  a  point  assumed  in 
the  upper  base ;  the  horizontal  trace  to  make  an  angle  of  ^f^ 
with  xy. 

Draw  the  projection  of  the  prism  and  make  the  assumed 
point  in  the  upper  base  the  vertex  of  a  right  cone  with 
generatrix  incUned  at  30".  A  tangent  line  to  the  circular 
base  of  this  cone  and  ma,king  an  angle  of  50°  with  xy  can  be 
drawn  for  the  horizontal  trace  of  the  section  plane.  Find 
the  vertical  trace,  and  work  the  section  as  in  the  other 
problems. 

Note.  The  method  of  working  by  means  of  the  auxiliary 
vertical  plane  of  projection  will  be  found  most  useful. 

Show  true  shape  of  section  and  plan  of  frustum  on  plane 
of  section,  and  work  the  development. 

4.  A  right  cylinder  2  in.  diameter,  axis  4  in.,  stands  on 
its  base  and  is  cut  by  an  oblique  plane  passing  through  the 
middle  point  of  the  axis  and  inclined  at  angles  of  60"  and  50" 
to  the  vertical  and  horizontal  planes  respectively.  Show  pro- 
jections and  true  shape  of  section  ;  also  the  projection  of  the 
frustum  on  the  plane  of  section  and  its  development. 

Find  the  oblique  plane  by  Probs.  7  and  16,  Chap.  in. 

Take  any  convenient  number  of  generatrices  of  the 
cylinder,  and  find  the  points  in  which  they  meet  the  section 
plane,  as  was  done  in  the  case  of  the  edges  of  the  prism, 
Prob.  I,  and  join  them  in  the  elevation  by  a  curve,  which  in 
this  case  will  be  an  ellipse.  In  plan  the  base  of  the  cyHnder 
and  the  projection  of  the  section  will  coincide. 


l66  SOLID  OR  DESCRIPTIVE  GEOMETRY. 

For  the  projection  of  the  frustum  use  the  auxiliary 
vertical  plane  of  projection  as  before,  and  develop  as  in 
Chap.  I. 

Note.  The  plan  of  the  section  of  any  right  prism 
standing  on  its  base,  or  cylindrical  surface  with  generatrices 
perpendicular  to  the  horizontal  plane,  by  an  inclined  or 
oblique  plane,  coincides  with  the  trace  of  the  form  on  the 
horizontal  plane  of  projection.  Hence,  any  given  figure 
whatever  may  be  the  plan  of  a  certain  figure  lying  in  a 
given  oblique  plane :  and  the  elevation  and  true  shape  of 
this  figure  may  be  deduced  in  the  same  way  as  a  section  of 
a  vertical  prism,  or  cylinder,  having  the  given  figure  for  its 
horizontal  trace,  would  be  worked  if  the  given  plane  were 
the  plane  of  section. 

5.  To  work  the  section  of  an  oblique  cylindrical  surface  by 
an  oblique  pla7te. 

In  the  particular  case  in  which  the  horizontal  trace 
of  the  cylinder  is  a  circle,  or  part  of  a*x:ircle,  a  series 
of  auxiliary  horizontal  planes  cutting  the  surface  in  similar 
circles,  and  the  section  plane  in  lines  parallel  to  its  horizontal 
trace,  would  be  most  convenient. 

In  the  general  case,  in  which  the  horizontal  trace  of  the 
cylinder  is  any  curved  line  whatever,  draw  a  number  of 
convenient  generatrices  and  determine  the  points  in  which 
these  generatrices  meet  the  section  plane.  An  auxiliary 
vertical  plane  of  projection  at  right  angles  to  the  horizontal 
trace  of  the  section  plane  facilitates  finding  the  required 
points  in  the  section. 

6.  Determine  a  regular  hexagonal  pyramid  of  ri^in. 
side  standing  on  its  base,  axis  4  in.  long,  and  one  side  of  the 
hexagon   making  an  angle  of  20°  with  xy.     Section  by  an 


SECTIONS  BY  OBLIQUE  PLANES.  167 

oblique  plane  passing  through  a  point  in  the  axis  i'5  in.  above 
the  base ;  t/ie  horizontal  and  vertical  traces  making  angles  of 
80"  and  40"  respectively  with  xy. 

A  line  from  the  given  point  in  which  the  section  plane 
meets  the  axis,  parallel  to  H.P.  and  inclined  at  80°  to  the 
V.P.,  will  determine  a  point  in  the  vertical  trace  of  the 
section  plane  from  which  the  traces  of  the  plane  may  be  at 
once  found. 

Determine  the  section  (i)  as  in  Prob.  i,  by  the  aid  of 
auxiliary  vertical  planes  containing  the  edges  of  the  pyramid; 
(2)  by  finding  the  intersections  of  the  given  oblique  plane 
with  the  indefinite  planes  of  the  faces  of  the  solid ;  or  (3)  by 
means  of  an  auxiliary  elevation  on  a  vertical  plane  taken  at 
right  angles  to  the  horizontal  trace  of  the  oblique  plane  of 
section. 

When  the  problem  is  worked  by  means  of  auxiliary 
vertical  planes  containing  the  slant  edges,  the  work  may  be 
shortened  if  it  be  noted  that  the  point  in  which  the  axis 
of  the  pyramid  meets  the  section  is  common  to  all  the 
intersections.  It  will  be,  therefore,  only  necessary  to  find  a 
second  point  in  each  intersection — such  as  that  in  which 
the  vertical  or  horizontal  trace  of  any  one  of  these  vertical 
planes  cuts  the  corresponding  trace  of  the  section  plane — 
to  determine  the  line  of  intersection  and,  therefore,  another 
point  in  the  required  section. 

Find  the  true  shape  of  the  section  by  rabatting  the  section 
plane  as  in  other  problems. 

To  draw  the  projection  of  the  frustum  on  t/ie  plane 
of  section. 

(i)    Work  for  the  points  in  the  section  as  in  Problem  i. 


1 68  SOLID   OR  DESCRIPTIVE  GEOMETRY. 

(2)  Treat  the  projection  of  the  vertex  on  the  plane  of  sec- 
tion as  the  points  T,  U,  &c.  were  treated  in  the  same  problem. 

(3)  The  lines  through  the  rabatted  projection  of  the 
vertex  on  the  section  plane  and  the  points  of  section  just 
determined,  will  be  the  indefinite  projections  of  the  slant 
edges  of  the  pyramid.  The  points  of  the  base  may  be 
fixed  by  perpendiculars  from  their  elevations  to  the  vertical 
trace  to  intersect  these  indefinite  projections  of  the  edges, 
and  when  found  may  be  joined  consecutively  to  complete 
the  required  projection  of  the  frustum. 

The  development  may  be  found  by  describing  a  circle 
with  radius  equal  to  a  slant  edge  of  the  pyramid,  setting 
along  the  circumference  the  edges  of  the  base  as  chords,  on 
one  of  which  the  square  for  the  base  must  be  described, 
and  joining  these  points  to  the  centre  of  the  circle.  The 
points  of  section  may  be  found  by  dividing  the  developed 
edges  in  the  same  proportion  as  the  corresponding  plans  or 
elevations  are  divided  by  the  projections  of  the  points  of 
section.     Fit  on  the  true  shape  of  the  section. 

7.  Assume  an  irregular  hexagonal  pyramid  and  cut  it  as 
in  the  last  problem  by  an  oblique  plane.  Show  the  true  shape 
of  the  section,  projection  of  frustum  on  plane  of  section,  and 
development. 

To  work  this  problem,  use  an  auxiliary  vertical  plane  of 
projection  at  right  angles  to  the  horizontal  trace  of  the 
given  oblique  plane. 

8.  A  right  cone,  axis  3  "5  in.,  diameter  of  base  2  in.,  stands 
upon  its  base  and  is  cut  by  an  oblique  plane  which  passes 
through  three  given  points  X,  Y,  Z,  on  its  stiff  ace.  Show 
true  shape  of  section,  projection  of  frustum  on  plane  of  section, 
and  development. 


SECTIONS  BY  OBLIQUE  PLANES.  1 69 

Find  the  traces  of  the  plane  which  contains  the  three 
given  points. 

To  work  the  section  assume  a  series  of  horizontal  auxili- 
ary planes  cutting  the  cone  and  the  plane  of  section.  The 
sections  of  the  cone  will  be  circles,  and  of  the  plane  straight 
lines,  parallel  to  the  horizontal  trace,  drawn  from  the  points 
in  which  the  vertical  trace  of  the  section  plane  meets  the 
vertical  traces  of  the  auxiliary  planes.  The  points  common 
to  the  circles  and  the  lines  of  section  will  be  required 
points  in  the  section  of  the  cone  by  the  given  plane. 

Complete  as  in  other  problems.  For  the  development 
see  Chap.  i. 

9.  To  work  the  section  by  a  given  oblique  plane  of  a  conical 
surface  whose  directrix  is  any  curve  whatever  and  vertex  a 
t>oint  anywhere  in  space. 

Find  a  convenient  number  of  generatrices  and  determine 
their  intersections  with  the  given  section  plane.  The  line 
drawn  through  these  points  will  be  the  section  required. 

10.  A  right  hexagonal  prism,  i'25  /;z.  side,  axis  4  in. 
^ong,  horizontal,  and  inclined  at  ^^'^  to  V.  P.;  lowest  edge  75 
'n.  above  H.  P.,  and  a  face  containing  it  inclined  at  20",  is  cut 
'iy  an  oblique  plane  which  passes  through  three  points  to  be 
xssumed  at  pleasure  on  any  three  edges  of  the  solid.  True 
hape  of  section,  projection  of  frustujn,  ^'C.  as  in  other  probletns. 

Use  either  vertical  or  horizontal  auxiliary  planes  con- 
aining  the  edges. 

11.  Take  a  right  cylinder,  axis  horizontal,  inclined  at  35" 
.  0  V.  P.,  instead  of  the  prism  in  the  last  problem,  and  work 
i  blique  section,  true  shape,  &'c. 


I/O  SOLID   OR  DESCRIPTIVE   GEOMETRY. 

Use  a  series  of  horizontal  auxiliary  planes  cutting  the 
cylinder  in  generatrices. 

12.  Take  a  right  pentagonal  pyramid,  side  of  base  i"5  in., 
axis  3*5  in.,  when  its  highest  face  is  horizontal  and  the  plan 
of  its  axis  inclined  at  40"  to  xy,  and  work  a  section  of  it  by  an 
oblique  plane. 

Use  auxiliary  vertical  planes  containing  the  edges,  except 
for  the  edges  of  the  upper  face,  in  which  case  a  horizontal 
auxiliary  plane  will  be  more  convenient. 

Note.  Since  all  the  vertical  planes  containing  the  long 
edges  of  the  pyramid  must  also  contain  a  perpendicular  to 
the  horizontal  plane  from  the  vertex  of  the  solid,  all  the 
intersections  of  these  planes  with  the  plane  of  section  must 
pass  through  the  point  in  which  the  perpendicular  in 
question  meets  this  section  plane.  Hence  this  point  may 
be  used  to  simplify  the  work,  just  as  the  intersection  of 
.  the  axis  was  used  in  Problem  6. 

13.  Work  section  by  an  oblique  plane  of  a  right  cone  with 
its  axis  horizontal  and  inclined  to  the  V.  P.  at  40°. 

Use  a  series  of  auxiliary  vertical  planes  cutting  the  cones 
in  generatrices,  and  observe  that  the  "Note"  to  the  last 
problem  applies  also  to  this.  Each  plane  will,  in  general, 
give  two  points. 

14.  Work  the  section  of  a  sphere  by  an  oblique  plane 
•which  passes  through  tivo  given  points  on  its  surface  and 
makes  an  angle  of  60"  with  If.  P, 

Determine  the  plane,  and  for  the  section  use  auxiliary 
planes  parallel  to  one  of  the  planes  of  projection. 

15.  Section  of  the  Hyperboloid  of  Ra^olution  of  one  Sheets 
given  in  the  last  Chapter,  by  an  oblique  plane. 


SECTIONS  BY  OBLIQUE  PLANES.  I/^ 

Use  horizontal  auxiliary  planes.  These  will  cut  the 
surface  of  revolution  in  circles  and  the  plane  of  section 
n  straight  Hnes — the  work  being  similar  in  every  respect  to 
:hat  of  Problem  8  above. 

Any  surface  of  revolution  with  its  axis  at  right  angles  to 
Dne  of  the  planes  of  projection  might  be  worked  in  the 
same  way;  e.g.  the  sphere  in  Prob.  14  above,  the  spheroid 
Prob.  I,  "Surfaces  of  Revolution,"  Chap,  vii.,  and  the 
"oUowing  : — 

16.  Section  of  the  annulus  or  anchor  ring.,  Prob.  5, 
^Surfaces  of  Revolution^'  Chapter  VII.,  by  a  tangentplane 
'vhich  passes  through  its  centre  and  makes  an  angle  of6Q°  with 
'he  vertical  plane  of  projection. 

1 7.  Section  by  an  oblique  plane  of  the  screw  surfaces  given 
it  the  end  of  Chap.   VII. 

Use   auxiliary  elevations   at  right  angles  to  the  H.  T. 
of  the   section  plane,  and  deduce  therefrom  the  required 
projections   of  the  points  in  which  the  generatrices   meet 
hat  plane. 

18.  To  find  the  section  by  an  oblique  plane  of  a  surface 
<  'f  revolution  with  its  axis  oblique  to  both  planes  of  projection 

ind  not  at  right  angles  to  the  oblique  plane. 

Use  auxiliary  planes  of  section  perpendicular  to  the 
ixis  of  the  surface  of  revolution.  These  cut  the  oblique 
)lane  in  straight  lines  and  the  surface  in  circles.  The 
)oints  common  to  these  lines  and  circles  are  points  in  the 
;urve  of  intersection. 

Note.  The  difficulty  of  projecting  the  surface  of  re- 
■olution  in  the  position  given  is  generally  sufficient  to  deter 


1/2  SOLID  OR  DESCRIPTIVE   GEOMETRY. 

any  but  the  most  courageous  of  students  from  attempting 
this  problem.  It  may,  however,  be  remarked  that  the 
projection  of  a  surface  of  revolution  on  a  plane  inclined  to 
its  axis,  is  the  section  by  that  plane  of  a  cylinder  enveloping 
the  surface  and  perpendicular  to  the  plane  of  projection. 
For  the  construction  of  a  cylinder  enveloping  a  surface  of 
revolution  see  Prob.  4,  "Problems  on  Surfaces  of  Revolu- 
tion," Chap,  VII. 


IX. 

INTERSECTIONS. 

General  Remarks. 

The  methods  by  which  the  intersection  of  two  surfaces 
i.re  determined  when  one  of  them  is  a  plane,  have  been 
])retty  fully  exemplified  in  the  preceding  chapter,  and  it 
remains  now  only  to  comment  briefly  on  the  general  case. 

If  a  section  plane  be  taken  through  any  two  intersecting 
surfaces  whatever,  and  the  lines  of  section  with  each  of  the 
surfaces  be  drawn,  the  point,  or  points,  in  which  the  lines 
c  f  section  meet  each  other,  will  be  in  the  intersection  of  the 
sarfaces.  Consequently,  by  taking  a  series  of  these  auxi- 
1  ary  section  planes,  a  sufficient  number  of  points  may  be 
f  )und,  and  the  line  of  intersection  drawn  through  them. 

Although  the  auxiliary  planes  may  be  chosen  arbitrarily, 
it  is  obviously  convenient,  in  general,  to  take  them  parallel, 
o  •  perpendicular,  to  one  of  the  planes  of  projection,  so  that 
o  le  projection  of  the  section  shall  always  be  a  straight  line. 
T  his  rule  has,  however,  sometimes  to  give  way  to  other 
c  )nsiderations,  as  when,  for  instance,  we  are  dealing  with 
J  uled  Surfaces,  in  which  case  it  often  facilitates  the  work  if 
tl  e  auxiliary  planes  be  so  taken  as  to  cut  the  surface  along 
it ;  rectilineal  generatrices. 


1/4  SOLID   OR  DESCRIPTIVE  GEOMETRY. 

Note.  For  the  application  of  the  principle  of  parallel 
sections  to  the  intersection  of  forms  given  by  their  figured 
plafis,  see  Prob.  ii,  Chapter  x.,  "Figured  Projections  and 
3cales  of  Slope." 

FORMS  BOUNDED  BY  PLANE  SURFACES. 

I.  Two  right  prisms,  with  square  bases  of  2-^  in.  side, 
intersect.  One  is  placed  with  its  edges  horizontal,  and  parallel 
to  the  vertical  plane ;  its  lowest  face  inclined  30°,  and  its 
lowest  edge  i  in.  above  the  horizontal  plane.  The  other  stands 
on  its  base  with  one  face  inclined  2^°  to  the  vertical  plane,  and 
its  axis  passing  throtigh  the  middle  point  of  the  highest  edge  of 
the  horizontal  prism.  Axes  of  the  prisms  4  in.  long.  Draw 
plan  and  elojation  showing  the  intersection,  and  develop  the 
vertical  prism. 

To  get  the  prisms  into  position : — 

Commence  with  an  auxiliary  elevation  of  the  horizontal 
prism  on  a  vertical  plane  perpendicular  to  its  axis,  so  that 
the  elevations  ^of  both  bases  coincide.  The  elevation  of  the 
prism  in  this  position  will  be  a  square,  2*5  in.  side,  the 
lowest  corner  i  in.  above,  and  one  side  making  an  angle  of 
30"  with  xy.  For  the  final  elevation  take  a  new  xy  parallel 
to  the  plan  of  the  axis. 

For  the  plan  of  the  vertical  prism,  describe  a  circle  of  2-5 
in.  diameter  from  the  middle  point  of  the  plan  of  the 
highest  edge  of  the  horizontal  prism  as  centre.  A  pair  of 
parallel  tangents  to  this  circle  making  angles  of  25<*  with 
the  new  xy,  and  a  second  pair  at  right  angles  to  the  former, 
will  determine  the  square  base  of  the  vertical  prism  in  the 
required  position.  Thence  determine  the  elevation  on  the 
new  vertical  plane. 


INTERSECTIONS.— PLANE  SURFACES.  1/5 

To  fijid  the  intersection : — 

The  forms  being  bounded  by  plane  faces,  the  lines  of 
intersection  will  be  straight.  And  since  but  one  straight 
line  can  be  drawn  between  two  points,  it  is  only  necessary 
to  determine  two  points  in  each  of  the  intersections  of  the 
faces,  and  to  draw  the  line  of  intersection  through  them. 
The  points  most  convenient  for  the  purpose  are  those  in 
which  each  of  the  edges  of  the  one  prism  pierces  the  faces 
of  the  other. 

Begin  with  the  edges  of  the  horizontal  prism.  The 
plans  of  the  points  in  which  these  meet  the  faces  of  the 
vertical  prism  are  shown  where  the  plans  of  the  horizontal 
edges  of  the  one  prism  cut  the  square  which  is  the  plan  of  the 
other.  Determine  the  elevations  of  these  points  by  drawing 
projecting-lines  from  their  plans  to  meet  the  elevations  of 
:he  edges  in  which  they  lie. 

Observe  : —  Each  edge  should  be  consideied  separately,  and 
xs  much  of  it  as  is  visible  be  put  in  as  a  dark  continuous  line ; 
■  iny  part  that  is  invisible  should  be  dotted,  and  that  portion 
which  passes  through  the  other  prism  rubbed  entirely  out. 

Unless  this  rule  be  strictly  attended  to  before  any 
ittempt  is  made  to  join  up  for  the  intersection,  inextricable 
:6nfusion  will  in  general  be  the  end  of  an  abortive  eftbrt 
:he  reverse  of  instructive. 

For  the  edges  of  the  vertical  prism,  Measure  the  heights 
It  which  they  pierce  the  faces  of  the  horizontal  prism  from 
he  auxiliary  elevation,  and  transfer  them  to  the  new  vertical 
)rojection. 

Treat  each  edge  of  the  vertical  prism  in  the  same  way 
is  recommended  for  the  horizontal  edges  in  the  observation 
jrinted  in  italics  above. 


176  SOLID  OR  DESCRIPTIVE  GEOMETRY. 

The  lines  of  intersection  can,  at  this  stage,  be  easily  put  111 . 
from  inspection. 

To  work  the  development : — 

Develop  the  entire  prism  as  In  Prob.  3,  Chap,  i,  and  for 
the  development  of  the  intersection,  find,  first,  the  points  on 
the  edges,  by  measuring  from  one  of  the  elevations,  and, 
second,  the  points  in  the  faces.  These  are  determined  by 
their  distances  from  the  edges  and  their  heights  above  the 
base. 

Prick  off  the  development  on  a  sheet  of  cardboard  and 
make  the  model  of  the  intersection. 

2.  Using  the  same  plan  and  auxiliary  elevation  as  in  the 
preceding  problem,  make  a  new  elevation  on  a  vertical  plane 
inclined  at  40"  to  the  long  edges  of  the  horizontal  prism,  so  as 
to  show  the  parts  which  were  invisible  in  the  elevation  in 
Problem  i.  Develop  the  horizontal  prism  and  deduce  a  second 
plan  of  the  intersecting  solids — xy  at  pleasure. 

3.  A  regular  pentagonal  prism,  i  '5  in.  side,  5  in.  long, 
stands  on  the  horizontal  plane  with  the  vertical  face  passing 
through  the  side  of  the  base  nearest  to  y^y  parallel  to  V.P.,  and 
is  intersected  by  a  similar  and  equal  prism  7vith  its  axis 
horizontal,  2'^  in.  high,  inclined  at  32°  to    V.P.  and  'ic^in. 

from  the  axis  of  the  other.  The  rectangular  face  of  the 
horizontal  prism  that  is  furtliest  from  V.P.  to  be  vertical. 
Plan,  elevation,  and  development. 

Begin  with  a  plan  of  the  vertical  prism  at  a  convenient 
distance  in  front  of  xy. 

Since  the  axis  of  the  horizontal  prism  is  Inclined  at  30" 
to  V.P.  the  base  must  be  inclined  at  60°,  the  complement 
of  the  angle  of  inclination  of  the  axis,  to  the  same  plane. 


INTERSECTIONS— PLANE  SURFACES.  lyj 

Determine  an  auxiliary  vertical  plane  of  projection  parallel 
io  the  base  of  the  horizontal  prism,  and  show  thereon  an 
elevation  of  the  vertical  prism.  A  point  in  this  elevation, 
2 "5  in.  above  xy  and  "25  in.  from  the  elevation  of  the  axis 
of  the  vertical  prism,  will  be  the  elevation  of  the  axis  of 
:he  horizontal  one,  and  if  a  circle  be  described  from  this 
:)oint,  with  the  proper  radius,  the  pentagon  which  is  the 
luxiliary  elevation  of  the  horizontal  prism  may  be  put 
about  it  with  one  side  vertical,  and  the  plan  and  new 
elevation  be  deduced  without  difficulty. 

The  intersection  is  worked  as  in  Probs.  i  and  2. 

4.  A  right  pristn,  4  in.  long,  bases  equilate?'al  triangles 
if  2  in.  side;  axis  horizontal,  inclined  30"  to  V.P. ;  lowest 
i-ige  -z^in.  above,  and  a  face  containing  it  inclined  15"  to 
/I. P.,  intersects  a  right  pyramid,  square  base  2*5  in.  side,  axis 
J  in.  long,  vertical,  and  passing  through  the  middle  point  of  the 
cxis  of  the  prism.  Side  of  square  base  inclined  at  30"  to  xy 
c  nd  not  parallel  to  the  long  edges  of  the  prism.  Show  intersec- 
t  'on  in  plan  and  elevation  and  work  development  of  the  pyramid. 

Determine  plan  and  elevation  of  prism  as  in  Problem  6, 
Chap.  I.,  page  31,  by  means  of  an  auxiliary  elevation  on  a 
\  lane  at  right  angles  to  the  axis. 

For  the  pyramid ; —  From  the  middle  of  the  plan  of  the 
a  ds  of  the  prism  as  centre,  describe  a  circle  of  2  '5  in. 
diameter  and  circumscribe  it  by  a  square,  in  the  right 
p  osition,  for  the  plan  of  the  base  of  the  pyramid.  Thence 
d2termine  its  elevation,  and  also  an  elevation  on  the  auxi- 
liiry  vertical  plane  of  projection  which  was  used  to  find 
t  le  plan  of  the  prism. 

To  work  the  intersection : — 

(i)     Find  the   projections  of  the   points  in  which  the 
e  Iges  of  the  pyramid  pierce  the  faces  of  the  prism. 
E.  G.  12 


1/8  SOLID   OR  DESCRIPTIVE   GEOMETRY. 

The  elevations  of  these  points  are  shown  in  the  auxi- 
liary plane  where  the  elevations  of  the  edges  of  the  pyramid 
cut  the  equilateral  triangle  which  is  the  projection  of  the 
prism  on  that  plane.  From  these  the  other  required  pro- 
jections may  be  found  by  drawing  projecting-lines  from 
them  to  meet  the  other  projections  of  the  lines  that  contain 
the  points. 

When  these  points  are  found,  put  in  the  edges  of  the 
pyramid  in  the  way  recommended  for  the  edges  of  the 
prism  Prob.  i. 

(2)  Find  the  projections  of  the  points  in  which  the 
edges  of  the  horizontal  prism  pierce  the  pyramid : — 

Assume  that  the  three  corners  of  the  equilateral  triangle 
in  the  auxiliary  elevation  are  the  elevations  of  the  points  in 
which  the  horizontal  edges  of  the  prism  meet  the  faces  of 
the  pyramid,  and,  on  this  assumption,  draw  lines  from  the 
vertex  of  the  pyramid  through  these  points  in  its  faces  and 
produce  them  to  the  base.  The  projections  of  these  lines 
on  the  auxiliary  plane  will  be  drawn  from  the  projection  of 
the  vertex  on  that  plane  through  the  three  corners  of  the 
triangle.  Each  of  these  is  the  elevation  of  two  lines — one 
passing  through  the  point  in  which  the  edge  of  the  prism 
enters,  and  the  other  through  the  point  in  which  it  leaves 
the  pyramid.  Determine  these  lines  in  plan.  The  points 
common  to  these  lines  and  the  plans  of  the  horizontal 
edges  of  the  prism  will  be  the  plans  of  the  required  points, 
from  which  the  other  elevations  may  be  at  once  obtained. 

Put  in  the  projections  of  the  edges  in  the  proper  manner 
and  then  join  up  for  the  projections  of  the  intersection. 

Develop  the  pyramid  as  in  Prob.  i,  Chap.  i.  The  points  in 
the  faces  may  be  readily  found  by  showing  in  the  develop- 


INTERSECTIONS— PLANE  SURFACES.  179 

ment  the  lines  which  pass  through  them,  and  determining 
them  in  these  Hnes  in  the  same  way  as  the  points  in  the 
edges  are  found. 

5.  For  the  prism  in  the  preceding  problem,  substitute  an 
irregidar  pentagonal  prism  with  a  re-entering  angle.  No  face 
to  be  either  horizontal  or  vertical :  axis  of  pyramid  to  pass 
throjfgh  the  middle  point  of  the  highest  edge  of  the  prism,  and 
the  latter  form  to  have  two  long  edges — one  on  each  side  of  the 
axis — entirely  free  from  the  pyramid.  Other  conditions  as  in 
Problem  4.  Plan,  elevation,  and  development  of  pyramid 
showing  the  lines  of  intersectioft. 

Begin  with  a  plan  of  the  pyramid  and  an  auxiliary 
elevation  on  a  vertical  plane  parallel  to  the  plane  of  the 
base  of  the  prism.  The  end  elevation  of  the  prism  may 
then  be  put  in  so  as  to  fulfil  the  conditions,  and  the  problem 
worked  as  in  Prob.  4  above. 

6.  Intersection  of  a  right  square  pyramid  and  a  regular 
i>entagonal  prism. 

Pyramid: — Axis  4  in.  long,  inclined  at  35"  to  H.P.,  and 
in  plan  making  an  angle  of  2^  with  xy  :  side  of  square  base 
2  in.  and  inclined  at  30°  to  the  horizontal  trace  of  the  plane  of 
'he  base :  lowest  corner  resting  upon  H.P. 

Prison : — Standing  on  the  H.P.  tvith  a  regular  pentagon 
f  \'^in.  side  for  its  base  ;  axis  3*5  in,  long  and  vertical. 

Inter sectio7i  when  the  prism  is  so  placed  that  the  vertical 

face  furthest  fro7n  xy  is  parallel  to  the  V.P.  of  projection,  and 

has  one  of  its  long  edges  free  from  the  pyramid.     All  other 

'dges  of  the  prism  to  pass  through  the  pyramid  and  one  of 

hem  to  meet  it  at  a  point  in  its  base. 

Work  also  a  development  of  the  pyramid. 
The  points  in  which  the  edges  of  the  pyramid  meet 

12 — 2 


l80  SOLID   OR  DESCRIPTIVE   GEOMETRY. 

the  vertical  faces  of  the  prism  are  shown  in  plan  where 
the  plans  of  these  edges  cross  the  sides  of  the  penta- 
gonal base,  and  can  thence  be  determined  in  elevation. 

Note  : — It  sometimes  happens  that  an  edge  of  the 
pyramid  pierces  a  base  of  the  prism,  in  which  case  the 
point  of  intersection  is  readily  deduced  from  the  elevation. 

The  intersections  of  the  vertical  edges  of  the  prism 
with  the  faces  of  the  pyramid  can  be  found  either  by  the 
general  method  for  the  intersection  of  a  straight  line  and  a 
plane,  or  by  the  following  equivalent  special  construction : — 

Draw  lines  from  the  plan  of  the  vertex  of  the  pyramid 
through  the  points  which  are  the  plans  of  the  vertical  edges 
of  the  prism,  and  produce  them  to  the  base  of  the  pyramid. 
Assuming  each  of  these  lines  to  be  the  plan  of  two  lines — 
one  drawn  from  the  vertex  through  the  point  in  which  the 
edge  of  the  prism  enters  one  face  of  the  pyramid  and  the 
other  from  the  vertex  through  the  point  in  which  it  leaves 
another — show  them  in  the  elevation.  The  intersections 
of  the  elevations  of  these  lines  with  the  elevations  of  the 
edges  through  which  they  are  drawn  in  plan,  give  the 
elevations  of  the  required  points  of  intersection. 

The  intersection  of  the  edge  of  the  prism  with  the  base 
of  the  pyramid  may  be  similarly  found  by  drawing  a  line, 
say,  from  one  corner  of  the  plan  of  the  square  base, 
through  the  plan  of  the  vertical  edge  meeting  it,  and 
showing  this  line  in  the  elevation  of  that  base. 

Complete  the  representation  of  the  edges  of  the  inter- 
secting forms  and  then  join  up  the  points  of  intersection 
from  inspection. 

7.     A  right  hexagonal  pyramid^  i"]^in.  side,  axis  51/7. 


INTERSECTIONS— PLANE  SURF  A  CES.  1 8 1 

long,  stajids  on  its  base  with  one  side  making  an  angle  of  40" 
with  xy,  a7id  is  intersected  by  a  right  square  prism,  2  in.  side, 
axis  passing  through  that  of  the  pyramid,  inclined  30°  to  H.P., 
a?id  in  plan  making  an  angle  of  ^0°  with  xy.  JVo  side  of  the 
square  to  be  horizontal.    Show  plan,  elevation,  and  intersection. 

Develop  the  prism. 

Find  the  points  in  which  the  edges  of  the  pyramid  meet 
the  faces  of  the  prism,  by  the  aid  of  auxiliary  vertical 
section  planes  cutting  the  prism  and  containing  the  slant 
edges  of  the  pyramid.  Similarly,  find  the  intersections  of 
the  edges  of  the  prism  with  the  faces  of  the  pyramid  by  the 
aid  of  vertical  planes  containing  the  former. 

8.      Work  the  following  general  cases : — 

a.  Two  intersecting  prisms  of  four  and  six  sides  respec- 
tively, axes  oblique  to  both  planes  of  projection.  Other  data 
^t  pleasure. 

p.  Two  intersecting  pyramids  of  four  and  five  sides 
■espectively,  axes  oblique  to  both  planes  of  projection.  Other 
lata  as  above, 

y.     General  case  of  irregular  prism  and  pyramid. 

When  the  traces  of  the  indefinite  planes  of  the  faces 
]  all  within  the  limits  of  the  paper  the  intersections  of  these 
;)lanes  may  be  found  by  Prob.  12,  Chap.  in.  and  that  of 
ihe  forms  put  in  therefrom.  As,  however,  it  seldom 
]  appens  that  all  the  traces  of  the  planes  intersect  within  the 
1  mits  of  the  drawing,  recourse  is  had  to  auxiliary  section 
1  lanes.  Two  such  planes — one  parallel  to  one  plane  of 
]  rojection  and  the  other  to  the  other — will  be  found  most 
(  onvenient  for  the  purpose.  From  the  points  furnished  by 
tie  sections,  and  such  of  the  traces  as  meet  on  the  paper, 
1  le  intersections  can  be  put  in  without  much  difficulty. 


1 82  SOLID  OR  DESCRIPTIVE  GEOMETRY. 


FORMS  BOUNDED  BY  CURVED  SURFACES. 

As  far  as  possible  the  following  exercises  have  been 
grouped  according  to  the  inost  convenient  methods  of  work- 
ing them.  Some  of  the  problems  admit  of  several  con- 
structions, but  on  the  whole  it  will  be  found  that  the  problems 
worked  in  each  group  afford  types  of  the  most  approved 
method  of  working  those  of  the  same  group  to  which  no 
solutions  are  appended. 

^^  Critical  lines"  and  ^^ Key  points."  In  all  intersections 
that  involve  curved  surfaces  the  student  should  direct  his 
attention  to  the  lines  in  which  the  projecting  surfaces  touch 
the  curved  forms  and  the  points  in  which  these  lines  on  the 
one  form  meet  the  surface,  or  surfaces,  bounding  the  other. 

The  line  of  intersection  always  passes  through  these 
points,  and  inasmuch  as  a  habit  of  attending  to  them,  and 
to  their  corresponding  lines,  greatly  simplifies  matters  by 
developing  a  power  of  prospective  insight  into  the  characte- 
ristic features  of  the  intersection  resulting  from  any  given 
combination,  they  have  been  specially  designated  the  critical 
lines  and  key  points. 

Group  I. 

I .  Intersection  of  a  right  cotie  and  a  sphere.  Cone  standing 
on  its  circular  base  of  2'<^  in.  radius,  axis  3  in.  long:  sphere 


INTERSECTIONS^CURVED  SURFACES.  1 83 

resting  upon  H.P.^  radius  1*25  in.,  centre  i  in.  from  the  axis 
of  the  cone  ;  line  joining  the  plans  of  the  centre  of  the  sphere 
and  the  vertex  of  the  cone  inclined  at  2i^^  to  xy.  Development 
of  cone  showing  curve  of  intersection. 

Take  a  series  of  sections  of  both  surfaces  by  auxiliary 
planes  parallel  to  H.P.  The  elevation  of  each  will  be  a 
straight  line  parallel  to  xy,  and  the  plan,  two  circles — one 
described  from  the  plan  of  the  centre  of  the  sphere  and  the 
other  from  the  plan  of  the  vertex  of  the  cone,  as  centres,  with 
radii  that  may  be  measured  from  the  elevation.  The  points 
in  which  each  pair  of  circles  intersect  will  be  points  in  the 
plan  of  the  intersection  of  the  forms,  and  from  these  the 
2levations  may  be  found  by  projecting,  and  the  projections 
Df  the  intersection  put  in. 

Note.  It  will  simplify  the  work  if  the  auxiliary  planes 
ire  taken  at  equal  distances  above  and  below  the  centre  of 
;he  sphere — so  that  one  circle  in  plan  shall  serve  for  two 
jections  of  that  surface. 

For  the  development. 

(i)  Divide  the  base  of  the  cone  in  plan  into  a  number 
)f  equal  parts  and  put  in  the  plans  of  the  generatrices  from 
;he  points  of  division  to  the  vertex. 

(2)  Develop  the  cone  and  show  these  generatrices 
n  the  development. 

(3)  Mark  the  points  where  the  curve  of  intersection,  in 
')lan,  meets  the  plans  of  the  generatrices,  and  divide  the 
corresponding  generatrices  in  the  development  in  the  same 
)roportion,  for  the  points  through  which  the  curve  of  inter- 
ection  passes. 

2.     Arrange  an  example  of  two   intersecting  spheres  of 


184  SOLID   OR  DESCRIPTIVE   GEOMETRY. 

unequal  sizes  and  unequal  heights;  the  line  joining  their  centres 
oblique  to  both  planes  of  projection. 

3.  Ititersection  of  a  sphere  and  a  surface  of  revolution 
with  vertical  axis — e.g.  the  Spheroid,  or  the  Hyperboloid,  in 
Chap.  VIL 

4.  Intersection  of  a  right  cone  and  a  right  cylinder  with 
their  axes  vertical. 

Note.  The  plan  of  the  intersection  in  this  problem,  and 
in  the  next,  coincides  with  the  circle  that  is  the  plan  of  the 
cylinder. 

5.  Intersection  of  a  sphere  and  a  right  cylinder  standing 
on  its  base. 

Note.  Any  surface  of  revolution  with  its  axis  vertical 
might  be  substituted  for  the  sphere  in  this  problem. 

6.-  Arrange  an  example  of  a  sphere,  or  a  spheroid,  axis 
vertical,  intersecting  an  oblique  cone  or  cylinder  with  a  circular 
horizontal  trace. 

7.  Intersection  of  a  sphere  with  the  anchor  ring,  Prob.  5, 
'■^Surfaces  of  Revolution^''  Chap.  VII. 

8.  Intersection  of  an  oblique  cone,  or  a  cylinder,  having 
a  circle  for  its  horizontal  trace,  with  an  anchor  ring,  axis 
vertical. 

9.  Intersection  of  a  right  cone  standing  on  its  base  and  an 
oblique  cone,  or  cylinder,  with  circular  horizontal  trace. 

I  o.      Work  one  of  the  following  combinations : — 

a.      Two  oblique  cylinders  with  circular  horizontal  traces. 

(3.     Two  oblique  cones  with  circular  horizontal  traces. 


INTERSECTIONS— CURVED  SURFACES.  1 85 

7.     Oblique  cone  and  oblique  cylinder  with  circular  horizon- 
tal traces. 


Group    1 1. 

I.  Intersection  of  two  right  cylinders.  One  2  in.  diameter  ; 
axis,  horizontal,  2*5  in.  above  If. P.,  inclined  40°  to  V.P.,  and 
5  in.  long ;  the  other  2*5  in.  diameter,  axis  vertical  and  5  in. 
long. 

a.  When  the  axis  of  the  vertical  cylinder  passes  through 
the  middle  point  of  the  horizontal  one. 

p.  When  the  axis  of  one  is  "25  in.  from  that  of  the  other 
so  that  the  cylinders  have  a  common  tangent-plane. 

y.      When  t/u  axis  of  one  is  '5  in.  from  that  of  the  other. 

Develop  the  horizontal  cylinder  in  the  last. 

To  find  the  intersection : — Divide  one  base  of  the  horizontal 
cylinder  into  any  number  of  equal  parts  (say  sixteen),  and 
through  the  points  of  division  draw  straight  lines  (i.e. 
generatrices)  on  the  cylindrical  surface,  and  show  them 
in  plan  and  elevation. 

( 1 )  To  show  the  lines  in  plan : — Rabat  the  base  about  its 
horizontal  diameter  till  parallel  to  the  H.P.  This  will  be 
a  circle  described  upon  a  short  side  of  the  rectangle  which 
is  the  plan  of  the  horizontal  cylinder.  Draw  half  of  the 
circle  only  and  divide  it  into  eight  equal  parts.  Lines  from 
the  points  of  division,  parallel  to  the  plan  of  the  axis,  will 
determine  the  plans  of  the  generatrices— each  line,  with  the 
exception  of  the  two  outside  ones,  representing  two  gene- 
ratrices— one  above  and  one  below  the  level  of  the  axis. 

(2)  To  show  the  lines  in  elevatioji: — Rabat  the  base 
about  its  vertical  diameter  till  parallel  to  the  V.P.     This 


1 86  SOLID   OR  DESCRIPTIVE  GEOMETRY. 

will  be  a  circle  described  upon  the  major  axis  of  the  ellipse 
which  is  the  elevation  of  the  base  of  the  horizontal  cylinder. 
Only  half  of  this  circle  need  be  shown,  and  this,  if  divided 
into  eight  equal  parts,  will  furnish  points  through  which  lines 
parallel  to  the  elevation  of  the  axis  may  be  drawn  for  the 
elevations  of  the  generatrices.  These  lines  in  elevation,  with 
the  exception  of  the  highest  and  lowest,  will  each  represent 
two  generatrices — one  on  the  front  and  one  on  the  back 
part  of  the  cylindrical  surface. 

Note.  Any  horizontal  lines  whatever  may  be  taken  on 
the  cylinder  in  plan,  and  their  elevations  determined  in  the 
vertical  projection  by  the  aid  of  an  auxiliary  elevation  of  the 
horizontal  cylinder  on  a  vertical  plane  at  right  angles  to  its 
axis,  whence  the  heights  can  be  measured :  but  it  is  much 
more  convenient  to  take  them  at  equal  distances  apart, 
especially  when  a  development  has  to  be  worked. 

Having  shown  these  generatrices  in  plan  and  elevation, 
the  points  in  which  they  meet  the  surface  of  the  vertical 
cylinder  may  be  determined  for  points  in  the  curve  of  inter- 
section. 

Iti  plan.,  the  curve  of  intersection  coincides  with  the 
circle  that  is  the  plan  of  the  vertical  cylinder,  and  the  plans 
of  the  points  in  which  the  lines  on  the  horizontal  cylinder 
meet  the  surface  of  the  vertical  cylinder  are  shown  where  the 
plans  of  these  horizontal  lines  cross  its  circular  base.  Project- 
ing-lines,  drawn  from  these  points  to  meet  the  corresponding 
horizontal  lines  in  elevation,  determine  the  points  through 
which  the  elevation  of  the  line  of  intersection  is  to  be  drawn. 

Note.  Pay  attention  to  the  critical  lines  and  key  points, 
and  number  the  generatrices  in  plan  and  elevation  to  avoid 
confusion. 


INTERSECTIONS— CURVED  SURFACES.  1 8/ 

The  development  may  be  worked  as  in  preceding 
chapters. 

2.  Intersection  of  a  right  cylinder  with  a  right  cone. 
Cylinder  2*5  in.  diameter,  axis  5  in.  long  and  vertical ;  cone 
with  base  2-5  in.  diameter;  axis,  5  in.  long,  horizontal,  in- 
clined 45"  to  V.F.  and  2*5  in.  above  H.P. 

a.  When  the  axis  of  the  cylinder  passes  through  the  middle 
point  of  the  axis  of  the  co?ie. 

p.      When  the  two  surfaces  have  a  common  iangentplane. 
y.      When  part  of  the  cone  is  without  the  cylinder. 
Develop  the  cone  in  ft. 

Divide  the  base  of  the  cone  into  a  number  of  equal  parts, 
as  was  done  with  the  base  of  the  cyhnder  in  the  foregoing 
problem,  and  show  plan  and  elevation  of  the  generatrices. — 
The  lines  in  this  case  instead  of  being  parallel  to  the  axis  will 
radiate  from  the  vertex  to  the  points  on  the  base,  and  the 
student  must  observe  that  although  in  the  plan  each  line  will, 
as  in  the  cylinder,  represent  two  generatrices,  in  the  elevation 
this  will  not  be  the  case,  /.  e.  the  front  and  back  lines  not 
being  parallel  to  the  axis  will  not  coincide. 

When  the  plans  and  elevations  of  the  generatrices  are 
determined,  the  intersection  is  found  as  before. 

The  development  is  worked  as  in  other  chapters. 

3.  Arrange  a  right  cylinder  standing  on  its  base  and  find 
its  line  of  section  with  a  right  cylinder  whose  axis  is  inclined 
to  both  planes  ofprofection. 

Take  any  number  of  convenient  points  in  the  horizontal 
trace  of  the  oblique  cylinder  and  draw  plans  and  elevations 
of  generatrices  passing  through  them.     The  elevation  of  the 


1 88  SOLID  OR  DESCRIPTIVE  GEOMETRY. 

intersection  can  then  be  found  from  the  points  where  the 
plans  of  the  generatrices  cross  the  horizontal  trace  of  the 
vertical  cylinder,  as  in  the  other  problems  of  this  group. 

Note.  The  oblique  cylinder  might  be  assumed  with  a 
circle  for  its  horizontal  trace  and  worked  as  above,  but  in  this 
case  it  could  be  as  conveniently  worked  by  a  series  of  hori- 
zontal auxiliary  sections,  as  in  Group  I. 

4.  Arrange  a  vertical  cylinder  and  an  oblique  cone  and 
find  their  intersection. 

The  generatrices  in  this  case  are  drawn  to  the  vertex  of 
the  cone ;  with  this  exception  the  description  of  the  con- 
struction for  the  preceding  problem,  and  the  "Note,"  apply 
to  this. 

5.  Find  the  intersection  of  the  anchor  ring,  Proh.  5, 
'•'■Problems  on  Surfaces  of  Perolution"  Chap.  VIL,  with  a 
vertical  cylinder. 

Divide  a  vertical  circular  section  of  the  ring  into  a  number 
of  equal  parts,  as  for  the  base  of  the  cylinder,  Prob.  i,  of  this 
group,  and  through  the  points  of  division  draw  horizontal 
circular  lines  on  its  surface.  The  ring  when  thus  prepared 
may  be  likened  in  some  respects  to  a  long  cylinder  bent 
into  a  circular  form,  and  the  intersection  will  be  found  to 
present  no  greater  difficulty  than  that  of  the  horizontal 
cylinder  in  Prob.  i  above. 

Note.  This  same  construction,  looked  at  in  another 
aspect,  might  be  regarded  as  an  illustration  of  the  method  of 
working  by  a  series  of  parallel  sections  as  in  Group  I. 

Group  III. 

I.  Intersection  of  a  right  cone  standing  on  a  circular  base 
of  I  in.  diameter,  axis  vertical  and  ^in.  long,  with  a  right 


INTERSECTIONS— CURVED  SURFACES.  1 89 

cylinder  resting  upon  H.P. ;  base  2  '5  in.  dia7neter,  axis 
horizofital,  5*5  in.  long,  inclined  /Hf^"  to  xy,  and  in  plari  75  in. 
distant  from  the  plan  of  the  vertex  of  the  cone.  Develop  the 
cone  with  curve  of  intersection. 

Determine  horizontal  lines  (generatrices)  on  the  surface 
of  the  cylinder  as  in  the  horizontal  cylinder,  Prob.  i,  Group 
II.,  and  work  sections  of  the  cone  by  auxiliary  horizontal 
planes  containing  these  lines. 

In  elevation  the  sections  of  the  cone  will  be  represented 
by  the  segments  of  the  elevations  of  the  generatrices  that  lie 
between  the  legs  of  the  triangle  forming  the  elevation  of  the 
cone. 

In  plan  these  sections  will  be  circles,  described  from  the 
plan  of  the  vertex  of  the  cone  as  centre,  with  radii  measured 
from  their  corresponding  elevations. 

The  points  in  plan  in  which  the  generatrices  of  the 
cylinder  meet  the  corresponding  circles  of  the  same  level, 
will  be  points  in  the  plan  of  the  intersection,  through  which 
the  curve  must  be  drawn  by  hand. 

Projecting-lines,  drawn  from  these  points  to  the  elevations 
of  the  generatrices  in  which  they  lie,  will  determine  points  in 
the  elevation  of  the  curve  of  intersection. 

2.  Liter  section  of  a  horizontal  cylinder  with  a  sphere. 
Sphere  4  in.  diameter,  and  centre  2  '5  in.  high. 

Cylinder  2  in.  diameter^  axis  horizotital,  inclined  45°  to 
V.P.,  in  plan  '5  in,  from,  and  in  elevation  75  in.  below,  centre 
f  sphere. 

Develop  the  cylinder. 

3.  If  iter  section  of  a  horizontal  cylinder  with  the  Hyper- 
'wloid  of  Revolution  of  one  sheet  given  in  Chapter  VII. 


1 90         SOLID  OR  DESCRIPTIVE  GEOMETRY. 

4.  A  surface  generated  by  the  revolution  of  a  circle  of  2,  in. 
diameter  about  a  tangent-line  as  axis,  rests  upon  H.P.  with  the 
axis  vertical ;  a  sphere  ^2*5  in.  diameter  rolls  along  the  H.P. 
on  a  line  inclined  at  40"  to  V.P.  and  tangential  to  the  circle  in 
which  the  surface  of  revolution  touches  the  H.P.  Show  the 
opening  through  which  the  sphere  would  pass. 

5.  Arrange  an  example  of  a  horizontal  right  cylinder, 
axis  inclined  to  V.P.,  intersecting  an  oblique  cone,  or  oblique 
cylinder,  which  has  a  circle  for  its  horizontal  trace. 

Group  IV. 

I .  A  r range  an  example  of  a  horizontal  right  cylinder,  axis 
inclined  to  V.P.,  intersecting  another  right  cylinder,  axis 
oblique  to  both  planes  of  projection,  and  show  the  curve  of 
intersection. 

Auxiliarysectionsof  the  cylinders  determined  by  horizon- 
tal planes  as  in  Group  III.  would  cut  the  horizontal  cylinder 
in  generatrices  and  the  oblique  cylinder  in  ellipses.  The 
labour,  however,  of  drawing  these  curves  would  be  consider- 
able, and  it  is,  therefore,  better  to  take  auxiliary  sections  by 
planes  containing  generatrices  of  the  horizontal  cyHnder  and 
parallel  to  those  of  the  oblique  cylinder. 

( 1 )  Prepare  the  horizontal  cylinder  by  drawing  a  number 
of  rectilineal  generatrices  on  its  surface,  as  in  preceding 
problems. 

(2)  Assume  a  point  in  a  generatrix  of  the  horizontal 
cylinder  and  draw  therefrom  a  line  parallel  to  the  generatrices 
(or  axis)  of  the  oblique  cylinder. 

(3)  Find  the  H.T.  of  this  line  and  through  it  draw  a 
straight  line  in  the  H.P.  parallel  to  the  plan  of  the  generatrices 
of  the  horizontal  cylinder. 


INTERSECTIONS—CURVED  SURFACES.  IQI 

The  line  last  drawn  will  be  the  H.T.  of  an  auxiliary 
plane  containing  a  generatrix  of  the  horizontal  cylinder 
and  parallel  to  the  generatrices  of  the  oblique  cylinder. 

(4)  From  each  of  the  two  points  in  which,  in  general, 
the  H.T.  of  this  auxiliary  plane  meets  the  curve  which  is  the 
horizontal  trace  of  the  oblique  cylinder,  draw  plans  of  the 
generatrices  of  the  latter  surface.  These  will  be  the  plans 
of  the  lines  in  which  the  auxiliary  section  plane  cuts  the 
oblique  cylinder,  and  the  points  in  which  these  lines  meet 
the  generatirix  of  the  horizontal  cylinder  will  be  points  in 
the  plan  of  the  curve  of  intersection  of  the  surfaces.  Simi- 
larly, other  points  may  be  found,  by  taking  auxiliary  planes 
through  each  of  the  generatrices  of  the  horizontal  cyHnder, 
and  the  curve  put  in  by  hand. 

The  elevation  may  be  readily  found  from  the  plan. 

2.  Arrange  an  example  of  a  horizontal  right  cylinder, 
axis  inclined  to  V.F.,  intersecting  an  oblique  right  cone,  and  show 
the  curve  of  intersection. 

Take  the  auxiliary  section  planes  in  this  case  containing 
generatrices  of  the  horizontal  cylinder  and  the  vertex  of  the 
oblique  cone. 

Note.  To  find  the  H.T.  of  the  section  planes,  the  Hnes 
drawn  from  the  assumed  points  in  the  generatrices  of  the 
horizontal  cylinder,  in  the  last  problem  parallel  to  the  axis  of 
the  oblique  surface,  must  now  be  drawn  from  the  vertex  of 
the  cone.  The  oblique  cylinder  may  in  fact  be  likened  to  a 
cone  with  its  vertex  at  an  infinite  distance. 

3.  Arrange  a  right  cone  standing  on  its  base  intersecting 
a  cylindrical  surface  oblique  to  both  platies  of  J>roJectiofi,  and 


192  SOLID  OR  DESCRIPTIVE  GEOMETRY. 

show  curve  of  intersection.     The  H.T.  of  the  cylinder  not  to  he 
a  circle. 

Take  auxiliary  section  planes  containing  the  vertex 
of  the  cone  and  parallel  to  generatrices  of  the  cylinder. 

Note.  The  horizontal  traces  of  these  planes  will  all 
pass  through  the  H.T.  of  a  line  drawn  from  the  vertex 
of  the  cone  parallel  to  the  generatrices  of  the  oblique 
cylinder. 

4.  Arrange  a  right  cone  standing  on  its  base  intersecting 
another  right  cone  whose  axis  is  inclined  at,  say,  40"  to  the  V.P., 
and  horizontal. 

Take  auxiliary  planes  containing  generatrices  of  the  cone 
whose  axis  is  horizontal  and  passing  through  the  vertex  of 
the  one  whose  axis  is  vertical. 

Note.  All  the  horizontal  traces  of  these  planes  will 
pass  through  the  H.T.  of  the  line  joining  the  vertices  of  the 
two  cones. 

(i)  Divide  the  base  of  the  horizontal  cone  into  equal 
parts  and  determine  the  lines  on  its  surface  as  in  Prob.  2, 
Group  II. 

(2)  Draw  a  straight  line  from  the  vertex  of  the  vertical 
cone  through  any  convenient  point  in  a  generatrix  of  the 
horizontal  one  and  find  its  H.T.  This  will  be  a  second  point 
in  the  H.T.  of  the  auxiliary  plane  containing  the  generatrix. 
Similarly,  the  horizontal  traces  of  the  other  auxiliary  planes 
may  be  determined,  and  the  intersection  worked  as  before, 

5.  Arrange  a  right  cotie  standing  on  its  base  a?idfind  its 
intersection  with  another  right  cone,  axis  oblique  to  both  planes 
of  projection. 


INTERSECTIONS— CURVED  SURFACES.  193 

General  Cases. 

I.  To  find  the  intersection  of  two  cylindrical  surfaces 
oblique  to  both  planes  of  projection. 

Take  auxiliary  section  planes  parallel  to  the  generatrices 
of  both  surfaces. 

Note.  Some  little  difficulty  is  usually  experienced  in 
getting  the  cylinders  into  a  position  that  will  give  a  satis- 
factory intersection,  unless  somewhat  cumbrous  and  elaborate 
data  are  supplied  for  their  arrangement.  One  of  the 
simplest  ways  of  arranging  an  intersection  of  these  surfaces 
is  to  begin  by  drawing  two  lines  inclined  at  an  angle  of  about 
120°,  and  treat  these  as  the  plans  of  the  axes  of  the  inter- 
secting cylinders.  Let  one,  for  example,  be  a  right  cylinder 
of  2*5  in.  diameter,  axis  inclined  55",  and  the  other  a  right 
cylinder  of  2  in.  diameter,  axis  inclined  at  40",  and,  say,  a 
little  below  the  other. 

To  find  their  intersection.  Determine  plan  and  elevation 
of  any  assumed  point  in  space,  and  the  horizontal  traces  of 
two  lines  drawn  from  the  point,  one  parallel  to  the  axis  of 
the  one  cylinder,  and  the  other  parallel  to  the  axis  of  the 
other.  The  line  joining  the  horizontal  traces  of  these  Hnes 
will  be  the  H.T.  of  a  plane  containing  them,  and  therefore 
oi  z  plane  parallel  to  the  generatrices  of  the  two  surfaces. 

A  series  of  lines  parallel  to  this  horizontal  trace  can  then 
be  taken,  cutting  the  horizontal  traces  of  the  cylinders,  for 
the  horizontal  traces  of  the  auxiliary  planes. 

Lines  from  the  points  in  which  the  horizontal  traces  of 
the  auxiliary  planes  meet  the  horizontal  traces  of  the  surfaces, 
parallel  to  the  plans  of  their  respective  axes,  determine  points 
in  the  plan  of  the  curve  of  intersection,  which  may  be  drawn 
n  by  hand  and  projected  for  the  elevation. 

E.  G.  13 


194  SOLID  OR  DESCRIPTIVE  GEOMETRY. 

Attention  should  be  given  to  the  critical  lines  and  key 
points. 

2.  To  find  the  intersection  of  two  conical  surf  aces  oblique 
to  both  planes  of  projection. 

Take  auxiliary  section  planes  passing  through  the  vertices 
of  both  cones  and,  therefore,  cutting  both  surfaces  in 
generatrices. 

Note.  The  horizontal  traces  of  all  the  auxiliary  section 
planes  pass  through  the  H.T.  of  the  line  joining  the  vertices 
of  the  two  surfaces,  and  the  vertical  traces  of  the  same  planes 
pass  through  the  V.  T.  of  that  line. 

Sometimes,  when  the  vertical  trace  of  one  of  the  cones 
is  more  accessible  than  its  horizontal  trace,  the  vertical 
traces  of  the  auxiliary  section  planes  are  required. 

3.  To  find  the  intersection  of  an  oblique  cylindrical 
surface  with  an  oblique  cone. 

Take  auxiliary  section  planes  passing  through  the 
vertex  of  the  cone  and  parallel  to  the  generatrices  of  the 
cylinder. 

Note,  The  horizontal  traces  of  these  auxiliary  planes 
all  pass  through  the  H.T.  of  a  line  drawn  from  the  vertex  of 
the  cone  parallel  to  the  generatrices  of  the  cylinder. 

Group  V. 

I.  Arrange  an  oblique  right  cylinder  intersecting  a  sphere^ 
and  find  the  curve  of  intersection. 

If  an  auxiliary  section  of  the  two  surfaces  were  taken  by 
a  horizontal  plane  the  curve  cut  from  the  sphere  would  be 
a  circle,  and  that  from  the  cylinder  an  ellipse,  similar  and 


I 


INTERSECTIONS— CURVED  SURFACES.         IQS 

equal  to  that  which  is  the  horizontal  trace  of  the  latter 
surface.  The  two  points  in  which  the  ellipse  and  circle  meet 
would  be  points  in  the  curve  of  intersection  of  the  surfaces. 
If,  now,  an  auxiliary  cylindrical  suj-face  were  determined, 
having  the  circle  which  is  cut  from  the  sphere  for  its 
directrix  and  lines  parallel  to  the  generatrices  of  the  oblique 
cylinder  for  its  generatrices,  its  horizontal  trace  would  be  a 
circle  similar  and  equal  to  that  which  is  the  directrix  of  the 
auxiliary  surface,  and  if  generatrices  of  this  cylinder  were 
drawn  from  the  points  in  which  the  circular  horizontal  trace 
of  the  auxiliary  surface  cuts  the  elliptic  trace  of  the  original 
oblique  cylinder,  they  would  cut  the  directing  circle  in  the 
same  points  as  the  ellipse  first  mentioned.  Hence  the 
following  construction : — 

(i)  Determine  a  series  of  sections  of  the  sphere  by  hori- 
zontal planes. 

(2)  From  the  centre  of  one  of  these  circles  draw  a  line 
parallel  to  the  obHque  cylinder. 

(3)  Find  the  H.T.  of  this  line,  and  from  that  point  as 
;entre  describe  a  circle  in  the  H.P.  with  radius  equal  to  that 
)f  the  circle  cut  from  the  sphere. 

(4)  From  the  points  in  which  this  circle  (which  is  the 
'.  I.T.  of  the  auxiliary  cylinder)  cuts  the  ellipse,  draw  gene- 
1  atrices  meeting  the  directing  circle,  for  points  in  the  curve 
( f  intersection. 

Similarly,  by  treating  the  other  circles  in  the  same  way  a 
r  umber  of  points  in  the  curve  of  intersection  will  be  found. 


2.     Arrange  an  oblique  cone  intersecting  a  sphere,  and 
J.  >id  the  curve  of  intersection. 

13—2 


196         SOLID  OR  DESCRIPTIVE  GEOMETRY. 

The  construction  in  this  example  is  similar  to  that  of  the 
preceding  problem,  except  that  the  auxiliary  surfaces  are 
cones,  having  the  same  vertex  as  the  given  oblique  cone, 
instead  of  cylinders;  the  directrices  being,  as  before,  the 
circular  sections  of  the  sphere. 

Note.  The  constructions  given  for  the  two  problems  of 
this  group  apply  to  the  intersection  of  an  oblique  cylinder  or 
cone  with  any  surface  whose  sections  by  horizontal  planes 
are  circles,  e.g.  the  Anchor  ring,  or  the  Spheroid,  with  ver- 
tical axis. 

Group  VI. 

I.  To  find  the  intersection  of  two  surfaces  of  revolution 
when  their  axes  lie  in  one  plane. 

There  are  two  cases  : — 

a.      When  the  axes  are  parallel. 

ft.     When  the  axes  meet. 

For  a : — Use  auxiliary  section  planes  at  right  angles  to 
the  axes. 

For  ft : — Take  auxiliary  sections  of  the  surfaces  by  a 
series  of  concentric  spheres,  described  from  the  point  in 
which  the  axes  meet  as  centre.  Every  such  auxiliary  sphere 
will  cut  each  of  the  surfaces  of  revolution  in  a  circle  whose 
plane  is  perpendicular,  to  the  axis  of  the  surface,  and  the 
points,  in  which  the  pair  of  circles  cut  by  each  sphere  meet, 
will  be  points  in  the  curve  of  intersection.  The  following 
special  exercise  is  deserving  of  attention  :— 

A  triangle  ABC  is  the  base  of  a  pyramid  whose  vertex  is 
V.  AB=sin.;BC=A  in.  ;  CA  =  ^  in.  The  angle  A  VB 
=  6o»;  B  VC=  45"  /  CVA^  40". 


INTERSECTIONS— CURVED  SURFACES.  I97 

Find  the  plan  of  the  point  V  afid  its  height  above  the  base. 

(i)  Draw  the  triangle  ABC,  and  on  AB  describe  a 
segment  of  a  circle  containing  an  angle  of  60",  on  BC  one 
containing  an  angle  of  45°,  and  on  CA  a  third  containing  an 
angle  of  40".     Euclid,  Book  III.  Prop.  2,2»' 

If  these  segments  be  rotated  about  the  lines  AB,  BC, 
CA  as  axes,  surfaces  of  revolution  will  be  generated  whose 
common  intersection  will  determine  V. 

(2)  Find  the  intersection  of  any  two  of  these  surfaces 
thus : — AB  and  BC  being  the  two  axes  of  the  surfaces 
whose  intersection  we  require,  from  B  with  any  radius 
describe  a  circle  cutting  the  segment  described  on  AB  in  /, 
and  the  segment  on  ^C  in  ^. 

If  the  segment  on  AB  revolve  about  that  line  as  an  axis, 
the  point/  will  trace  a  circle,  in  a  plane  perpendicular  to 
AB,  on  the  surface  of  a  sphere  with  centre  B  and  radius  Bp. 
The  plan  of  the  locus  of/  is,  therefore,  a  straight  line  drawn 
from  /  at  right  angles  to  AB  and  produced  to  an  equal 
distance  on  the  other  side  of  that  axis. 

Similarly,  the  plan  of  the  locus  of  q — which  also  traces  a 
circle  on  the  above-mentioned  sphere — will  lie  in  the  perpen- 
dicular from  g  io  BC 

The  point  of  intersection  of  the  plans  of  the  loci  is  a 
point  in  the  plan  of  the  intersection  of  the  two  surfaces  of 
revolution. 

In  like  manner  any  number  of  points  may  be  found 
in  the  plan  of  the  intersection,  and  the  latter  line  drawn 
through  them. 

(3)  Find  the  intersection  of  the  third  surface  with  one 
of  the  foregoing.  The  point  v,  where  the  plans  of  the  two 
intersections  cross,  is  the  plan  of  the  vertex  V. 


IpS  SOLID   OR  DESCRIPTIVE   GEOMETRY. 

It  may  be  remarked  that  there  are  two  points  answering 
to  V,  one  above,  and  one  below  the  plane  of  the  triangle 
ABC.     The  latter  is  neglected. 

(4)  Complete  the  plan  by  drawing  lines  from  v  to  A,  B, 
and  C,  and  find  the  height  of  Fthus  : — 

Draw  a  line  from  v  perpendicular  to  one  of  the  axes  (say 
AB)  and  produce  it  to  meet  the  segment  of  the  circle  described 
on  this  line  in  the  point  x.  Then  Bx  will  be  the  real  length 
of  the  edge  of  which  Bv  is  the  plan.  From  these  data  the 
height  of  V  can  be  at  once  determined. 

2.  To  find  the  intersection  of  two  surfaces  of  revolution 
when  their  axes  are  not  in  the  same  plane. 

Take  a  series  of  sections  of  both  surfaces  by  auxiliary 
planes  and  construct  the  points  in  which  the  curves  of  section 
meet  for  points  in  the  intersection  of  the  surfaces. 

Note.  This  problem  presents  no  difficulty  except  the 
mechanical  one  of  drawing  the  curves  of  section,  and  to 
simpUfy  this  as  much  as  possible  assume  one  of  the  planes  of 
projection  at  right  angles  to  one  of  the  axes,  and  the  other 
plane  of  projection  parallel  to  both.  The  auxiliary  section 
planes  may  then  be  taken  parallel  to  the  former  plane  of  pro- 
jection and  the  intersection  deduced  without  much  trouble. 

COMBINATION     OF     CURVED     FORMS    WITH 
FORMS  BOUNDED  BY  PLANE  SURFACES. 

I.  Intersection  of  a  right  hexagonal  prism  standing  on 
its  base  of  1  in.  side  with  a  sphere  of  2'<,  in.  diameter.  One 
face  of  the  prism  making  an  angle  ^20"  with  xy;  centre  of 
sphere  2  in.  above  H.P.  and  in  plan  '5  in.  from  the  plan  of  the 
axis  of  the  prism. 


INTERSECTION'S—  COMB  IN  A  TIONS.  1 99 

Take  auxiliary  sections  of  the  sphere  by  a  series  of 
horizontal  planes  in  pairs  at  equal  distances  above  and 
below  the  centre  of  the  sphere. 

The  plan  of  the  intersection  coincides  with  the  base  of 
the  prism.  The  elevation  can  be  found  by  projection  from 
the  points  furnished  in  plan  by  the  auxiliary  sections. 

Note.  Any  surface  of  revolution  with  vertical  axis, 
or  oblique  cylinder  or  cone  with  circular  horizontal  trace, 
might  be  substituted  for  the  sphere  in  this  problem. 

2.  Arrange  an  example  of  a  right  square  prism,  axis 
horizontal  and  inclined  to  V.F.,  intersecting  a  right  cylinder, 
axis  vertical. 

Divide  each  edge  of  the  base  in  plan  and  in  elevation 
into  a  convenient  number  of  similar  parts,  and  draw  through 
the  points  of  division  a  series  of  lines  in  each  face  parallel 
to  the  long  edges  of  the  prism. 

The  plan  of  the  intersection  coincides  with  the  base  of 
the  cylinder.  For  the  elevation,  it  will  be  necessary  to  draw 
projecting-lines  from  the  points  where  the  parallel  lines  in 
plan,  and  the  edges  of  the  prism,  cross  the  circle,  to  the 
corresponding  lines  in  elevation.  Special  attention  should 
be  given  to  the  edges  of  the  prism,  which  should  be  put  in 
dark  or  dotted,  or  partly  rubbed  out  as  they  require,  before 
joining  the  points  for  the  curves  of  intersection. 

Note.  The  same  construction  would  apply  were  the 
prism  oblique. 

3.  Arrafige  an  oblique  pyramid  intersecting  a  vertical 
right  cylinder. 

Divide  each  edge  of  the  base  of  the  pyramid  as  before, 
and  draw  the  lines  to  the  vertex.  The  rest  of  the  work  is 
the  same  as  in  the  preceding  problem. 


200  SOLID   OR  DESCRIPTIVE   GEOMETRY. 

4.  Arrange  a  right  pentagonal  prism  standing  on  its  base 
intersecting  a  horizontal  right  cylijider. 

Use  horizontal  lines  (generatrices)  on  the  surface  of  the 
cylinder  as  in  previous  cases  of  the  intersection  of  the 
horizontal  cylinder. 

Pay  attention  to  the  key  points  and  critical  lines. 

Note,  The  cone  might  be  substituted  for  the  cylinder 
in  this  problem,  in  which  case  the  lines  would  be  drawn 
from  the  points  of  division  of  the  base  to  the  vertex. 

5.  Arrange  a  right  pentagonal  prism  standing  on  its  base 
intersecting  a  right  cone,  axis  oblique  to  both  planes  of  projectioji. 

6.  Intersection  of  Cone  and  Prism. 

Prism,  axis  horizontal,  4  in.  long,  and  inclined  -^d^  to  V.P., 
bases  equilateral  triangles  of  2  in.  side,  lowest  face  inclined  1 5°, 
and  lowest  edge  "25  in.  above  H.P. 

Cone,  axis  vertical,  3  in.  long,  and  passing  through  the 
middle  point  of  the  axis  of  the  prism,  base  1*5  in.  radius. 

Draw  a  series  of  horizontal  lines  in  each  face  of  the 
prism,  as  in  the  prism  Prob.  2  above. 

Horizontal  planes,  passing  through  these  lines  and 
cutting  the  cone  in  circular  sections  parallel  to  its  base,  may 
now  be  taken,  and  the  points  in  which  the  plans  of  the 
horizontal  lines  and  circular  sections  of  the  same  levels 
meet,  joined  up  for  the  plan  of  the  intersection.  The 
elevation  can  be  completed  by  projecting. 

7.  Arrange  a  square  prism,  axis  horizontal,  intersecting  a 
sphere. 

Same  method  as  in  Prob.  6.  Any  surface  of  revolution 
with  vertical  axis,  or  an  oblique  cone  with  a  circular  trace, 
can  be  substituted  for  the  sphere. 


I 


INTERSECTIONS—SHADOWS.  201 

8.  Arrange  a  square  prism,  axis  horizontal  or  oblique, 
intersecting  a  right  cone,  axis  inclined  to  both  planes  of 
projection. 

Prepare  the  prism  by  drawing  lines  in  its  faces  parallel 
to  the  edges,  as  in  other  problems,  and  use  section  planes 
containing  these  lines  and  the  vertex  of  the  cone. 

Note.  An  oblique  cylinder  might  be  substituted  for 
the  cone,  but  the  vertex  being  at  an  infinite  distance  the 
auxihary  planes  would  be  parallel. 

9.  Arrange  a  sphere  intersecting  an  irregular  pyramid. 

Use  a  series  of  horizontal  auxiliary  sections  of  both 
forms. 

SHADOWS. 

In  a  homogeneous  medium  light  travels  in  straight  lines, 
called  rays,  which  are  parallel,  divergent,  or  convergent, 
according  to  the  source  whence  the  light  proceeds  and  the 
configuration  of  the  media  through  which  it  has  passed. 

The  sun's  rays  are  sensibly  parallel ;  rays  from  the 
electric  light  are  divergent,  and,  for  all  practical  purposes, 
may  be  considered  as  diverging  from  a  point ;  while  rays  of 
light  that  have  passed  .through  a  double  convex  lens,  for 
example,  converge  towards  a  point  ox  focus. 

If  an  opaque  body  be  placed  between  a  source  of  light 
and  the  object  or  objects  illuminated,  some  of  the  rays  will 
be  received  upon  the  interposed  body  and  will,  consequently, 
be  cut  off  from  illuminating  certain  parts  of  the  objects 
beyond.  Popularly  the  opaque  body  would  be  said  to 
throw  or  cast  a  shadow.  There  is,  however,  an  ambiguity 
in  the  term   shadow  as  commonly  employed,  which  it  is 


202  SOLID   OR  DESCRIPTIVE    GEOMETRY. 

convenient,  when  treating  of  the  projection  of  shadows,  to 
avoid  by  suitable  nomenclature. 

It  will  be  evident  after  a  little  consideration  that,  when 
the  light  from  a  given  source  is  partly  intercepted  by  an 
opaque  body,  there  is  carved  as  it  were  out  of  the  surround- 
ing bundle  of  luminous  rays,  a  central  part  or  core  of 
darkness  of  a  definite  form,  the  surface  of  which  derives  its 
configuration  from  the  interposed  body.  To  the  surface 
which  separates  this  core  from  the  surrounding  rays  of  light 
the  term  shadow-surface  will  be  given.  The  line  of  inter- 
section of  the  shadow-surface  with  the  forms  or  surfaces  it 
meets  will  be  called  the  shadow-trace.  The  figure  bounded 
by  the  shadow-trace  is  plainly  the  shadow  cast  by  the 
interposed  body.  The  shade-line,  or  line  of  contact  of  the 
shadow- surface  with  the  form  whose  shadow  is  projected, 
marks  the  separation  of  light  and  shade  thereon.  The  shade- 
line  is  oftentimes  an  edge  or  boundary  of  the  form  itself. 

From  these  considerations  it  is  apparent  that  the  problem 
for  the  student  in  any  given  case  is  to  find  the  shadow- 
surface,  and  its  intersection  with  the  surfaces  on  which  the 
shadow  falls.  Sometimes  the  shadow-surface  is  treated  as 
the  boundary  of  a  definite  form,  at  others  a  convenient 
number  of  lines  (rays)  are  drawn  in  the  shadow-surface 
from  points  in  the  shade-line,  and  their  intersections  deter- 
mined with  the  forms  or  surfaces  on  which  the  shadow  is 
cast;  but  in  both  cases  the  problem  is  one  of  "Sections  by 
Planes"  (oblique  or  otherwise)  or  of  "Intersections." 

It  may  be  remarked  that  the  shadows  we  deal  with  are 
purely  geometrical,  and  their  boundaries,  or  traces,  sharp 
distinct  lines.  Those  actually  seen  in  nature,  called  physi- 
cal shadows,  are  more  or  less  hazy  about  the  region  of  the 


INTERSECTIONS— SHADOWS.  203 

shadow-trace,  due  partly  to  inflexion  and  partly  to  the 
formation  of  a  penumbra — a  fringing  semi-shadow  which 
makes  its  appearance  when  the  source  of  light  has  an 
appreciable  angular  magnitude. 

I.     Shadows  cast  by  Parallel  Rays. 

The  rays  of  light  are  in  all  cases  represented  by  straight 
lines  given  in  direction. 

One  of  the  parallel  rays  being  determined  by  its  plan 
and  elevation,  the  shadow-surface  is  supposed  everywhere 
parallel  to  the  given  ray  and  enveloping  the  form  whose 
shadow  is  to  be  projected. 

For  example,  if  the  form  is  a  sphere  the  shadow-surface 
is  the  enveloping  cylinder  whose  axis  is  the  line  drawn 
from  the  centre  of  the  sphere  parallel  to  the  given  ray. 
The  shadoiv-trace  is  the  intersection  of  this  cylinder  with 
the  forms  or  surfaces  on  which  the  shadow  falls.  The 
"circle  of  contact"  is  the  shade-line  which  separates  the 
part  of  the  sphere  in  light  from  the  part  in  shade. 

Taking  in  lieu  of  the  sphere  a  right  cone  in  space,  axis 
oblique  to  the  co-ordinate  planes,  its  shadow-surface  would 
be  partly  determined  by  a  pair  of  tangent  planes  parallel  to 
the  ray,  thus : — Draw  a  line  from  the  vertex  of  the  cone 
parallel  to  the  given  ray  and  determine  two  tangent-planes  to 
the  conic  surface  containing  this  line.  The  planes  so  drawn 
form  a  portion  of  the  shadow-surface  required,  and  if  the 
plane  of  the  base  of  the  cone  were  parallel  to  the  given  ray, 
that  plane  produced  would  intersect  the  two  tangent-planes 
in  parallel  lines,  giving  rise  to  a  triangular  prism  as  the 
complete  shadow-surface.  The  intersections  of  this  prism 
with  the  surfaces  or  forms  on  which  the  shadow  falls  will 


204  SOLID   OR  DESCRIPTIVE   GEOMETRY. 

give  the  shadow,  and  shadow-trace,  as  in  other  cases.  The 
"lines  of  contact"  of  the  tangent-planes,  and  the  arc  of  the 
base  that  lies  between  them  on  the  portion  of  the  conic 
surface  turned  towards  the  light,  form  the  shade-line.  Were 
the  plane  of  the  base  not  parallel  to  the  given  ray,  this  part 
of  the  shade-line  would  be  treated  as  the  directrix  of  a 
cylindrical  surface ;  that  is,  points  would  be  assumed  in  it,- 
and  lines  drawn  from  them  parallel  to  the  given  ray,  and  the 
intersection  of  these  lines  with  the  objects  they  meet  deter- 
mined for  points  in  the  shadow-trace.  Generally,  lines  drawn 
parallel  to  the  given  ray  from  assumed  points  in  the  shade- 
line  will  intersect  the  plane  or  curved  surfaces  on  which  the 
shadow  is  cast  in  points  which  determine  the  shadow-trace. 
In  many  cases  a  few  points  suffice.  For  example,  in  finding 
the  shadow  cast  on  the  horizontal  plane  by  a  cube,  it  will 
be  seen  that  the  shadow-surface  is  bounded  by  planes 
parallel  to  the  given  ray  containing  certain  edges  (those 
forming  the  shade-line)  of  the  solid,  and  that  the  shadow- 
trace  is  made  up  of  parts  of  the  horizontal  traces  of  these 
planes.  If,  therefore,  two  lines  be  drawn  from  the  extremi- 
ties of  one  of  these  edges  parallel  to  the  given  ray,  and  their 
horizontal  traces  found  and  joined  by  a  straight  line,  this 
line  will  be  one  side  of  the  shadow-trace. 

Exercises. 

I.  The  shadow  of  a  cone  on  a  cylinder  and  of  both  on  the 
H.P.  Cylinder  resting  on  If. P.,  axis  horizontal  and  inclined 
at  30°  to  V.F.,  base  1*5  in.  diameter.  Cone  resting  on  H.P.^ 
base  2  in.  diameter,  axis  4*5  in.  long,  vertical  and  1*5  in.  from 
nearest  generatrix  of  cylinder.  Rays  inclined  45°  and  in  plan 
at  right  angles  to  the  plan  of  the  axis  of  the  cylinder.  Show 
the  shade-lines  on  both  surfaces. 


INTERSECTIONS— SHADOWS.  205 

2.  A  circular  slab  3  m,  diameter^  '4  in.  thick,  rests  symme- 
trically over  the  upper  base  of  a  right  hexagonal  prism,  axis 
2*5  in.,  side  of  base  v^.  Vertical  face  of  prism  inclined  at 
20°  to  V.P.  Shadow  of  the  slab  on  the  prism  and  of  both  on 
the  H.P.  by  rays  of  light  making  angles  of  45"  with  xy  in 
plan  and  elevation. 

■  3.  A  right  cone  axis  4  in.  long,  base  2  in.  diameter,  resting 
upon  H.P.  with  the  rim  of  its  base  touching  the  V.P.  and  the 
plan  of  its  axis  making  an  angle  of  60°  with  xy,  is  truncated 
by  a  plane  at  right  angles  to  the  axis  and  1  in.  from  the  vertex. 
Shadow  of  frustum  by  rays  of  light  inclined  45"  to  xy  in 
elevation  and  50**  in  plan. 

4.  Assume  an  oblique  prism  and  a  sphere,  and  show  the 
shadow  cast  by  the  sphere  on  the  prism. 

5.  A  niche  is  formed,  in  a  vertical  wall  parallel  to  the 
V.P,  by  a  semi-cylindrical  recess  of  2'^  in.  diameter  capped  by 
a  semi-hemispherical  dome.  Show  the  shadow  in  the  niche 
as  cast  when  the  rays  of  light  are  inclined  in  plan  and  elevation 
at  equal  angles  of  4^^^  to  xy. 

II.    Shadows  cast  by  Rays  that  meet  in  a  Point. 

This  division  includes  the  projection  of  shadows  cast 
by  rays  divergent  and  convergent.  The  former  is  the  only 
case  that  will  receive  attention,  inasmuch  as  whatever  is  said 
concerning  the  one,  is,  with  very  obvious  modifications, 
equally  applicable  to  the  other. 

The  source  of  light  is  a  point,  given  by  its  projections, 
from  which  the  rays  diverge  in  every  direction. 

A  line   passing   through   the   given   point  and  moving 


206  SOLID  OR  DESCRIPTIVE   GEOMETRY. 

round  the  body  whose  shadow  is  to  be  projected,  generates 
the  shadow- surface  and  determines  the  shade-line. 

Thus,  the  shadow- surface  of  a  sphere  is  an  enveloping 
cone  having  its  vertex  in  the  given  luminous  point.  The 
"circle  of  contact"  will  be  the  line  of  shade,  and  the 
shadow-trace  will  be  determined  in  any  given  case  by  the 
methods  specially  applicable  to  the  intersection  of  the  cone 
and  the  particular  form  or  surface  on  which  the  shadow  is 
cast. 

If  the  cone  selected  as  an  illustration  under  parallel 
rays  be  taken,  the  work  is  substantially  the  same  as  there 
given.  The  two  tangent-planes  to  the  cone  that  pass 
through  the  given  luminous  point  constitute  a  portion  of  the 
shadow-surface,  the  rest  of  which  will  be  a  cone  having  the 
luminous  point  as  vertex  and  a  segment  of  the  base  as 
directrix.  Any  required  points  in  the  shadow-trace  can 
always  be  found  by  lines  from  the  vertex,  through  points  in 
the  shade-line,  produced  to  meet  the  objects  on  which  the 
shadow  falls. 

The  shadow-surface  of  a  solid  bounded  wholly  by  plane 
figures  will  be  a  more  or  less  irregular  pyramid  having  its 
vertex  in  the  luminous  point.  The  plane  faces  of  this 
pyramid  will  contain  certain  of  the  edges  of  the  form  whose 
shadow  has  to  be  projected,  and  in  this  case,  as  in  the  case 
of  parallel  rays,  the  work  may  sometimes  necessitate  that 
two  lines  only  be  drawn  from  the  point — one  through  each 
extremity  of  an  edge— to  determine  one  side  of  the 
shadow-trace. 

Exercises. 
I.     Arrange  a   right  cone  standing  on   its  base  and  a 
cylindrical  slab  resting  on  H.P.  with  its  axis  vertical,  and 


INTERSE  CTIONS—SHAD  0  WS.  207 

show  the  shadow  cast  by  the  cone  upon  the  cylinder,  and  of 
both  upon  H.P.,  by  rays  diverging  from  an  assumed  point  in 
space. 

2.  Assume  an  oblique  plane,  a  vertical  circular  disc,  and 
a  point  in  space,  and  shotv  the  shadow  of  the  disc  upon  the 
oblique  plane  as  cast  by  rays  of  light  diverging  from  the  point. 

3.  A  hemispherical  bowl  rests  upon  the  H.P.  with  the 
cavity  upwards,  shoiv  the  shado7V  cast  inside,  the  shadow  of 
the  hemisphere  on  the  H.P.  and  the  line  of  shade,  when  the 
rays  radiate  from  an  assumed  point  in  space. 

4.  Arrange  an  example  showing  the  shadow  of  a  sphere 
cast  upon  an  oblique  pentagonal  pyramid  by  rays  diverging 
from  a  point. 

III.    Shadows  cast  bv  Luminous  Bodies  of  appreciable 
Angular  Magnitude. 

For  example,  suppose  the  luminous  body  to  be  the  sun, 
and  the  body  whose  shadow  is  cast  the  moon.  There  are 
in  this  case  two  shadow-surfaces,  y'lt.  the  two  enveloping 
cones  j  the  trace  of  the  inner  surface  bounding  the  true 
shadow,  and  that  of  the  outer  surface  bounding  the  pen- 
umbra. 

The  arrangement  of  a  few  examples  to  illustrate  the 
projection  of  the  penumbra  must  be  left  to  the  ingenuity  of 
the  student.  One  that  may  be  suggested,  is  the  shadow  of 
a  disc  by  a  white-hot  platinum  wire  of  a  given  length. 
The  source  of  light  in  this  case  being  treated  as  a  line  of 
ight,  show  the  section  of  the  shadow-surfaces  by  an 
oblique  plane. 


X, 


FIGURED  PROJECTIONS  AND  SCALES  OF 
SLOPE. 

Def.  The  index  of  a  point  is  a  number  affixed  to  its 
projection  on  a  plane  to  denote  the  distance  of  the  point 
from  that  plane. 

K  point  in  space  is  completely  determined  when  its  plan 
and  the  affixed  index  denoting  the  length  of  its  projector  are 
given.  Similarly,  a  straight  line  is  determined  by  its  plan 
and  the  indices  denoting  the  length  of  the  projectors  of  any 
two  fixed  points  in  it.  The  height  of  any  other  point  in 
such  a  line,  is,  of  course,  readily  determinable  by  a  rabat- 
ment  of  the  projecting  plane  of  the  line  into  the  plane  of 
projection. 

A  minus  sign  is  prefixed  to  indices  that  denote  points 
below  the  plane.  Points  above  the  plane  are  indexed  by 
numbers  in  the  usual  way,  the  positive  sign  being  under- 
stood. The  index  of  a  point  in  the  plane  is  of  course  always 
zero. 

Def.  Points,  lines,  figures,  and  forms,  determined  by 
one  projection  and  its  affixed  indices,  are  said  to  be  given 
by  XhQix  figured  plans. 

A  plane  is  determined  when  three  contained  points,  or 


FIGURED  PLANS  AND   SCALES   OF  SLOPE.      209 

two  straight  lines  (meeting  or  parallel)  in  it,  are  given  by 
their  figured  plans.  But  for  convenience  in  representing  in- 
definite planes  by  the  method  of  figured  plans,  recourse  is  had 
to  a  device  called  the  Scale  of  Slope,  the  nature  of  >vhich  will 
be  readily  understood  from  the  following  considerations : — 

If  a  pair  of  horizontal  lines  be  drawn  in  a  given  plane, 
they  will  both  be  parallel  to  its  horizontal  trace,  and,  there- 
fore, to  one  another.  Any  two  such  horizontals  (one  of  which 
might  be  the  H.T.  itself),  if  properly  indexed,  would  deter- 
mine the  plane.  A  line  drawn  at  right  angles  to  the  figured 
plans  of  the  pair  of  horizontals  might  be  the  plan  of  a  line 
measuring  the  inclination  of  the  plane,  and  from  the  known 
indices  of  the  two  points  where  the  line  of  inclination  meets 
the  pair  of  parallels,  a  number  of  other  indexed  points 
(denoting  a  series  of  horizontals)  might  be  at  once  deter- 
mined in  the  line  of  inclination,  and  a  scale  showing  heights 
in  the  plane  constructed.  This  is  what  is  done  in  the  Scale 
of  Slope,  which  is  always  drawn  as  a  double  line  with  an 
interspace  of  about  ^g  th  of  an  inch.  The  line  on  the  left- 
hand  side  of  the  scale  in  the  ascending  direction  should  be 
made  thicker  than  the  other,  as  a  convention  to  assist  the 
eye  in  reaching  the  drawings  in  which  Scales  of  Slope  are 
employed. 

A  circle  is  completely  determined  when  its  plan  and  the 
index  of  its  centre  are  given.  Also,  a  sphere  is  similarly 
determined  when  its  plan  and  the  index  of  its  centre  are 
given. 

A  few  problems  to  be  worked  by  the  method  of  figured 
plans  follow.  The  student  who  wishes  for  others  will  have 
no  difficulty  in  selecting  good  examples  from  among  the 
exercises  already  given  for  solution  in  the  usual  way. 

E.  G.  14 


2IO  SOLID  OR  DESCRIPTIVE   GEOMETRY. 

Problems. 

1.  An  equilateral  triangle  abc  of  t,  in.  side,  is  the  plan 
of  another  triangle  whose  corners  A,B,C,  are  i  in.,  1*5  in.,  and 
2*5  in.  respectively  above  H.P,;  determine  the  Scale  of  Slope  of 
the  plane  of  the  triangle  ABC,  and  the  index  of  a  point,  P,  in 
this  plane  of  which  the  pla?i,  p,  is  the  centre  of  the  equilateral 
triangle. 

Find  in  ac  the  plan  of  a  point  having  the  same  index  as 
B  {i.e.  i'5).  A  line  from  b  through  this  point  will  be  the 
plan  of  a  line  of  level  in  the  plane  of  the  triangle.  Draw 
the  pair  of  lines  for  the  Scale  of  Slope  at  right  angles  to  this 
line.  A  second  horizontal  from  ^  or  C  will  determine 
another  point  of  known  height,  whence  the  Scale  of  Slope 
may  be  completed  by  divisions  showing,  for  example, 
differences  of  level  of  (say)  ^th  of  an  inch. 

A  horizontal  from  p  to  the  Scale  of  Slope  will  ascertain 
the  index  of  P. 

2.  To  find  the  index  ofV  without  recourse  to  the  Scale  of 
Slope. 

Join  one  of  the  points,  as  a,  with  p,  and  produce  the  line 
ap  to  meet  be  in  0.  Find  the  index  of  0  in  JBC,  and  then 
of  P  in  A  O,  by  rabatting  the  projecting  planes  of  these  lines. 

3.  To  find  the  true  shape  of  the  triangle  ABC. 

Rabat  the  plane  about  one  of  the  horizontals  till  parallel 
to  H.P. 

4.  Assume  two  inclined  planes  by  their  Scales  of  Slope, 
and  deter?nine  the  figured  plan  of  the  intersection,  and  its 
inclination. 

Draw  in  the  one  plane  a  pair  of  horizontals  through  any 


FIGURED  PLANS  AND  SCALES  OF  SLOPE.      2ll 

two  points  of  division  in  its  Scale  of  Slope.  A  similar  pair 
in  the  second  plane,  having  the  same  indices  as  those  first 
drawn,  will  meet  them  in  points  in  the  plan  of  the  inter- 
section, whence  the  line  may  be  indexed,  and  its  inclination 
determined  by  rabatting  its  projecting  plane. 

5.  Given  a  plane  by  its  Scale  of  Slope  and  a  straight  line 
by  its  figured  plan,  find  the  point  of  intersection  of  the  plane 
and  line. 

Assume  any  coriprenient  plane  containing  the  line,  and 
determine  its  intersection  with  the  given  plane.  The  point 
in  which  the  plan  of  the  line  meets  the  plan  of  the  inter- 
section of  the  planes  will  be  the  point  required. 

6.  Given  a  regular  hexagonal  pyramid  by  its  figured 
plan,  and  an  inclined  section  plane  at  60"  by  its  Scale  of  Slope, 
determine  the  figured  plan  and  true  shape  of  the  section. 

7.  Assutne  two  lines,  neither  parallel  nor  meeting,  by 
their  figured  plans,  and  determine  the  surface  generated  by  a 
third  straight  line,  which  moves  so  as  always  to  meet  the  two 
given  lines  and  be  parallel  to  the  plajie  ofprojection. 

This  surface,  which  is  the  Hyperbolic-Paraboloid,  is 
determined  by  drawing  a  series  of  generatrices  from  points 
in  one  directrix  to  points  in  the  other  having  similar 
indices. 

8.  Work  a  section  of  the  above  surface  by  an  inclined 
Mane  assumed  by  its  Scale  of  Slope. 

9.  Work  Prob.  7  above  when  the  directing  plane  is  not 
^he  plane  ofprojection,  but  is  given  by  its  Scale  of  Slope. 

10.  An  ellipse,  which  is  the  figured  plan  of  a  circle,  and 
in  inclined  line,  are  the  directrices  of  a  Qonoidal  surface,  the. 

14—2 


212  SOLID  OR  DESCRIPTIVE  GEOMETRY. 

plane  of  projection  being  the  directing-plane^  or  plane  to  which 
the  generatrices  are  parallel;  determine  the  surface,  and  the 
section  by  an  inclined  plane  givejt  by  its  Scale  of  Slope. 

Note,  When  working  Probs.  7,  8,  9,  and  10,  the 
student  should  refer  to  remarks  on  "Undevelopable  Ruled 
Surfaces,"  Chap.  vii. 

II.  To  determine  by  the  method  of  parallel  sections  the 
intersection  of  two  or  more  forms  with  plane  faces.,  when  the 
forms  are  given  by  their  figured  plans. 

To  work  also  a  section  by  a  given  vertical  plane. 

For  the  first  example  take  two  irregular  polygons  with 
several  edges  of  the  one  cutting  those  of  the  other,  and 
assume  them  to  be  the  bases  of  two  pyramids  resting  upon 
H.P.  Assume  two  points  for  the  plans  of  the  vertices  and 
attach  indices  to  denote  their  heights  above  the  plane  of 
projection.  Join  the  plans  of  the  vertices  with  the  corners 
of  the  corresponding  bases  for  the  complete  plans  of  the 
intersecting  forms. 

To  work  the  intersection: — A  little  attention  bestowed 
upon  the  plan  and  the  heights  of  the  vertices  will  make  it 
evident  what  faces  intersect,  and  the  general  direction  of  the 
intersections.  Sections  of  any  pair  of  intersecting  faces  by 
two  horizontal  planes  will  determine  two  points  in  the  inter- 
section of  the  indefinite  planes  of  those  faces,  through  which 
points  the  line  of  intersection  can  be  drawn.  The  horizon- 
tal plane  of  projection  itself  will  serve  as  one  of  the  auxiliary 
section  planes.  Thus,  the  points  in  which  the  polygonal 
bases  meet  will  be  points  in  the  intersection  of  the  faces 
passing  through  them.  To  find  another  set  of  points,  take 
an  auxiliary  section  of  both  forms  by  a  horizontal  plane  at  a 
•  convenient  height ;  that  is,  draw  horizontal  lines  at  the  same 


FIGURED  PLANS  AND  SCALES  OF  SLOPE.      213 

level  in  each  pair  of  intersecting  faces.  The  points  in  w?iich 
the  plans  of  these  lines  at  the  same  level  meet,  will  be  the 
plans  of  the  points  required. 

When  two  points  in  each  line  of  intersection  are  found, 
the  segment  of  the  indefinite  intersection  that  is  common  to, 
and  limited  by  the  intersecting  faces,  may  be  put  in,  and  so 
on  till  the  whole  intersection  is  complete. 

Note.  It  is  sometimes  necessary  to  produce  the  sides 
of  the  bases,  or  the  lines  of  level,  beyond  the  limits  of  the 
intersecting  faces,  in  order  to  determine  conveniently  the 
required  segment  of  the  indefinite  intersection. 

Work  is  sometimes  saved  by  taking  the  horizontal  lines 
in  the  planes  of  the  faces  of  the  pyramid  with  the  higher 
vertex,  at  the  same  level  as  the  vertex  of  the  lower  one. 

For  another  exercise  take  two  pyramids,  one  with  its  base 
resting  on  H.P.  and  the  other  inclined. 

The  intersection  is  found,  as  before,  by  means  of  lines  of 
level  in  the  intersecting  faces,  but  in  this  case  the  inclined 
base  has  to  be  taken  into  consideration.  Generally  it  is 
best  to  begin  with  the  section  of  the  one  pyramid  by  the 
inclined  plane  of  the  base  of  the  other,  and  it  sometimes 
simplifies  matters  if  the  horizontal  trace  of  the  latter  form  is 
found  before  working  the  rest  of  the  intersection. 

To  work  the  section  by  the  given  vertical  plane  it  is  only 
necessary  to  find  the  indices  of  the  points  where  the  line 
which  is  the  plan  of  the  section  plane  cuts  the  edges  and 
lines  of  intersection  of  the  forms.  The  section-plane,  with 
these  points  of  section,  can  then  be  rabatted,  and  the  true 
shape  of  the  section  determined. 

12.     Find  the  Scale  of  Slope  of  a  plane  inclined  at  40", 


214  SOLID   OR  DESCRIPTIVE   GEOMETRY. 

and  the  plan  and  true  shape  of  the  section  cut  by  it  from  the 
intersecting  pyramids  in  the  preceding  problem. 

13.  Determine  the  intersection  of  a  right  cone  and  an 
irregular  Pyramid,  both  resting  upon  H.P.,  by  the  7nethod  of 
parallel  sections,  using  only  the  figured  plans  of  the  forms. 

14.  A  cube,  diagonal  of  the  form  vertical,  stands  with  its 
lowest  corner  on  the  horizontal  plane.  Determine  the  Scales  of 
Slope  of  the  planes  of  the  three  faces  meeting  in  that  corner. 


XL 


TRIMETRIC  METHODS  OF  PROJECTION. 

In  the  preceding  chapter  a  method  is  given  of  working 
with  figured  plans,  by  which  the  necessity  for  a  second 
projection  on  a  vertical  plane  is,  in  a  measure,  obviated.  It 
is  intended  to  discuss  here  some  developments  of  this 
method,  one  of  which  is  of  considerable  practical  utility 
from  the  ease  with  which  it  can  be  applied  to  the  construc- 
tion of  very  intelligible  drawings. 

All  forms  can  be  completely  determined  by  referring 
them  to  a  system  of  three  rectangular  planes.  Such  a  system 
of  reference  planes  intersect  in,  and  are  themselves  deter- 
mined by  three  rectangular  axes  meeting  in  a  point. 
Inasmuch  as  the  forms  most  common  in  practical  construc- 
tions are  chiefly  developed  in  the  three  directions  of  length, 
breadth,  and  thickness,  it  is  obvious  they  might  be  readily 
referred  to  such  a  system  of  rectilineal  axes,  and  the  lines 
for  their  drawings  laid  off  about  the  projections  of  these 
axes  on  a  plane  oblique  to  them.  The  following  consider- 
ations will  make  this  clear  : — 

If  the  three  rectangular  axes  were  projected  on  a  plane 
oblique  to  all  and  differently  inclined  to  each,  three  scales 
(one  for  each  axis)  might  be  constructed  to  show  the 
projected  lengths  of  the  unit  when  set  off  in  the  three  several 
directions  of  the  axes.     Drawings  made  with  reference  to 


2l6  SOLID   OR  DESCRIPTIVE   GEOMETRY. 

these  axes  and  their  scales,  would  be  true  orthographic 
projections,  showing  at  a  glance,  in  one  projection,  the  three 
principal  directions  in  which  the  forms  projected  are  deve- 
loped, and  affording  a  means  of  readily  ascertaining  their 
dimensions.  Such  drawings  might  be  conveniently  called 
"Trimetric  Projections."  If  two  of  the  axes  were  equally 
inclined  to  the  plane  of  projection  at  angles  different  from 
the  third,  two  scales  only  would  be  required,  and  the  some- 
what simplified  projection  might,  in  this  sense,  be  called 
Dimetric.  In  the  particular  case  in  which  the  three  axes 
are  equally  inclined,  the  projection  is  monometric,  or,  as  it  is 
usually  termed,  "Isometric" — only  one  scale  being  required 
for  the  three  directions. 

For  some  purposes  {e.g.  the  projection  of  the  forms  of 
crystals)  it  is  convenient  to  regard  the  intersections  of  the 
three  planes  as  axes  of  symmetry.  In  the  forms  of  the 
cubic  system,  for  example,  since  the  faces  can  be  disposed 
symmetrically  about  three  such  axes  of  equal  length,  any 
conventional  sign  denoting  one  face  equally  symbolises  the 
whole  form,  and  if  the  data  for  fixing  one  face  be  given  the 
projection  of  the  complete  form  can  be  readily  determined. 
This  method  is  a  particular  case  of  Trimetric  Projection,  the 
application  of  which  to  the  projection  of  symmetrical 
forms  will  be  touched  upon  briefly  under  the  heading  of 
"Axial  Projection." 

TRIMETRIC  PROJECTION. 

Before  the  exercises  can  be  attempted  the  student  must 
.solve  the  following  problem  : — 

Draw  three  straight  lines,  ox,  oy,  oz,  forming  three  unequal 
obtuse  angles  xoy,  yoz,  zox,  at  a  pointy  o,  and  assuming  these 


TRIMETRIC  projection:  217 

lines  to  be  the  plans  of  the  three  rectangular  axes,  OX,  OY, 
OZ,  determine  the  set  of  trimetric  scales  corresponding  to 
them. 

Read  Prob.  XXVII.,  Chap.  iii. 

It  will  be  necessary  first  to  find  the  inclination  of  each 
axis  to  the  plane  of  projection.  This  done,  the  scales  can 
be  readily  constructed. 

To  find  the  inclination  of  OX  : — Since  OX  is  perpendi- 
cular to  the  plane  of  YOZ,  the  horizontal  trace  of  this  plane 
will  be  at  right  angles  to  ox,  the  projection  of  OX.  Draw, 
therefore,  an  assumed  horizontal  trace  of  the  plane  YOZ  at 
right  angles  to  ox  and  meeting  oy,  oz,  in  s  and  t  respectively. 
Similarly,  from  s  and  t  draw  the  horizontal  traces  sr  and  tr 
of  the  planes  XO  V,  XOZ,  meeting  ox  in  r. 

Assume  the  vertical  plane  containing  ox  to  be  rabatted 
thus  : — Produce  xo  to  meet  st  in  m,  and  on  rm  as  diameter 
describe  a  semicircle.  A  line  at  right  angles  to  rm  from  o 
will  cut  the  semicircle  in  O' — the  point  O  rabatted — and 
the  angle  O'ro  will  be  the  inclination  of  OX  required. 

Similarly  the  inclinations  of  OY  a.nd  OZ  may  be  found. 

To  construct  the  trimetric  scale  of  OX  : — Draw  two  lines 
making  an  angle  equal  to  the  inclination  of  OX :  on  one  of 
them  lay  off  the  divisions  of  the  scale  of  units  employed 
(inches  in  the  following  exercises),  and  from  the  points  of 
division  draw  a  series  of  perpendiculars  to  the  other  line. 
These  will  determine  the  divisions  of  the  trimetric  scale  of 
OX.  Work  similarly  for  the  others,  and  finish  the  scales 
neatly  to  show  inches  and  tenths.  The  scales  must  be 
carefully  named  to  guard  against  the  error  which  would 
result  from  interchanging  them. 


2l8  SOLID   OR  DESCRIPTIVE   GEOMETRY. 


Exercises. 

1.  T?'imetric  projection  of  a  brick,  9  in.  long,  4*5  wide, 
and  3  in.  thick,  to  half  scale. 

Measure  the  dimensions  (taken  from  the  trimetric 
scales)  along  the  corresponding  axes,  and  complete  by  draw- 
ing lines  parallel. 

2.  Trimetric  projection  of  a  right  cylinder,  3*5  in  long 
and  2  in.  diameter. 

Assuming  the  cylinder  to  be  circumscribed  by  a  square 
prism,  determine  the  projection  of  the  latter.  Two  ellipses 
inscribed  in  the  trimetric  projection  of  the  square  ends  will 
be  the  projections  of  the  circular  bases  of  the  cylinder. 
Two  parallel  tangents  to  the  elHpses  will  complete  the 
required  projection. 

3.  Show  the  trimetric  projection  of  a  circular  hole  of 
2  in.  diameter  bored  through  the  middle  of  the  brick.  Exercise  i., 
at  right  angles  to  its  large  faces. 

4.  Trimetric  projection  of  your  T-square  to  a  convenient 
scale. 

5.  Trimetric  projection  of  a  mortise  and  tenon  joint. 

6.  ''  Trimetric  projection  of  your  instrument  case  with  the 
lid  open  at  right  angles. 

7.  A  large  central  semicircular  arch-way,  "^oft.  span  and 
40  ft.  high  to  the  springing  of  the  arch,  is  flanked  by  two 
smaller  archways — one  each  side — of  10  //.  span  and  25  //. 
to  the  springing  of  the  arches.  The  ce?itral  piers  are  ^ft. 
and  the  outer  ones  2  ft.  6  in.  thick.     Width  at  right-angles  to 


TRIMETRIC  PROJECTION.  219 

the  plane  of  its  face  2^  ft.     Other  dimensions  and  emhellish- 
jnents  at  pleasure.     Show  its  trimetric  projection. 
Scale  lo  ft.  to  i  in. 

8.  Trimetric  projection  of  a  brick  with  two  semicircular 
grooves  of  I'^in.  radius  cut  in  a  large  face — one  running 
parallel  to  its  long  edges  through  the  middle  of  the  face,  arid 
the  other  bisecting  the  former  at  right  angles.  Show  the 
intersection  of  the  grooves.     To  half  scale. 

9.  Trimetric  projection  of  hexagonal  right  pyramid ; 
axis  3  "5  in.  long,  side  of  base  i  i?i. 

Let  abcdef  be  the  hexagonal  base.  Produce  two  of  its 
parallel  sides  as  ab,  de,  and  through  c  and  /  draw  a  pair  of 
parallel  lines  at  right  angles  to,  and  meeting  the  produced 
sides  ab,  de.  This  forms  a  rectangle  circumscribing  the 
hexagon. 

Set  off  the  reduced  lengths  of  the  sides  of  this  rectangle 
along  two  of  the  projected  axes,  as  oy,  oz,  and  complete  the 
parallelogram  for  the  projection  of  the  circumscribing 
rectangle.  The  corners  of  the  hexagonal  base  may  now  be 
found,  in  the  sides  of  the  parallelogram,  and  the  projection 
of  the  base  drawn  in. 

For  the  vertex,  draw  from  the  centre  of  the  parallelogram 
a  straight  line  (the  projection  of  the  axis)  parallel  to  ox,  and 
make  it  equal  to  3  "5  in.  on  the  scale  of  OX.  Join  the 
vertex  with  the  corners  of  the  hexagonal  base  for  the  com- 
plete projection  of  the  pyramid. 

Note.  By  the  the  device  of  an  enveloping  rectangle, 
)r,  more  generally,  by  the  aid  of  rectangular  ordinates,  any 
)lane  figure  whatever,  or  any  prismatic  or  pyramidal  form, 


220  SOLID    OR  DESCRIPTIVE   GEOMETRY. 

may   be  projected  as  above.     The  general  cases  will  be, 
however,  treated  under  Trihedral  Projection  below. 

10.  A  figure  formed  by  describing  two  semicircles  on  two 
parallel  sides  of  a  regular  hexagon  of  i' 2^  in.  side  as  diameter 
— the  concavity  of  the  semicircles  being  turned  in  the  same 
direction — is  the  base  of  a  prismatic  form  4*5  in  long.  Show 
its  trimetric projection. 

11.  Show  the  Trimetric  projection  of  your  instrument 
case  when  the  lid  is  open  at  an  angle  of  60". 


ISOMETRIC  PROJECTION. 

Isometric  Projection  has  been  already  defined  as  the 
particular  case  of  Trimetric  Projection  that  results  from  the 
three  axes  being  equally  inclined  to  the  plane  of  projection. 
In  this  case  it  is  clear  that  the  obtuse  angles  formed  by  the 
meeting  of  the  plans  of  the  three  axes  at  the  point,  0,  are 
all  equal — /.  e.  are  each=  120°. 

We  may  therefore  begin  by  drawing  the  plans  of  the 
three  axes  at  equal  angles  of  120",  and  proceed  to  deter- 
mine the  scale  of  one  of  the  axes  as  for  the  trimetric  scale 
above.  This  scale,  which  also  serves  for  the  other  two  axes, 
is  called  the  Isometric  Scale. 

The  ratio  of  a  line,  measured  along  an  isometric  axis, 
to  its  isometric  projection  may  be  readily  proved  to  be  as 
J 2,  is  to  J 2.  We  may,  therefore,  avail  ourselves  of  this 
established  ratio  and  construct  as  follows  the 

Isometric  Scale: — Draw  a  right-angled  triangle  having 
one  side  i  in.  long  and  the  other  equal  in  length  to  the 
diagonal  of  a  square  of  i  in.  side.     The  diagonal  of  a 


AXIAL  PROJECTION.  '  221 

square  whose  side  is  unity  is  equal  to  Ji,  and  the 
hypotenuse  of  a  right-angled  triangle  with  one  side  equal  to 
unity  and  the  other  to  J 2,  will  be  equal  to  J^  (Euclid 
47,  Bk.  I.).  That  is,  the  hypotenuse  of  this  triangle  is  to . 
its  longest  side  as  J^  is  to  J 2.  Therefore,  real  lengths  may 
be  measured  along  line  Jt,  and  the  corresponding  isometric 
lengths  found  in  line  ^2  by  drawing  perpendiculars  to  that 
line  from  the  points  set  off  in  the  hypotenuse  ^3,  or 
a  scale  of  isometric  inches  may  be  similarly  constructed. 

Isometric  being  but  the  most  simple  form  of  Trimetric 
projection,  it  is  unnecessary  to  give  further  exercises.  All 
those  given  under  Trimetric  projection,  or  any  that  the 
student'  has  omitted,  may,  however,  be  worked  here  at  his 
own  or  the  teacher's  discretion. 


AXIAL  PROJECTION. 

The  term  Axial  Projection  is  used  to  denote  a  trimetric 
method  which  lends  itself  with  great  facility  to  the  pro- 
jection of  symmetric  forms  bounded  by  plane  faces,  such 
as  the  forms  of  crystals. 

The  investigation  of  the  geometrical  properties  of  these 
forms,  and  of  the  number  and  position  of  the  axes  of 
symmetry  to  which  their  faces  can  be  most  conveniently 
referred,  is  the  special  province  of  crystallography.  The 
principles  on  which  they  are  projected  fall,  however,  within 
the  scope  of  Descriptive  Geometry,  while  many  of  the 
forms  have  an  interest  attaching  to  them  quite  apart  from 
their  crystalogical  significance. 

OX,    OY,    OZ,   being  the    three  rectangular  axes    as 


222  SOLID   OR  DESCRIPTIVE   GEOMETRY. 

before,  it  is  evident  that  an  indefinite  plane  will  in  general 
meet  them  in  three  points,  which  may  be  called  the  axial 
traces  of  the  plane.  Let  a,  )3,  y  denote  the  distances  of  the 
points  of  section  or  axial  traces  along  OX,  OY,  OZ 
respectively.  A  plane,  relatively  to  the  three  axes,  is 
obviously  completely  determined  by  its  axial  traces.  The 
plane  of  the  face  of  a  form  referred  in  this  way  to  three 
rectangular  axes  can  therefore  be  expressed  by  the  ratio  of 
the  parameters  of  the  face : — a  :  y8  :  y. 

The  student  will  observe  that  it  is  in  general  convenient 
to  treat  the  axes  as  three  lines  drawn  yr^;«  the  point,  O,  to 
indicate  three  directions  in  which  certain  measurements  are 
to  be  set  off,  but  for  the  present  purpose  the  axes  must  be 
produced  through  the  origin,  O,  on  both  sides.  The 
system  of  intersecting  planes  that  give  rise  to  the  indefinite 
axes  XX',  YY',  ZZ',  in  question,  form  eight  similar  trihe- 
dral angles,  whence  it  follows  that  for  every  plane  determined 
by  its  axial  traces  in  one  of  these  trihedral  angles  there  will 
be  seven  other  planes  similarly*  related  to  the  axes — one 
in  each  of  the  other  seven  trihedral  angles. 

If  the  three  fundamental  lines,  ox,  oy,  oz,  used  as  the 
projection  of  the  axes  in  previous  problems,  be  produced 
and  lettered  xox',  yoy',  zoz',  any  required  trihedral  angle 
can  be  specified  by  the  extremities  of  the  semi-axes,  as  xyz, 
yzx',  &c. 


Exercises. 

I.  One  face  of  a  form  symmetrical  about  three  equal 
rectangular  axes  is  given  by  the  formula  a:/8:y=i  :  r  :i; 
determine  its  axial  projection. 


AXIAL  projection;  222, 

The  projections  of  the  three  indefinite  axes  and  the  set  of 
scales  may  be  determined  once  for  all,  as  in  the  case  of  the 
trimetric  exercises  above. 

The  greatest  number  of  faces  similar  to  the  one  given 
that  can  be  disposed  about  the  axes  is  eight — one  in  each 
trihedral  angle.  The  faces  are  equilateral  triangles,  and  the 
form  is,  therefore,  a  regular  octahedron. 

To  draw  the  projection  it  is  only  necessary  to  cut  off,  by 
means  of  the  scales,  equal  lengths  on  each  axis  from  O,  on 
both  sides  of  that  point,  and  join  up  the  points  so  found  for 
the  edges  of  the  octahedron.  For  example,  let  i,  2,  3,  be 
the  plans  of  the  points  on  the  semi-axes  ox,  oy,  oz,  respect- 
ively, then  if  i  be  joined  with  2,  2  with  3,  and  3  with  i,  the 
triangle  123  will  be  the  projection  of  the  face  situated  in 
the  trihedral  angle  xyz.  Similarly  the  other  faces  may  be 
drawn, 

2.  Determine  the  form  produced  by  the  intersection  of  the 
indefinite  planes  of  the  alternate  faces  of  the  octahedron  drawn 
in  the  preceding  problem. 

Lines  through  the  points  i,  2,  3,  parallel  respectively  to 
lines  23,  31,  12,  will  give  the  intersections  of  the  face  123 
with  the  alternate  faces  of  the  trihedral  angles  xy'z',  x'yz', 
x'y'z.  These  intersections  are  three  edges  of  one  face  of 
the  tetrahedron — which  is  the  form  produced.  Similarly 
the  other  edges  can  be  found. 

3.  Axial  projection  of  a  syjnmetrical  form  one  face  of 
which  is  given  by  the  formula  a:^:y:  =  oc:i:oc.  The 
sign  oc  =  infinity. 

The  faces  of  this  form  are  six  in  number — each  meeting 
one  axis  at  unit  distance  from  O  and  parallel  to  the  other 
two.     The  form  produced  is,  therefore,  a  cube. 


224  SOLID   OR  DESCRIPTIVE  GEOMETRY. 

The  intersections  of  the  planes  being  parallel  to  the 
axes,  no  difficulty  will  be  experienced  in  drawing  the  form. 

4.  Axial  projection  of  the  form  produced  by  planes 
parallel  to  one  axis  and  meeting  the  other  two  at  the  unit 
distance:  a  :  /S  :  y  =  c<    :  i  :  i. 

This  form,  which  is  the  Rhombic  Dodecahedron,  will 
serve  as  an  additional  exercise.  There  are  several  others, 
the  most  general  of  which  is  a  forty-eight-faced  form  having 
the  formula  m  :  1  \  n,  but  their  discussion  belongs  more  to 
crystallography  than  to  Descriptive  Geometry. 


TRIHEDRAL  PROJECTION. 

Hitherto  all  forms  have  been  regarded  as  trimetrically 
developed  about  rectangular  axes  in  three  directions  from 
their  origin  O.  But  the  most  general  aspect  in  which  a 
problem  can  present  itself  is  when  the  position  of  the  form 
is  not  referred  directly  to  the  axes  but  to  the  three  intersect- 
ing planes.  A  Trimetric  projection  of  this  kind  would  also 
be  Trihedral,  and  the  latter  term  serves  very  conveniently  to 
denote  this  most  general  aspect  of  the  trimetric  method. 

As  was  before  observed,  three  indefinite  rectangular 
planes  intersect  in  eight  similar  trihedral  angles,  but  in  order 
to  avoid  the  necessity  for  a  cumbrous  system  of  signs  to 
specify  which  of  these  trihedral  angles  is  meant,  it  will  be 
best  to  represent  all  points,  lines,  figures,  and  forms  in  one 
of  these  angles  only,  viz.  that  formed  by  the  planes  XOYy 
YOZ,  ZOX,  and  limited  by  the  semi-axes  OX,  OY,  OZ. 
For  the  purposes  of  construction  the  planes  may  of  course 
be  considered  as  indefinite. 


TRIHEDRAL  PROJECTION.  225 

Problems. 

1.  To  determine  a  point,  P,  in  space  trihedrally. 

Let  a,  b,  c,  be  the  respective  distances  of  P  from  the 
planes  YOZ,  ZOX,  XOY.  Measure  the  trimetric  length 
of  a  from  o  along  ox  and  from  the  point  thus  found 
draw  a  parallel  to  oy  equal  to  b  as  shown  on  the  trimetric 
scale  oi  OY.  A  parallel  to  oz  from  the  point  last  deter- 
mined, equal  in  length  to  c  on  the  scale  of  OZ,  will  give  p, 
the  trihedral  projection  of  P. 

2.  Given  p,  the  trihedral  projection  of  P,  and  c,  the 
distance  of  P  from  the  plane  XOY,  find  a  and  b,  its  distances 
frOm  planes  YOZ  and  ZOX, 

Shotv  also  the  trihedral  projections  of  the  orthographic 
projection  ofVon  each  of  the  reference  planes. 

Theorem.  A  point  in  space  is  completely  determined 
when  its  trihedral  projection  and  its  distance  from  one  of 
the  three  reference  planes  are  given. 

The  particular  trihedral  angle  in  which  P  is  situated 
will  depend  on  the  position  of  /  and  the  length  of  c.  The 
following  construction  although  perfectly  general,  if  the 
student  interpret  from  his  drawing  the  direction  in  which 
;he  parallels  are  to  be  drawn  from  /,  is  intended  to  apply 
:)nly  to  the  one  trihedral  angle  used  before  : — 

Draw  from  /  a  parallel  to  zo  equal  to  the  trimetric 
ength  of  c.  The  extremity  of  this  parallel  is  the  trihedral 
)rojection  of  the  orthographic  projection  of  ^  on  the  plane 
XOY.  Two  parallels  to  xo  and  yo  from  the  extremity  of 
•  he  parallel  first  drawn,  meeting  oy  and  ox  respectively, 
(  etermine  a  and  b. 

E.  G.  15 


226  SOLID   OR  DESCRIPTIVE   GEOMETRY. 

From  a  and  b  thus  found  the  other  required  projections 
of  F  can  be  readily  determined. 

Note.  The  student  will  observe  that  a  line  is  deter- 
mined trihedrally  by  two  contained  points  and  a  plane  by 
three,  but,  more  generally,  by  their  trihedral  traces. 

Def.  The  trihedral  traces  of  a  line  are  the  points  in 
which  it  pierces  the  planes  of  reference. 

Def.  The  trihedral  traces  of  a  plane  are  the  lines  in 
which  it  intersects  the  planes  of  reference. 

It  is  obvious  that  the  trihedral  traces  of  a  plane  pass 
through  the  axial  traces  of  the  same  plane. 

3.  Given  the  data  that  determine  the  trihedral  projections 
of  two  points,  P  and  Q,  in  space  find  the  trihedral  traces  of  the 
line  joining  them. 

Assuming  that  the  three  reference  planes  are  limited  by 
the  semi-axes  OX,  OV,  OZ,  it  is  plain  a  straight  line  can 
have  but  two  trihedral  traces  and  these  will  always  suffice  to 
determine  it.  Unless,  however,  the  line  be  parallel  to  the 
third  plane  the  two  will  intersect  if  produced,  and,  for  the 
purposes  of  construction,  this  third  trace  might  be  found 
useful. 

It  will  be  easily  seen  from  inspection  in  which  plane 
there  is  no  trace.  This  we  will  suppose  to  be  the  plane 
ZOX.  Through  the  projections  of  F  and  Q  on  this  plane 
draw  a  straight  line  (that  is  the  projection  of  the  Hne  FQ 
on  plane  ZOX),  and  produce  it  to  meet  oz  and  ox  in  /  and 
m.  Parallels  from  /  and  m  to  oy  will  meet  the  trihedral 
plan  of  the  line  FQ  produced  in  the  required  traces  of  the 
line. 


TRIHEDRAL  PROJECTION.  22/ 

Note.  The  trihedral  traces  of  a  plane  containing  three 
given  points  will  of  course  pass  through  the  traces  of  the 
lines  joining  them  two  and  two.  Hence  the  traces  of  such 
a  plane  might  be  readily  determined. 

The  foregoing  problems  are  of  a  somewhat  general 
character  and  are  offered  merely  as  illustrations  of  a  method 
of  projection  that  has  never  yet  been  sufficiently  worked 
out.  To  the  student  who  has  leisure,  and  the  inclination, 
to  think  out  some  of  its  applications  for  himself  they  will,  it 
is  hoped,  suggest  a  method  that  promises  some  interesting 
results. 


15—2 


XII. 


MISCELLANEOUS  PROBLEMS. 

I.  Plan  and  elevation  of  a  cube  of  3  inches  edge,  when 
three  of  its  corners  are  i,  17,  and  3  inches,  respectively,  above 
the  horizontal  plane. 

Refer  to  Chapter  in.  Problem  21. 

Take  one  inch  to  equal  10  units. 

(i)  Draw  the  square  ABCD,  and  index  ^  as  i  inch  or 
10  units,  -5  as  17  units,  and  C  as  30  units,  that  is,  the 
respective  heights  to  which  these  points  are  to  be  lifted. 

(2)  To  find  a  'horizontal,'  in  order  that  the  square  may 
be  folded  about  it  into  the  required  position  \-t- 

If  A  and  6",  which  are  to  be  at  heights  of  10  and  30 
units,  be  joined,  there  will  be  a  point  in  the  line  ACi'j 
units  high,  that  is,  at  the  same  level  as  point  B.  Find  this 
point,  M,  by  dividing  the  line  in  due  proportion,  and  the  line 
drawn  through  B  and  J/ will  be  the  horizontal  required,  at 
level  17  units  or  17  in.  above  the  horizontal  plane. 

(3)  A  vertical  plane  at  right  angles  to  plane  of  face 
ABCD  may  now  be  assumed  by  taking  a  line  at  right 
angles  to  BM,  for  the  intersection,  xy,  of  this  vertical  plane 


MISCELLANEOUS  PROBLEMS.  22() 

and  the  horizontal  plane  at  level  17  units.  The  vertical 
trace  of  plane  of  face  ABCD  may  be  easily  determined 
because  a  and  c  will,  from  the  position  of  the  assumed 
vertical  plane,  be  in  the  vertical  trace,  and  their  heights  are 
known. 

(4)  The  square,  ABCD,  is  one  face  of  the  cube  rabatted 
about  the  horizontal  BM  into  a  horizontal  plane  at  level 
1 7  units.  The  rest  of  the  work  is  similar  to  that  in  former 
problems. 

Note.  The  heights  of  the  three  points  determine  the 
inclination  of  two  adjacent  edges  of  the  square  face ;  hence, 
another  construction  based  on  Chap.  v.  (see  Prob.  7)  may 
be  used. 

2.  Plan  and  elevation  of  a  tetrahedron,  edge  3  inches, 
when  the  three  corners  of  its  lowest  face  are  '8,  1*5,  and  2*2 
inches  above  the  horizontal  plane. 

3.  A  right  pentagonal  pyramid,  side  of  base  1*5  in.,  axis 
3*5  /;z.,  has  its  veriex  3  in.  high  and  the  extremities  of  one 
edge  of  the  base  i  in.  and  i'2  5  in.  high  respectively..  Flan  and 
elevation. 

Prob.  26,  Chapter  iir.  required. 

4.  Plan  and  elevation  of  a  cube  of  7,  inches  edge,  when  the 
indefinite  plans  of  three  adjacent  edges  meeting  in  point  A  are 
given. 

Compare  Chapter  in.  Problem  27,  and  Problem  i,  page 
216,  "Trimetric  Projection." 

(i)  Draw  the  three  given  lines  as  ab,  ac,  and  ^^  forming 
three  obtuse  angles  at  a.  Since  the  projections  of  a  perpen- 
dicular to  a  plane  are  at  right  angles  to  its  traces,  a  line 
drawn  at  right  angles  to  any  one  of  these  edges,  as  to  ab,  will 


230         SOLID  OR  DESCRIPTIVE   GEOMETRY. 

be  a  horizontal  in  the  plane  containing  the  other  two. 
Draw  such  a  line  to  cut  ac  in  ^  and  ad  in  s.  Then  the 
angle /^j  is  the  plan  of  a  right  angle  the  plane  of  which  may 
be  rabatted  about  the  horizontal  ps  by  bisecting  ps  and 
describing  a  semicircle  on  it.  The  angle  in  a  semicircle 
being  a  right  angle,  point  A  when  rabatted  will  be  in  the 
intersection  of  the  semicircle  with  fl;A  If/  and  A,  and  A 
and  s,  be  joined,  the  edge  of  the  cube,  3  inches,  can  be 
measured  from  A  on  these  lines  Ap  and  As^  or  the  same 
produced,  and  the  square  can  be  completed. 

(2)  The  plan  and  elevation  of  the  whole  may  be  finish- 
ed by  taking  a  vertical  plane  at  right  angles  to  the  horizon- 
tal ps  and  working  from  the  square  rabatted. 

5.  Given  an  oblique  plane  by  its  traces,  equally  inclined 
at  angles  of  j\o°  to  xy.  Determine  the  plan  and  elevation  of  a 
tetrahedron,  3*5  in.  edge,  resting  upon  it.  No  edge  of  tetra- 
hedron to  be  horizontal. 

Determine  the  inclination  of  the  plane,  Chapter  iii. 
Problem  5.  Take  a  new  ground  line  at  right  angles  to  the 
horizontal  trace,  set  up  the  vertical  trace  in  this  new  vertical 
plane,  complete  elevation  and  plan  therein,  and  thence 
derive  the  elevation  upon  the  first  or  given  vertical  plane. 

6.  An  oblique  pyramid  stands  on  its  base,  which  is  a 
regular  hexagon  ABC...F  ^  1*5  inches  side.  V  is  the  vertex, 
the  face  Y  KB  is  inclined  at  75°  to  the  base,  VBC  at  65°,  VCD 
at  60".  Draw  the  plan  of  the  solid.  Science  Examination, 
1870. 

( 1 )  Draw  the  hexagon. 

(2)  Determine  the  traces  of  plane  75",  taking  the  xy  at 
right  angles  to  the  horizontal  trace. 


MISCELLANEOUS  PROBLEMS.  23 1 

(3)  The  same  with  plane  65*. 

(4)  In  each  of  these  planes  determine  a  horizontal  line 
at  the  same  height.  The  intersection  of  these  lines  gives  a 
point,  /,  in  the  plan  of  the  intersection  of  the  two  planes. 

Another  point  in  the  same  intersection  is  b,  and  the  line 
bp  being  the  plan  of  one  edge  of  the  pyramid  contains  v. 

(5)  Work  similarly  with  planes  65°  and  60°,  and  the 
edge  cv  is  determined,  also  containing  v.  The  point  v  is 
fixed  by  the  intersection  of  the  lines  bpv  and  cv. 

7.  Given  the  plan  of  a  sphere,  required  to  determine  the 
plan  of  a  cube  inscribed  in  if,  a,  the  plan  of  one  of  its  corners^ 
and  the  direction,  ab,  in  plan  of  one  edge  being  also  given. 

(i)  To  find  the  true  length  of  the  edge  of  the  inscribed 
cube.  On  the  diameter  of  the  circle  which  is  the  plan  of  the 
sphere,  mark  off  one-third  of  its  Length  from  one  extremity, 
and  from  the  point  so  found  erect  a  perpendicular  to  meet 
the  circumference.  Join  the  extremities  of  the  diameter  to 
this  point  in  the  circumference.  The  shorter  line  gives  the 
length  of  the  edge  of  the  inscribed  cube,  whilst  the  longer 
gives  that  of  the  edge  of  the  inscribed  tetrahedron.  Or,  by 
computation,  if  d  equals  the  diameter  of  the  sphere  in 
inches,  and  e  the  required  edge  of  the  cube,  thei) 

J 2^  ••  J ^  ••••  d  :  e. 

(2)  Make  an  elevation  of  AB  on  ab  used  .as  a  line  of 
level  at  the  height  of  the  centre  of  the  sphere,  and  determine 
b.  To  do  this,  describe  a  circle  on  the  part  of  ab  produced 
which  cuts  the  circle  forming  the  plan  of  the  sphere.  Erect 
a  perpendicular  to  ab  from  point  a  to  meet  the  cjrcumr 
ferenca     This  gives  a'.     From  a'  measure  AB  as  a  chord. 


232  SOLID   OR  DESCRIPTIVE  GEOMETRY. 

This  gives  b',  and  from  b'  a  perpendicular  to  ab — which 
being  used  as  a  line  of  level  corresponds  for  this  work  to  a 
ground  line — determines  b. 

(3)  A  line  drawn  from  b  through  the  centre  o  of  the 
sphere  with  the  distance  from  b  to  0  measured  on  it  from  0, 
will  give  the  plan  of^,  another  point  in  the  face  A...G. 

(4)  Determine  by  its  traces  the  plane  containing  aa\ 
and  perpendicular  to  ab,  db'.  This  will  be  the  plane  of  the 
face  A...G  of  the  cube.  To  do  this,  through  point  a  draw 
the  vertical  trace  at  right  angles  to  a'b',  and  through  m, 
where  this  vertical  trace  crosses  ab,  the  line  of  level,  draw 
horizontal  trace  at  right  angles  to  ab. 

(5)  Rabat  this  plane  and  with  it  points  aa!,  gg.  Join 
AG  which  will  be  a  diagonal  of  this  face  of  the  cube 
rabatted.  Complete  the  square,  and  therefrom  determine 
its  plan  by  rotating  the  plane  back  to  its  original  position. 

(6)  The  edge  ab  being  already  drawn,  parallels  to  ab, 
and  of  the  same  length,  through  the  points  a...g  will  com- 
plete the  plan  of  the  solid. 

8.  PlaH  drid  eteVation  of  a  tetrahedron  inscribed  in  a 
sphere  of  3  in^  radius.  Two  adjacent  edges  of  the  solid  inclined 
at  angles  of  zo^  and  30°  respectively.  Centre  of  sphere  3  in. 
above  hotizontal  plane. 

(i)  Find  edge  of  tetrahedron  inscribed  in  a  sphere 
of  2  inches  radius.     See  preceding  Problem. 

Note.  The  edge  of  tetrahedron  is  a  diagonal  of  face  of 
inscribed  cube. 


MISCELLANEOUS  PROBLEMS.  233 

(2)  Draw  an  equilateral  triangle  with  sides  equal  to 
edge  of  inscribed  tetrahedron,  and  determine  the  diameter 
of  a  circle  circumscribing  this  triangle.  Set  off  the  diameter 
of  this  circle  as  a  chord  of  a  great  circle  of  given  sphere,  and 
find  the  length  of  a  perpendicular  from  the  centre  of  great 
circle  to  the  chord  drawn.  Call  Ox  the  length  of  this  per- 
pendicular ;  then  Ox  is  the  length  of  a  perpendicular  from 
centre  of  given  sphere  to  plane  of  face  of  inscribed  tetra- 
hedron. 

(3)  Determine  on  a  convenient  part  of  the  paper  the 
plane  of  a  face  of  any  tetrahedron  having  two  adjacent 
edges  of  that  face  inclined  as  above.  Chapter  iii.  Problem 
20. 

(4)  Draw  from  Centre,  O,  of  the  given  sphere,  a  line 
perpendicular  to  the  plane  determined  in  (3)  and  cut  off  on 
this  line  from  the  centre  of  given  sphere  a  length  equal  to 
Ox.  A  plane  through  the  point  x  at  right  angles  to  the 
perpendicular  Ox,  will  be  parallel  to  the  plane  of  the  face 
of  the  auxiliary  tetrahedron  first  drawn,  and  will  also  be  the 
olane  of  one  face  of  the  solid  required. 

(5)  Rabat  this  plane  with  the  circle  in  which  it  cuts 
he  given  sphere. 

(6)  Inscribe  in  the  rabatted  circle  the  triangular  face 
of  the  required  tetrahedron.  To  do  this: — draw  three 
lingents  to  the  circle  parallel  to  the  three  sides  of  the 
1  dangular  face  of  the  auxiliary  tetrahedron  first  drawn  for 
the  purpose  of  determining  the  plane  of  a  face  of  that 
J  olid.  The  three  tangents  constitute  an  equilateral  triangle 
( ircumscribing  the  circle.  Draw  lines  from  the  three  corners 
(  f  this  figure  to  the  centre  of  the  circle,  and  join  the  points 


234  SOLID   OR  DESCRIPTIVE   GEOMETRY. 

where  these  lines  cut  the  circumference.  This  will  be 
another  equilateral  triangle  with  sides  parallel  to  the  one 
inscribed  in  the  rabatted  circle,  and  will  be,  moreover, 
the  rabatted  plan  of  the  face  of  the  required  inscribed 
tetrahedron,  from  which  the  projections  can  be  at  once 
completed. 

9.  The  centre,  O,  of  a  sphere  of  2  inches  radius  is  3 
inches  above  the  plane  of  the  paper.  A  point,  a,  on  the  surface, 
4*25  inches  high,  is  the  plan  of  one  corner,  A,  of  an  octahedron 
inscribed  in  the  sphere.  The  edge  AB  is  inclined  45".  Draw 
the  plan  and  an  elevation  on  a  verticcil  plane  inclined  30°  to 
the  line  ao. 

(i)  Determine  the  edge  of  the  octahedron.  This  will 
be  equal  to  the  side  of  a  square  inscribed  in  a  great  circle 
of  the  given  sphere. 

If  r  =  radius  of  sphere,  x  =  side  of  inscribed  octahedron, 
then 

r  :  X  -.w  :  J 2. 

(2)  Make  plan  and  elevation  of  sphere,  and  determine 
the  plan  and  elevation  of  A  on  its  surfacp.  To  do  this  : — 
draw,  in  the  elevation,  a  line,  at  the  height  of  4*25  inches, 
parallel  to  xy  and  cutting  the  elevation  of  the  sphere.  This 
may  be  considered  as  the  vertical  trace  of  a  horizontal  plane 
passing  through  sphere  at  the  assumed  level.  The  section 
of  the  sphere  by  this  plane  will  be  a  circle,  shown  in  true 
form  in  plan,  and  as  a  straight  line  in  the  trace  of  the  plane 
in  elevation.  The  point  A  is  on  the  circumference  of  the 
circle  constituting  the  plane  section  of  sphere,  and  its  pro- 
jections a,  a  are  on  the  corresponding  projections  of  the 
section.     To  find  a,  draw  a  line  from  0,  the  plan  of  the 


MISCELLANEOUS  PROBLEMS.  235 

centre  of  the  sphere,  making  an  angle  of  30°  with  xy,  and 
produce  it  to  meet  the  circle  which  is  the  plan  of  the 
section.  The  point  of  intersection  is  a.  d  is  at  once 
found  by  projecting  a  to  the  straight  line  drawn  at  4*25 
inches  above  xy. 

(3)  Make  A  the  vertex  of  a  right  cone,  axis  vertical, 
generatrix  inclined  45"  and  equal  in  length  to  the  side  of 
the  octahedron  (i). 

The  base  of  this  cone  in  elevation  is  a  straight  line  and 
in  plan  a  circle,  with  radius  equal  to  that  of  the  given 
sphere,  described  from  the  point  a  as  centre.  The  section 
of  the  sphere  by  the  plane  of  the  base  of  the  cone  produced, 
is  another  circle.  Show  the  latter  in  plan.  Either  of  the 
two  points  where  the  plan  of  the  circular  base  of  the  cone 
meets  the  plan  of  the  circle  cut  from  the  sphere  by  the 
plane  of  the  base  of  the  cone  produced,  may  be  taken  as 
the  plan  b  of  the  other  extremity  B  of  the  edge  AB. 

(4)  Determine  //  and  draw  ab,  dV.  These  are  the 
projections  of  the  edge  AB. 

(5)  Determine  the  plane  containing  the  straight  line 
4,B  and  the  centre  of  the  sphere  0.  Rabat  it  and  complete 
he  square  A  BCD,  which  will  be  inscribed  in  a  great  circle 
jf  the  sphere, 

(6)  Determine  the  plan  abed,  and  draw  a  perpendicular 
o   the  plane  of  ABCD  from  O,  meeting  the   surface  of 

•  he  sphere  on  both  sides  of  the  plane  at  P  and  R. 

(7)  Complete  the  plan  of  the  solid  by  joining  /  with 
(  bed  and  r  with  the  same  points,  and  the  elevation  by  joining 
, '  and  r  with  db'c'd'. 


236  SOLID   OR  DESCRIPTIVE   GEOMETRY. 

10.  A  parallelogram,  sides  3  inches  and  2  inches,  included 
angle  Go'',  is  Ihe plan  of  a  square.  Determine  the  side  of  the 
square  and  the  inclination  of  its  plane.  Science  Exam.  Hon. 
1871. 

(i)     Draw  parallelogram  and  inscribe  in  it  an  ellipse. 

This  ellipse  will  be  the  plan  of  a  circle  inscribed  in 
the  square  of  which  the  parallelogram  is  the  given  plan, 
and  which  circle  will  necessarily  lie  in  the  same  plane  with 
the  square;  that  is,  in  the  plane  whose  inclination  we  have 
to  find. 

(2)  Determine  the  major  and  minor  axes  of  this  ellipse. 

The  former  is  a  horizontal  line  lying  in  the  plane  of  the 
square,  and  is  therefore  the  diameter  of  the  circle  of  which 
the  ellipse  is  a  plan.  This  latter  property  carries  with  it 
two  others,  viz.  that  of  being  the  diameter  of  the  circle  in- 
scribed in  the  square  whose  side  is  required,  and  also  that  of 
being  eqiial  to  that  side. 

(3)  To  determine  the  inclination  of  the  plane,  it  is 
only  necessary  to  make  a  projection  of  the  minor  axis  on  a 
vertical  plane  assumed  at  right  angles  to  the  major  axis. 
The  plane  of  the  square  and  the  assumed  vertical  plane  are 
perpendicular,  hence,  the  elevation  of  the  minor  axis  of  the 
ellipse  will  fall  in  the  vertical  trace  of  the  plane  of  the  square, 
and  its  angle  of  inclination  with  xy  will  therefore  be  the 
same  as  that  of  the  plane. 

The  student  will  require  no  further  assistance  in  deter- 
mining this  plane,  or  in  rabatting  it  with  its  contained  square 
for  the  purpose  of  verifying  the  side  found  from  the  major 
axis  of  the  ellipse. 


MISCELLANEOUS  PROBLEMS.  237 

11.  A  7-ight  hexagonal  pyramid,  5  inches  high,  has  its  axis 
inclined  50°  and  one  dianieter  AD,  4  inches  long,  of  its  hexago- 
nal base  inclined  25".  Draw  plan  and  elevation,  and  deter- 
mine the  circumscribing  sphere. 

(i)  If  the  axis  is  inclined  50",  the  plane  of  the  base 
will  be  inclined  40°.  Determine  this  plane  by  its  traces,  and 
draw  plan  and  elevation  of  the  solid  resting  on  it.  Refer  to 
Chapter  iv.  Problem  i. 

(2)  To  determine  the  sphere.  Rabat  the  plane  con- 
taining the  vertex  and  a  diameter  of  the  base  of  the 
pyramid.  The  horizontal  traces  of  the  two  slant  edges  of 
the  pyramid  springing  from  this  diameter  to  the  vertex  will 
be  in  the  horizontal  trace  of  plane  required.  Determine 
the  centre  of  the  triangle  formed  by  this  diameter  and  the 
slant  edges  of  solid  thus  rabatted,  and  describe  circle 
:herefrom  passing  through  the  three  corners  of  the  triangle. 
Phis  gives  the  rabatted  centre  of  the  required  sphere  and  its 
"adius,  whence  the  required  projections  of  the  circum- 
scribing sphere  can  be  readily  determined. 

12.  Plan  and  elevation  of  a  pentagonal  pyramid  when 
me  face  is  vertical  and  one  long  edge  of  that  face  inclined  at  50". 
Side  of  base  i"25  in.,  axis  3*5  in.    Science  Exam.  Hon.  1872. 

(i)  Determine  angle  0  between  base  and  one  face  of 
)yramid. 

(2)  Determine  a  line  in  the  vertical  plane  of  projection 
inclined  50",  and  on  this  line  draw  a  triangular  face  of  the 
\  )yramid,  one  long  edge  of  that  face  lying  in  the  line. 

If  the  vertical  face  of  the  pyramid  be  assumed  parallel  to 
1  he  vertical  plane  of  projection,  the  triangular  face  of  the 
jiyramid  drawn  as  above  will  be  an  elevation  of  a  face  of  the 


238  SOLID    OR  DESCRIPTIVE   GEOMETRY. 

form,  and  the  plan  of  this  face  will  be  a  straight  line 
parallel  to  xy  at  any  chosen  distance  in  front  of  the  vertical 
plane. 

(3)  Assume  two  points  in  the  short  side  (i.  e.  the  base), 
or  short  side  produced,  of  the  triangular  face  of  the  pyramid, 
and  make  them  the  vertices  of  two  right  cones  having  their 
bases  in  the  vertical  plane  and  generatrices  inclined  to  bases 
at  Q  degrees. 

(4)  Draw  the  tangent  plane  to  these  cones.  The 
vertical  trace  will  be  a  line  tangential  to  the  circular  bases, 
and  the  horizontal  trace  a  line  drawn  through  the  horizontal 
trace  of  the  short  side  or  base  of  the  triangular  face  of  the 
pyramid  and  the  point  where  xy  is  met  by  the  vertical 
trace. 

(5)  Rabat  this  plane  and  the  short  side  of  the  triangular 
face  with  it.  On  this  side  rabatted  describe  the  pentagon 
which  is  the  base  of  the  pyramid,  and  determine  plan  and 
elevation  therefrom. 

The  projections  can  be  at  once  completed  by  joining  the 
proper  points,  all  of  which  are  thus  determined. 

13.  A  right  pyramid,  having  a  hexagon  of  1*25  in.  side 
for  its  base,  and  an  axis  of  2,'S  ^^-j  ^^^^  with  one  edge  on  the 
horizontal  plane,  a?td  a  face  Containing  that  edge  is  inclined  at 
25".     Draw  plan  and  elevation.     Science  Exam.  Hon.  1870. 

(i)  Determine  the  angle  a"  between  the  base  and  one 
face  of  the  pyramid. 

(2)  Determine  plane  of  25"  by  its  traces. 

(3)  Rabat  this  plane  and  draw  in  it  a  triangular  face 
of  the  pyramid  with  one  of  its  equal  sides  lying  in  the 
horizontal  trace,  and  thence  determine  plan. 


MISCELLANEOUS  PROBLEMS.  239 

(4)  Produce  the  plan  of  the  base  of  the  triangular  face 
and  rabat  it. 

Note.  The  line  thus  rabatted  will  be  the  produced  base 
of  the  isosceles  triangle  first  drawn. 

(5)  Describe  a  circle  of  any  convenient  radius  touching 
this  line  and  mark,  P,  the  point  of  contact.  Assuming  this 
circle  to  be  the  base  of  a  cone  with  generatrix  inclined  a", 
find  the  vertex,  V,  and  lift  the  plane,  with  the  cone  on  it, 
back  to  its  former  position^ 

(6)  Find  the  projection s^//',  vv,  oi  PV,  and  the  hori- 
zontal trace  of  a  straight  line  drawn  from  V  to  P. 

The  line  drawn  through  this  tf-ace  and  the  point  where 
tangent  line  to  the  rabatted  base  of  the  cone  at  point  P 
meets  the  horizontal  trace  of  plafie  25",  will  be  the  hori- 
zontal trace  of  a  plafie  inclined  a"  to  this  plane,  and  will 
contain  the  base  of  the  triangular  face  of  pyramid  deter- 
mined in  (3),  and  hence  is  the  plane  of  the  base  of  that 
pyramid.  This  plane  can  now  be  rabatted  and  the 
projections  determined  as  in  previous  problems. 

It  may  be  remarked  that  the  line  PV  is  that  in  which 
the  plane  of  the  base  of  the  pyramid  touches  the  cone. 

14.  T/ie  hvo  extremities  A,  B,  of  one  edge  of  a  cube  are 
at  heights  of'^ifi.,  i'6in.  above  the  paper.  The  centre,  O,  of 
the  solid  is  at  a  height  of  \'\  in.  Draw  the  plan  of  the  solid. 
Edge  =  3  inches.     Science  Exam.  Hon.  1870. 

The  section  of  the  cube  made  by  a  plane  passing  through 
the  edge  AB  and  centre  O  of  the  solid  is  a  rectangle,  two 
sides  of  which  are  edges  of  the  cube  and  two  diagonals  of 
the  faces. 


240  SOLID    OR  DESCRIPTIVE    GEOMETRY. 

By  the  conditions,  the  heights  of  three  points — two  ex- 
tremities of  one  short  side,  and  the  centre — of  this  rectangle 
are  given. 

(i)  Determine  plan  and  elevation  of  this  rectangle  as 
in  Problem  i. 

(2)  Determine  a  plane  at  right  angles  to  one  of  the 
short  edges  of  the  rectangle  and  containing  one  of  the  long 
edges  of  the  latter.  This  condition  is  complied  with  if  the 
plane  be  drawn  through  one  extremity  of  a  short  side  and 
perpendicular  to  it.     Converse  of  Chapter  in.  Prob.  13. 

Or,  the  horizontal  trace  of  the  plane  may  be  drawn  at 
once  through  the  horizontal  trace  of  one  long  side  of  the 
rectangle  at  right  angles  to  the  plans  of  short  sides  of  the 
latter. 

Note.  This  trace  will  be  the  point  where  the  plan  of 
the  long  side  produced  meets  the  horizontal  trace  of  the 
plane  of  the  rectangle. 

(3)  Rabat  the  plane  thus  drawn,  which  is  the  plane 
of  one  face  of  the  cube,  and  with  it  the  long  side  of  the  rect- 
angle. 

The  latter  being  a  diagonal  of  a  face  of  the  cube,  the 
plan  of  the  solid  can  be  completed  therefrom  as  in  previous 
problems. 

15.  An  irregular  pyramid  has  for  its  base  a  triangle 
ABC-  AB  =  3  in.  ;  AC  =  3'5  in. ;  BC  =  4  in. :  the  plan  d  of 
the  fourth  corner  projected  on  the  plane  of  the  base  is  2  in.  from 
A,  I  "5  in.  from  ^;  the  true  length  of  the  remaining  edge  CD  «• 
37  in.  Draw  the  plan  of  this  pyramid,  when  standing  on  its 
base,  and  an  elevation  on  a  plajie  parallel  to  the  edge  AD. 
Determine  the  lengths  of  the  edges  AD,  BD,  and  the  height  of 
D  above  the  base. 


MISCELLANEOUS  PROBLEMS.  2\\ 

a.  Determine  the  inclinations  of  the  faces  ABD,  ACD,  to 
the  plane  of  the  base. 

p.  Draw  a  plan  of  this  pyramid  when  the  edge  CD  is 
vertical.     Science  Exam.  1868, 

(i)  Draw  the  triangle  ABC.  The  point  d  will  be 
found  by  describing  a  circle  of  2  in,  radius  from  A  a.s  a. 
centre  and  another  of  i'5  in.  radius  from  B ;  the  point  where 
the  circles  intersect  on  that  side  of  AB  nearest  to  C,  is  the 
point  required. 

(2)  Join  dC,  and  from  d  draw  a  perpendicular  to  this 
line.  A  circle  of  37  in.  radius  described  from  C  as  a  centre 
will  cut  this  perpendicular  at  D,  and  Dd  will  be  the  height 
of  the  vertex  of  the  pyramid. 

(3)  To  determine  the  lengths  of  the  edges  AD,  BD, 
draw  from  d  lines  perpendicular  respectively  to  Ad  and  Bd. 
Make  these  perpendiculars  equal  to  height  of  vertex  =  dD, 
md  complete  the  right-angled  triangles  AdD,  BdD ;  the 
lypotenuses  AD,   BD,   of    these   triangles   are  the  edges 

:  equired. 

(4)  For  the  required  elevation,  xy  must  be  taken 
]  'arallel  to  the  plan  dA  of  the  edge  AD.  The  rest  of  the 
A  'ork  presents  no  difficulty. 

Note.  The  elevation  of  AD  on  this  vertical  plane  is  in 
t  ue  length. 

(5)  For  the  inclination  of  the  faces  ABD,  ACD,  eleva- 
ti  Dns  may  be  made  on  vertical  planes  at  right  angles  to  the 
li  les  AB  and  A  C  respectively. 

(6)  For  the  plan  (^S)  make  an  elevation  on  a  plane 
p  irallel  to  CD,  and  from  this  elevation  deduce  the  plan 
n  quired,  by  taking  xy  at  right  angles  to  the  elevation  c'd'  of 
CD. 

E.  G.  16 


242  SOLID  OR  DESCRIPTIVE  GEOMETRY. 

1 6.  A  tetrahedron,  of  3  //;,  ^dge,  stands  with  one  corner  on 
the  fwrizontal plane,  so  that  the  plans  of  tJie  tiuo  edges  meeting 
at  that  corner  are  I'g  in.  and  i"5  /;/.  respectively.  Draw  the 
plan  of  the  solid.     Science  Exam,  Hon.  1871. 

(i)  Determine  the  plane  of  the  face  containing  the 
edges  whose  lengths  in  plan  are  given. 

To  do  this : — draw  a  line  and  make  it  equal  in  length 
to  either  of  the  plans  of  the  given  edges.  Show  an  eleva- 
tion of  this  edge  on  a  parallel  vertical  plane.  For  this 
purpose  the  xy  must  be  taken  parallel  to  the  line  drawn, 
and  should  be  assumed  at  a  convenient  distance  therefrom. 
An  elevation  on  this  plane  will  show  the  edge  in  true  length, 
i.e.  3  inches. 

This  determines  one  edge  of  the  solid.  The  other  is  of 
the  same  real  length  and  inclined  at  a  constant  -angle  of 
60"  to  the  first.  It  is  plain,  therefore,  that  the  second  will 
lie  somewhere  on  a  right  conical  surface  having  the  first  for 
an  axis  and  the  second  for  a  generatrix ;  that  is,  on  a  cone 
whose  vertex  is  at  the  point  where  the  first  edge  meets  the 
horizontal  plane, — vertical  angle  120",  generatrix  3  inches, 
and  axis  coincident  with  the  edge  whose  projections  we 
have  found.- 

Determine  plan  and  elevation  of  this  cone.  The  eleva- 
tion of  the  base  will  be  a  straight  line  bisecting  at  right 
angles  the  vertical  projection  of  the  edge  first  found,  and 
the  plan,  an  ellipse,  having  its  centre  at  the  middle  point  of 
the  plan  of  that  edge. 

Describe  a  circle  from  the  vertex  of  the  cone  in  plan 
with  a  radius  equal  to  the  length  of  the  plan  of  the  second 
edge;  and  from  either  point  common  to  circle  and  ellipse 
draw  lines  to  the  two  extremities  of  the  first  edge. 


MISCELLANEOUS  PROBLEMS.  243 

The  triangle  thus  drawn  is  the  plan  of  the  face  of  the 
tetrahedron  containing  the  two  given  edges.  From  this  the 
plane  of  the  face  can  be  at  once  drawn. 

(2)  Rabat  this  plane  and  complete  plan  of  tetrahedron 
as  in  previous  examples. 

17.  An  irregular  triangular  pyramid  has  its  edges  AB 
=  2;AC  =  2-5;  BC  =  3  5  AD^s'ss  ;  BD=3-45;  €0  =  3-84 
inches.  Determine  the  plan  of  the  sphere  circumscribing  this 
solid  when  the  plane  of  the  face  ABC  is  inclined  40"  and  one 
edge  AB  of  that  face  30*  to  the  horizontal  plane. 

(i)  Determine  the  plane  of  40**  and  the  line  inclined 
30"  lying  in  it. 

(2)  Rabat  the  plane  with  the  line  of  30",  and  on  the 
ine  so  rabatted  describe  the  triangle  ABC. 

(3)  On  the  side  AB  describe  another  triangle  ABD. 
[magine  the  triangle  ABD,  which  is  a  face  of  the  pyramid, 
o  revolve  about  AB  as  an  axis ;  the  point  D  will  describe 

; ,  circle  in  a  plane  perpendicular  to  AB  and  the  locus  of  D 
;.bout  the  axis  AB  will  be,  in  plan,  a  straight  line  drawn 
1  "om  the  vertex  D  of  the  triangle  ^  i5Z>  at  right  angles  to  AB. 
Draw  the  plan  of  this  locus  and  produce  it  indefinitely. 

(4)  On  the  side  BC  describe  a  triangle  BCD. 

A  line  from  D  perpendicular  to  CB  will  be  the  plan  of 
t  le  locus  of  D  about  CB  as  axis. 

Draw  this  line  and  produce  it  to  cut  the  line  determined 
ii(3). 

(5)  The  point  d  where  the  loci  intersect,  in  plan,  is 
t  e  plan  of  the  vertex  D.  The  height  of  this  point  can 
b  i  readily  determined  by  an  elevation  on  a  vertical  plane 
p  irallel  to  one  of  the  slant  edges. 

16 — 2 


244  SOLID   OR  DESCRIPTIVE   GEOMETRY.  . 

The  plan  and  elevation  of  the  solid  on  the  plane  of  40" 
present  no  difficulties.     See  Chapter  iv. 

The  four  points  A,  B,  C,  D,  situated  at  the  solid  angles 
of  the  pyramid,  are  those  through  which  the  spherical 
surface  has  to  pass,  and  the  centre  of  this  sphere  is  found 
as  in  Problem  i  on  "The  Sphere,"  Chap.  vii. 

18.  Draw  a  triangle  ABC  having  KB  =  2"j,  BC  =  3*2, 
AC  =  2-3  inches.  The  points  A,  B,  C,  are  the  stations  of 
three  observers  who  at  the  same  moment  take  the  altitude  of 
a  point  P  in  space.  The  angle  by  A  is  ■T)'^,  by  B  37°,  a?id 
by  C  46".  Show  the  plan  of  the  point  and  find  its  height 
in  inches  above  the  plane  of  ABC.  Science  Exam.  Hon. 
1872. 

The  point  P  is  the  common  intersection  of  three  in- 
verted cones  with  axes  vertical,  vertices  at  the  points  A,  By 
and  C,  and  generatrices  inclined  to  the  planes  of  the  bases 
at  angles  of  2>-^y  37*  and  46"  respectively. 

To  work  the  problem  : — 

(i)  Make  elevations  of  the  cones  on  a  vertical  plane, 
and  across  the  elevations  draw  a  series  of  lines,  at  con- 
venient distances  apart,  each  parallel  to  xy.  These  lines 
are  to  be  regarded  as  the  vertical  traces  of  a  series  of  hori- 
zontal planes  at  the  levels  shown  by  the  lines  in  elevation. 

(2)  The  sections  of  the  cones  by  the  horizontal  planes 
are  circles,  which  are  shown  in  true  form  in  plan.  Deter- 
mine the  plans  of  the  sections  of  any  two  of  the  cones. 
The  points  where  the  circles  which  are  the  plans  of  the 
sections  of  the  two  cones  under  consideration  by  the  same 
horizontal  planes  cut  one  another,  are  points  in  the  plan  of 
the  intersection  of  those  cones,  and  a  curve  drawn  through 
these  points  will  be  the  plan  of  that  intersection. 


MISCELLANEOUS  PROBLEMS.  245 

Similarly,  the  plan  of  the  intersection  of  one  of  the  above 
cones  with  the  third  can  be  found. 

The  point  where  the  two  intersections  in  plan  cross  is 
the  plan,  /,  of  the  point  P. 

(3)  The  elevation  /',  and  therefore  the  height  of  P, 
can  be  easily  determined  thus  -.  From  the  plan  of  the  vertex 
Df  one  of  the  cones,  as  A^  describe  a  circle  to  pass  through 
J>.  This  circle  is  the  plan  of  a  section  of  the  cone  whose 
vertex  is  ^  by  a  horizontal  plane  passing  through  P.  A 
line  at  right  angles  to  xy  and  a  tangent  to  the  circle  will  cut 
1  he  slant  side  of  the  cone  in  elevation  at  a  point  the  height 
<  )f  P  above  xy. 

19.  ABCV  is  a  pyramid  of  which  ABC  is  the  base. 
iV.B  =  3-5z>?.,  BC=3  in.^  CA-2"]^in.  The  vertex,  V,  is  4in., 
z  '2^in.,  and  4."j^in.  from  A^  B,  and  C,  respectively.  Deter- 
1.  line  the  inscribed  sphere. 

Note.  Find  the  vertex  as  the  common  intersection  of 
t  iree  spheres  of  4  in.,  4*25  in.,  and  475  in.  radii,  described 
fiom  the  centres  A,  B,  and  C,  respectively.  (Compare 
I  rob.  17  of  this  Chap.) 

The  centre  of  the  inscribed  sphere  will  be  the  common 
ir  tersection  of  three  planes — two  bisecting  any  two  of  the 
tl  ree  dihedral  angles  formed  by  the  slant  faces  of  the 
p  Tamid,  and  the  third  bisecting  one  of  the  dihedral  angles 
fcrmed  by  the  plane  of  the  base  and  a  slant  face.  The 
ra  Jius  will  be  determined  by  a  perpendicular  from  the 
ce  itre  to  one  of  the  faces. 

20.  Assume  three  intersecting  planes  and  determine  a 
sp  'lere  of  i  in.  diameter  touching  them. 


246  SOLID   OR  DESCRIPTIVE   GEOMETRY. 

The  centre  of  the  required  sphere  will  be  the  common 
intersection  of  three  planes,  each  -5  in.  distant  from,  and 
parallel  to,  one  of  the  three  touching  planes. 

21,  Plan  and  elevation  of  a  right  pentagonal  pyramid 
under  the  following  conditions  : — Plane  of  face  VAB  inclined 
50"  to  horizontal  and  75"  to  vertical  planes,  one  long  edge  VA 
of  face  inclined  25"  and  lowest  corner.  A,  i  inch  above  the 
horizontal  plane.     Other  di?nensions  at  pleasure. 

(i)  Determine  angle  a  between  the  base  and  one  of 
the  faces. 

(2)  Determine  the  face  VAB  lying  in  a  plane  inclined 
50"  to  horizontal  and  75*  to  vertical  planes.     Chapter  iii.,t 
Probs.  7  and  19. 

(3)  Determine  a  plane  containing  the  edge  AB  of  the 
face  VAB  and  inclined  to  the  plane  of  that  face  at  the 
angle  a.     Chapter  iii.,  Prob.  15,  converse. 

(4)  Rabat  the  plane  determined  in  (3),  and  on  the 
edge  AB  contained  therein  describe  the  pentagonal  base 
of  the  given  pyramid.  Determine  the  projections  of  the 
base  and  join  abcde,  plans  of  the  corners  of  the  base,  with 
V,  and  a'b'c'd'e'  with  v',  for  the  complete  projections  of  the 
solid. 

22.  A  right  cone,  base  J' 5  in.  diameter,  axis  7,  in.  long, 
rolls  over  the  convex  surface  of  another  right  cone  standing 
upon  a  circtdar  base  of  \-^  in.  diameter.  The  two  cones  have 
a  common  vertex  and  their  generatrices  are  of  equal  length,  so 
that  the  circle  which  is  the  base  of  the  one  cone  rolls  round 
that  of  the  other  in  a  plane  inclined  to  it  at  a  cojtstant  angle. 
Draiv  the  plan  of  the  curve  traced  by  a  point  in  the  circumfer- 
ence of  the  rolling  cone. 


MISCELLANEOUS  PROBLEMS.  247 

It  is  obvious  from  the  mode  of  generation,  that  every 
point  in  the  curve  is  upon  the  surface  of  a  sphere  having 
the  common  vertex  for  its  centre  and  a  generatrix  of  one 
of  the  cones  for  radius. 

When  a  circle  rolls  over  the  circumference  of  another,  so 
that  the  two  always  have  a  tangent  in  common  at  the  point 
of  contact,  the  curve  generated  by  a  point  in  the  circumfer- 
ence of  the  rolling  circle  is  either  an  epicycloid  or  a 
hypocycloid,  according  as  it  rolls  along  the  convex  or 
concave  side  of  the  fixed  circumference. 

When  the  two  circles  are  coplanar  the  curve  generated 
must  of  course  be  in  the  same  plane  with  them.  The 
curve  generated  in  the  above  problem  is,  however,  not  plane 
but  of  double  curvature — the  Spherical  Epicycloid. 

To  construct  tJie  plan  of  the  curve : — 

(i)  Measure  a  segment  of  the  circumference  of  the 
larger  fixed  circular  base  equal  in  length  to  the  whole 
circumference  of  the  rolling  base. 

Note.  The  circumferences  of  circles  being  directly 
proportional  to  their  diameters,  the  required  segment  will, 
in  this  case,  be  exactly  one  third  of  the  whole  circumfer- 
ence. 

(2)  Bisect  the  segment  in  0,  and  rabat  the  base  of  the 
-oiling  cone  that  touches  it  in  that  point,  about  its  horizontal 
race  {i.  e.  about  the  tangent  to  the  segment  at  0')  into  H.P., 
)y  aid  of  an  auxiliary  vertical  plane  of  elevation  passing 
hrough  0  and  the  plan,  v,  of  the  common  vertex. 

{3)  Produce  the  line  vo  through  the  centre  of  the 
abatted  base  to  meet  its  circumference  at  P,  and  let  P  be 
he  rabatted  point  that  traces  the  epicycloid.     Find  its  plan, 


248  SOLID   OR  DESCRIPTIVE   GEOMETRY. 

p,  when  the  base  is  lifted  back  to  its  proper  position.     This 
will  be  one  point  in  the  required  curve. 

(4)  Divide  half  the  rabatted  base  from  0  io  F  into  a 
number  of  equal  parts  i,  2,  3,  &c.,  and  the  half  of  the  seg- 
ment of  the  fixed  base  on  the  same  side  into  the  same  num- 
ber of  equal  parts  i',  2',  3',  &c. 

If,  now,  the  rabatted  circle  roll  so  that  i  coincides  with 
i'  the  position  of  Pwill  be  one  division  nearer,  and  by  aid 
of  a  new  auxiliary  vertical  plane  of  elevation  through  vi 
the  base  may  be  rotated  about  the  tangent  to  the  segment 
at  the  point  1',  and  a  second  point  /'  in  the  plan  of  the 
epicycloid  found. 

Similarly,  as  the  rabatted  base  rolls  to  the  points  2',  3', 
&c.,  other  points  in  this  half  of  the  curve  will  be  determined. 
The  other  half  may  be  found  by  a  similar  operation  on  that 
part  of  the  segment. 

Note.  The  student  will  readily  see  from  his  drawing 
that  the  vertical  plane  of  elevation  first  drawn  can  be  made 
to  serve  for  all  the  points  (i',  2',  3',  &c.),  and  thus  somewhat 
lessen  the  labour  of  construction. 

23.  To  determine  a  tangent  to  the  spherical  epicycloid  at 
a  point  in  the  curve. 

The  tangent  must  evidently  lie  in  the  tangent-plane  to 
the  sphere  on  which  the  curve  is  traced.  Also,  it  can  be 
easily  proved  that  the  tangent  lies  in  the  tangent-plane  to 
the  sphere  whose  centre  is  the  point  of  contact  of  the 
generating  circle,  and  radius  the  line  drawn  from  the  point 
of  contact  to  the  corresponding  position  of  the  tracing 
point. 


MISCELLANEOUS  PROBLEMS.  249 

Thus,  if  p'  were  the  plan  of  the  given  point,  the  tangent 
to  the  curve  would  be  the  common  section  of  a  tangent- 
plane  to  the  first-mentioned  sphere  at  that  point,  with  the 
tangent-plane  at  the  same  point  to  a  sphere  whose  centre  is 
1'  and  radius  the  true  length  of  \'p'. 

When  the  spheres  are  determined,  the  tangent-line  can 
be  drawn  through  P  at  right  angles  to  the  plane  of  the 
normals  to  the  two  spheres  at  that  point.  See  General 
Remarks,  "Tangent-Planes  and  Normals  to  Curved  Sur- 
faces," Chapter  vii. 


XIII. 
SOLUTIONS  OF  THE  TRIHEDRAL  ANGLE. 

("SPHERICAL   TRIANGLES.") 

Case  I. 

Given  the  three  faces  a,  b,  and  c  ;  to  determine  the  three 
dihedral  angles  A,  B,  and  C. 

Note,  In  'spherical  triangles'  the  dihedral  angles  A, 
B,  and  C  are  those  which  are  opposite  to  the  faces  a,  b, 
and  c,  each  to  each;  namely  A  \.o  a,  B  X.Q  b,  and  C  to  c. 

Let  the  three  faces  be  developed  on  the  horizontal  plane 
of  one  of  them,  viz.,  b. 

Take  a  plane  of  elevation,  xy,  at  right  angles  to  01,  the 
common  edge  of  the  faces  c  and  h;  and  let  it  meet  the 
second  edge  of  face  c  in  point  F^,  and  cut  01  in  e  and  Om 
in/ 

Make  OF^  on  the  second  edge  of  a  equal  to  OP^ ;  since 
OP^  and  OP„  are  really  the  same  line,  viz.,  the  common 
edge  of  faces  c  and  a. 

Then  eP^  ef,  fP^  are  the  developed  sections  of  the  three 
faces  by  the  vertical  plane  through  xy. 

Rabat  the  section  ep'f,  in  this  plane ;  taking  the  centre  e 
and  radius  eP„  and  centre  f  and  radius  /Pj,  and  describing 
arcs  intersecting  in  /' :  join  p'e  and  /'/  Then  p,  the  plan 
of  P,  is  determined  in  xy  from  /',  and  Op  is  the  plan  of 
the  edge  OP  on  the  plane  of  b.  Angle  p'ef  is  the  angle 
A.  The  plane  through  /  at  right  angles  to  Otn  gives  the 
angle  C.  And  the  third  angle  B  is  determined  by  a  plane 
through  P,  in  space,  at  right  angles  to  the  third  edge  OP, 
in  space,  by  Problem  15,  Chap.  in.  /;/;  is  the  horizontal 
trace  of  this  plane,  which  is  that  of  the  profile  angle  of  B. 


'^SPHERICAL    triangles:'  25 1 


Case  I. 


Given  tc.  5  ctnci  c. 


/\\y 

/ 

V 

\r 

/ 

\      / 

V 

N 

V       /\ 

"A 

'"■ 

N.  / 

'/-^  ^'> 

\ 

/          ^ 

e 

^ 

Vs 

r 

\       ^N 

\            ^s 

252         SOLID  OR  DESCRIPTIVE  GEOMETRY, 


Case  IL 

Given  the  faces  a,  b,  and  the  angle  C  :  to  determine  A,  B, 
and  c — /.  e.  two  faces  and  the  included  dihedral  angle. 

Let  the  face  a  be  developed  on  the  horizontal  plane  of 
b  :  since  the  angle  C  between  these  faces  is  known,  a  plane, 
xj,  at  right  angles  to  their  intersection  Om,  exhibits  this 
angle  Jfp' ;  fp'  being  the  vertical  trace  of  the  face  a,  the 
length^'  being  made  equal  to/P,,  /  in  xy,  determined  from 
/',  gives  Op,  which  produced  is  the  indefinite  plan  of  the 
third  edge  OP  on  the  plane  of  b.  A  plane  through  /  at 
right  angles  to  the  edge  01  gives  the  angle  A,  and  enables 
us  to  rabat  the  third  face  c  about  01 — its  horizontal  trace. 
A  plane  through  P  in  space,  and  at  right  angles  to  the  edge 
OP  in  space,  and  having  Im  for  its  horizontal  trace,  will 
determine  the  profile  angle  of  B. 


''SPHERICAL  TRIANGLES." 
Case  II. 


253 


Giren  a,  d  and  C. 


A\^ 


P, 


254  SOLID   OR  DESCRIPTIVE   GEOMETRY. 


Case  III. 

Giveji  the  dihedral  angles  B  and  C  and  the  face  a,  that  is, 
two  angles,  and  their  adjacent  face  :  to  detennine  the  angle  A 
and  tlie  faces  b  and  c. 

On  the  horizontal  plane  take  the  face  a,  of  which  01, 
Ofn  are  the  edges. 

Take  the  plane  xy  at  right  angles  to  01. 

The  vertical  trace  of  the  face  c  on  this  plane  forms  the 
angle,  B,  with  xy,  which  angle  being  known  this  trace  can 
be  drawn. 

On  a  plane  xj\,  at  right  angles  to  Oin,  the  vertical 
trace  of  face  b  forming  the  known  angle,  C,  with  x^y^  can  be 
drawn. 

By  the  aid  of  the  elevation  on  xy  determine  the  vertical 
trace  of  face  c  on  plane  x^y^ ;  the  two  vertical  traces  on  the 
latter  plane  intersect. in  a  point  i',  whence  /  is  found  in  xj^ 
and  //'  is  a  point  in  the  third  edge  01,  of  which  draw  the 
indefinite //<7«  01. 

The  faces  b  and  c  can  now  be  readily  rabatted  into  the 
plane  of  a  by  rotation  on  Om  and  01  respectively.  Angle 
A  is  found  by  Problem  15,  Chapter  iii.,  as  before,  by  a 
plane  through  point  P  ox  I  and  perpendicular  to  01. 


"  SPHERICAL   triangles:' 


255 


Case  III. 


Given  B ,  C  izncC  a. 


<c/ 


256  SOLID   OR  DESCRIPTIVE   GEOMETRY. 


Case  IV, 

Given  two  faces  a  and  b,  and  the  dihedral  angle,  A,  (fhat 
is,  two  faces,  and  an  angle  opposite  to  one  of  them) :  to  deter- 
mine the  dihedral  angles  B  and  C,  and  face  c. 

Assume  the  given  face  b  on  the  horizontal  plane,  edges 
01  and  Om,  and  face  a  rabatted  into  that  plane  about 
Om. 

On  plane  xy,  at  right  angles  to  01,  make  the  given  angle 
A  with  xy,  the  second  line  of  this  angle  is  the  vertical  trace 
of  face  c  on  this  plane. 

On  a  second  plane,  x.^y^,  assumed  at  right  angles  to  Om, 
determine  the  vertical  trace  ep'  of  the  face  c  by  the  aid  of 
the  former  elevation  :  cut  this  trace  in  /'  by  the  circular  arc 
described  on  plane  x^y^  by  point  P^  on  the  third  edge  : 
join  pf,  this  is  the  vertical  trace  of  face  a ;  obtain/  in  xj^, 
then  //'  is  a  second  point  determined  in  the  third  edge  OP 
of  the  spherical  triangle.  Draw  the  indefinite  plan  Op  and 
obtain  the  third  angle,  B,  as  before,  by  Problem  15,  Chapter 
III. 


'' SPHERICAL    TRIANGLES. 


257 


Case  IV. 


Giren  a,  "b   and  A 


E.G. 


17 


258  SOLID  OR  DESCRIPTIVE   GEOMETRY. 


Case  V, 

Given  the  dihedral  angles  B  and  C,  and  a  face  b  opposite 
to  one  of  the7n :  to  determine  faces  a  and  c,  and  the  third 
dihedral  angle  A. 

Let  the  face  b  be  drawn  on  the  horizontal  plane,  and 
revolved  about  its  edge  Oni  until  it  makes  the  angle  C  with 
that  plane. 

The  indefinite  line^'  will  "then  be  the  vertical  trace  of 
face  b  on  the  plane  xy  of  the  profile  angle  of  C ;  which  is, 
of  course,  perpendicular  to  Om :  and  fp'  on  this  trace, 
taken  equal  to  /P,,  gives  the  elevation,  /',  of  a  point  P  in 
the  second  edge  of  face  b.  p  in  xy,  obtained  from  p\ 
determines  the  pointy'  in  this  second  edge  :  and  Op  is  the 
plan  of  that  edge. 

Determine  a  plane  to  contain  the  line  OP,  in  space, 
just  found,  and  inclined  at  the  known  angle  of  B,  by 
Problem  9,  Chapter  iii. :  01,  the  horizontal  trace  of  this 
plane,  is  the  third  edge  of  the  required  spherical  triangle. 

The  face  c  can  now  be  rabatted  on  the  plane  of  a 
about  01.  The  angle  A  may  be  determined  as  before, 
by  a  plane  at  right  angles  to  OP  through  point  P. 


"  SPHERICAL    triangles:' 


259 


Case  V. 


Given  B .  Cf  an^  t. 


\ 


260  SOLID   OR  DESCRIPTIVE   GEOMETRY. 


Case  VI. 

Given  the  three  dihedral  angles  A,  B,  and  C  .*  to  determine 
the  three  faces  a,  b,  and  c. 

If  the  dihedral  angle  C  has  one  face  in  the  horizontal 
plane,  the  traces  of  a  second  face  can  be  readily  drawn ; 
and  of  these  the  horizontal  trace  is  an  indefinite  edge  of  the 
required  spherical  triangle. 

A  third  plane  making  the  angle  of  A  with  its  horizontal, 
and  of  i8o-^  with  the  inclined  plane,  determined  by 
Problem  26,  Chap,  in.,  will  complete  the  required  spherical 
triangle.  The  intersections  01,  OP,  of  this  third  plane  with 
the  planes  of  the  angle  C,  form  the  second  and  third  edges 
of  the  spherical  triangle.  The  face  b  will  thus  be  deter- 
mined in  the  horizontal  plane;  into  which  the  remaining 
faces  a  and  c  can  be  rabatted  about  their  respective  edges 
or  horizontal  traces  into  the  plane  of  the  face  b. 


"  SPHERICAL   triangles:' 


261 


Case  YI. 


Given  A.  B  cmd  C. 


Cambrftrge : 

PRINTED   BY  C.    J.  CLAV,    M.A. 
AT   THE  UNIVERSITY  PRESS. 


MATHEMATICAL  WORKS. 

BY  BARNARD   SMITH,   M.A. 

ARITHMETIC  AND  ALGEBRA,     los.  6d. 

ARITHMETIC  FOR  THE  USE  OF  SCHOOLS.    4^.  6d. 
KEY,  8s.  6d. 

SHILLING  BOOK  OF  ARITHMETIC.     For  National 

and  Elementary  Schools.  i8mo.  \s.  Or  separately,  Part  I.,  id.  ; 
Part  II.,  id.',  Part  III.,  7^.  With  ANSWERS,  is.  (yd.  KEY, 
4J.  6d. 

SCHOOL   CLASS-BOOK  OF  ARITHMETIC.     i8mo. 
3^.     Or  in  Three  Parts,  u.  each.    KEY,  Three  Parts,  2^.  6d.  each. 


ARITHMETIC  IN  THEORY  AND   PRACTICE.     By 
J.  BROOK-SMITH.     4^.  (>d. 

EXERCISES  IN  ARITHMETIC.     By  S.  PEDLEY.     5^. 

With  over  7,000  Examples. 

ALGEBRAICAL  EXERCISES.     Progressively   arranged. 
By  C.  A.  JONES,  M.A.,  and  C.  H.  CHEYNE,  M.A.     is.  6d. 


BY  T.   DALTON,   M.A. 

RULES  AND  EXAMPLES  IN  ARITHMETIC.     2s.  6d. 

RULES  AND  EXAMPLES  IN  ALGEBRA.     Part  I.,  2s.; 
Part  II.,  IS.  6d. 

BY  L  TODHUNTER,   M.A.,  F.R.S. 

:UCLID  FOR  COLLEGES  AND  SCHOOLS.     3^.  ed. 

;  lENSURATION  FOR  BEGINNERS.     2s.  6d. 

iXGEBRA    FOR    BEGINNERS.     With  numerous   Ex- 
amples.    IS.  6d.    KEY,  6s.  6d. 

T  RIGONOMETRY  FOR  BEGINNERS.     2s.  ed     KEY, 


MATHEMATICAL    WORKS. 

MECHANICS  FOR  BEGINNERS    4^-.  bd.    KEY,  ds.  6d. 

ALGEBRA    FOR    TI^E    USE   OF   COLLEGES  AND 
SCHOOLS.     75.  6d.   ■  KEY,  10s.  6d. 

THE  THEORY  OF  EQUATIONS.    -7^.  6d. 

PLANE  TRIGONOMETRY.     5^-.     KEY,  10s.  6d 


GEOMETRICAL  NOTE-BOOK,  containing  easy  Pro- 
blems in  Geometrical  Drawing  preparatory  to  the  Study  of 
Geometry.  By  F.  E.  KITCHENER,  M.A.,  Mathematical  Master 
at  Rugby.     410.  2s. 


BY  J.    M.   WILSON,   M.A. 
RY  GEOMETRY.     Books   1 

arged.     ^s.  6d. 

SOLID  GEOMETRY  AND  CONIC  SECTIONS.    4^.  Gd 


ELEMENTARY  GEOMETRY.     Books   I.  to  V.     New 

Edition,  Enlarged.     4J.  6d. 


NATURAL  GEOMETRY:  an  Introduction  to  the  Logical 
Study  of  Mathematics.  With  Explanatory  Models.  By  A. 
Mault.     is. 

Models  to  illustrate  the  above,  in  Box,  12s.  6d. 

PLANE  TRIGONOMETRY:  Graduated  Exercises  in. 
By  J.  WILSON,  M.A.,  and  S.  R.  WILSON,  B.A.     4J-.  6d. 

ELEMENTS  OF  DESCRIPTIVE  GEOMETRY.  By 
J.  B.  MILLAR,  C.E.    6s, 

CONIC  SECTIONS:  a  Gepmetrical  Treatise.  By  W.  H 
DREW.     Crown  8vo.  5J.— SOLUTIONS,  4J.  6d. 

AN  ELEMENTARY  TREATISE  ON  CONIC  SEC 
TIONS  and  ALGEBRAIC  GEOMETRY.  By  G.  H.  BUCKLE 
M.A.     7j.  6d. 

MATHEMATICAL  PROBLEMS.    Devised  a 
by  J.  WOLSTENHOt^tt:.  •  New  Edition, 

396         O 


PLEASE  DO  NOT  REMOVE 
CARDS  OR  SLIPS  FROM  THIS  POCKET 


UNIVERSITY  OF  TORONTO  LIBRARY 


p&ASc2,"